Download as pdf or txt
Download as pdf or txt
You are on page 1of 68

Solution & Colligative Properties

 Marked Questions may have for Revision Questions.


 fpfUgr iz
'u nksgjkus;ksX; iz'u gSA
PART - I : SUBJECTIVE QUESTIONS
Hkkx - I : fo"k;kRed iz'u ¼SUBJECTIVE QUESTIONS½
Section (A) : Concentration terms (Revision of mole)
[k.M (A) : lkUnzrk in (eksy d k la'kks/ku)

Commit to memory :

no. of moles of solute no. of moles of solute


Molarity = ; Molality = × 1000
volume of solution (in L) W(g) of solvent

molarity  1000
Molality = 1000d  molarity  m.wt.of solute (where d is density of solution in g/ml)

w m (g) n
%  solute  100 ; xsolute = (where, n is moles of solute, N is moles of solvent)
w msolution (g) nN
xsolute + xsolvent = 1

Commit to memory :

foy s; d seksy ksad h la[; k foy s; d seksy ksad h la[; k


eksy jrk = foy; u d k vk; ru (L es)a ; eksy y rk= foyk; d d k Hkkj (g) × 1000

eksy jrk 1000


eksy y rk= 1000d  eksy jrk foy s; d k v.kqHkkj (t gk¡d foy ;u d k ?kuRo g/ml esagSA)

w m (g) n
%  foy s;  100 ; xfoy s; = (t gk¡
, n foy s
; d seksy gS] N foy k;d d seksy gSA)
w mfoy ;u (g) nN

xfoy s; + xfoy k;d = 1

A-1. Calculate the concentration of NaOH solution in g/ml which has the same molarity as that of a solution of
HCl of concentration 0.0365 g/ml.
ml NaOH foy ;u d h g/ml esalkUnzrk ifjd fy r d hft ,] ft ld h eksy jrk 0.0365 g/ml lkUnzrk oky sHCl foy ;u d s
leku gSA
Ans. 0.04 g/mL

A-2. The density of 3M solution of sodium thiosulphate (Na2S2O3) is 1.58 g/ml. Calculate
(i) amount of Na2S2O3 in % w/w
(ii) mole fraction of Na2S2O3
(iii) molality of Na+ and S2O32- ions.
lksfM;e Fkk;ksl YQ sV (Na2S2O3) d s3 M foy ;u d k ?kuRo 1.58 g/ml gSA ifjd fy r d hft ,A
(i) % w/w es
aNa2S2O3 d h ek=kk
(ii) Na2S2O3 d k eks
y
izHkkt
(iii) Na o S2O3 vk;uks
+
ad h eksy y rk
2-

Ans. (i) 30%, (ii) 0.046 (iii) Na+ = 5.42 m, S2O32– = 2.71 m
Corporate Office: CG Tower, A-46 & 52, IPIA, Near City Mall, Jhalawar Road, Kota (Raj.)-324005
Website : www.resonance.ac.in | E-mail : contact@resonance.ac.in
Toll Free : 1800 200 2244 | 1800 258 5555 | CIN: U80302RJ2007PLC024029
ADV-SCP -1
Solution & Colligative Properties
A-3. Calculate the molality and molarity of a solution made by mixing equal volumes of 30% by weight of H2SO4
(density = 1.20 g/mL) and 70% by weight of H2SO4 (density = 1.60 g/mL).
H2SO4 (?kuRo = 1.20 g/mL) d s30% (w/w) rFkk H2SO4 (?kuRo = 1.60 g/mL) d s70% (w/w) d sleku vk;ruks ad ks
fey kd j cuk, x;sfoy ;u d h eksy y rk o eksy jrk d h x.kuk d hft ,A
Ans. Molality eks
y y rk= 11.44 m, Molarity eksy jrk= 7.55 M
Section (B) : General Introduction & types of solution
[k.M (B) : lkekU; çLrkouk o foy ;u d sçd kj
B-1. What are the characteristics of the supersaturated solution.
vfrlarÌr foy ;u d svfHky {k.k crkb;s\
Sol. Some of the characteristics of supersaturated solution are given below
(i) If a crystal of solute is added to supersaturated solution, crystallisation occurs rapidly.
(ii) Supersaturated solutions contain more solute than they should have at a particular temperature.
gy - d qN vfrlarÌr foy ;u d svfHky {k.k uhpsfn;sx;sgS%
(i) ;fn vla rÌr foy ;u esafoy s; d sfØ LVy fey krsgS] rc fØ LVy hd j.k 'kh?kzrk lsgksrk gSA
(ii) fuf'pr rkieku ij vfrla rÌr foy ;u esafoy s; inkFkZlarÌr foy ;u d h rqy uk esavf/kd ek=kk esamifLFkr gksrsgSA
B-2. What do you mean by Hygroscopic compound ?
vknzZrkxzkgh ;kSfxd kslsvki D;k le>rsgS\
Sol. Certain compounds combine with the moisture of atmosphere and are converted into hydroxides or hydrates.
Such substances are called hygroscopic. e.g., anhydrous CuSO4, quick lime (CaO), anhydrous Na2CO3
etc.
gy . d qN ;kSfxd ok;qe.My d h uehd slkFk la;ksft r gksrsgSrFkk ;g gkbMªkWDlkbM ;k gkbMªsV esaifjofrZr gksrsgSA bl izd kj
d sinkFkZvknzZrkxzkgh d gy krsgSmnkgj.k fut Zy CuSO4, fcuk cq> k pwuk (CaO), fut Zy h; Na2CO3 vkfnA
B-3. Why are some solution processes exothermic whereas others are endothermic ?
d qN foy ;u izØ e Å "ek{ksih gksrsgSA t cfd vU; Å "ek'kks"kh gksrsgS] D;ksa\
Sol. The overall energy change associated with dissolution depends on the relative magnitude of the solute–
solute , solvent–solvent and solute–solvent interactions. The process is exothermic if the new interaction
release more energy than disrupting the old interactions requires, it is endothermic if opposite is true.
gy . foy k;d u lslEcfU/kr lEiw.kZÅ t kZifjorZu foy s;&foy s;]foy k;d &foy k;d rFkk foy s;&foy k;d vU;ksU; fØ ;kvksad s
vkisf{kd ifjek.k ij fuHkZj d jrk gSA ;g izØ e Å "ek{ksih rc gksrk gSat c LFkkfir gksusoky h u;h vU;ksU; fØ ;k lsigy s
oky h vU;ksU; fØ ;k d ksvLr&O;Lr fd ;sfcuk vf/kd Å t kZeqDr gksrh gSA bld k foijhr gksusij ;g izØ e Å "ek'kks"kh
gkst k;sxkA
Section (C) : Vapour Pressure
[k.M (C) : ok"i nkc

Commit to memory :
* The partial pressure of vapours of X in equilibrium with X at a given temperature is called as its vapour
pressure. (X is given pure solid or pure liquid)
* A gas is said to be saturated with vapours of a liquid if the partial pressure of liquid vapours is equal to its
(saturated) vapour pressure.
* Partial pressure of vapours of X (pure solid or liquid) will remain equal to its vapour pressure till the
vapours of X are in equilibrium with X.

Corporate Office: CG Tower, A-46 & 52, IPIA, Near City Mall, Jhalawar Road, Kota (Raj.)-324005
Website : www.resonance.ac.in | E-mail : contact@resonance.ac.in
Toll Free : 1800 200 2244 | 1800 258 5555 | CIN: U80302RJ2007PLC024029
ADV-SCP -2
Solution & Colligative Properties

Commit to memory :
* fn;sx;srki ij X d slkFk lkE; esaok"i d k vkaf'kd nkc ok"i nkc d gy krk gSA (X fn;k x;k 'kq) Bksl ;k 'kq) nzo
gSA)
* ,d xSl t ksnzo d sok"i d slkFk larÌr d gy krh gS;fn nzo ok"i d k vkaf'kd nkc bld s(larÌr) ok"i nkc d scjkcj
gksrk gSA
* X ('kq
) Bksl ;k nzo) d sok"i d k vkaf'kd nkc bld h ok"i nkc d st c rd cjkcj jgsxk t c rd fd X d h ok"i X d s
lkFk lkE; esagksrh gSA
C-1. The vapour pressure of water at 80ºC is 355 torr. A 100 ml vessel contained watersaturated oxygen at
80º C, the total gas pressure being 760 torr. The contents of the vessel were pumped into a 50.0 ml, vessel
at the same temperature. What were the partial pressures of oxygen and of water vapour and the total
pressure in the final equilibrium state ? Neglect the volume of any water which might condense.
80ºC ij t y d k ok"i nkc 355 Vksj gSA 80ºC ij ,d 100 ml d sik=k esat y &larÌr vkWDlht u mifLFkr gSA rc xSl
d k d qy nkc 760 Vksj gSA leku rki ij ik=k d svo;oksad ks,d 50.0 ml ik=k esaLFkkukUrfjr fd ;k x;k rc vkWDlht u
rFkk t y ok"i d k vkaf'kd nkc D;k gksxk \ vfUre lkE;koLFkk ij d qy nkc D;k gS\ t y d sla?kfur vk;ru d ksux.;
ekfu;sA
Ans. PO2 = 810 mm Hg, PH2O = 355 mm Hg , Ptotalal = 1165 mm Hg

C-2. A vessel has nitrogen gas and water vapours in equilibrium with liquid water at a total pressure of 1 atm.
The partial pressure of water vapours is 0.3 atm. The volume of this vessel is reduced to one third of the
original volume, at the same temperature, then find total pressure of the system. (Neglect volume occupied
by liquid water)
ukbVªkst u xSl o t y ok"i ;qDr ,d ik=k 1 atm d qy nkc ij nzo t y d slkFk lkE; esagSA t y ok"i d k vkaf'kd nkc
0.3 atm gS
A leku rki ij bl ik=kd svk;ru d ksbld sokLrfod vk;ru ls,d frgkbZrd d e d j nsrsgSA rc fud k;
d k d qy nkc D;k gksxk \ (nzo t y }kjk ?ksjk x;k vk;ru ux.; gSA)
Ans. 2.4 atm

Section (D) : Solutions of Solid and Gases in Liquids


[k.M (D) : nzo esaBksl rFkk xSl ksad k foy ;u

Commit to memory :
* Henry’s law : The solubility of gas in a liquid at a given temperature is directly proportional to its partial
pressure above liquid in which it is dissolved.
P = KHx (where x is mole fraction of unreacted, dissolved gas and P is its partial pressure above liquid.)

Commit to memory :
* gsujh fu;e : fn;sx;srki ij nzo esaxSl d h foy s;rk nzo ij bld svkaf'kd nkc d slekuqikrh gksrh gSA blesa;g
fo;ksft r gksrk gSA
P = KHx (t gk¡x vfØ ;k'khy ] fo;ks
ft r xSl d k eksy izHkkt gSrFkk P nzo ij bld k vkaf'kd nkc gSA)
D-1. The partial pressure of ethane over a solution containing 6.56 × 10–3 g of ethane is 1 bar. If the solution
contains 5.00 × 10–2 g of ethane, then what shall be the partial pressure of the gas ?
6.56 × 10–3 g ,Fks
u ;qDr ,d foy ;u esa,Fksu d k vkaf'kd nkc 1 bar gSA ;fn foy ;u esa5.00 × 10–2 g ,Fksu gks]rksxSl
d k vkaf'kd nkc D;k gksxk \
Ans. 7.62 bar

Corporate Office: CG Tower, A-46 & 52, IPIA, Near City Mall, Jhalawar Road, Kota (Raj.)-324005
Website : www.resonance.ac.in | E-mail : contact@resonance.ac.in
Toll Free : 1800 200 2244 | 1800 258 5555 | CIN: U80302RJ2007PLC024029
ADV-SCP -3
Solution & Colligative Properties
D-2. If N2 gas is bubbled through water at 293 K, how many millimoles of N2 gas would dissolve in 1 litre of water,
if N2 exerts a partial pressure of 0.987 bar. Given that Henry's law constant for N2 at 293 K is 76.48 kbar.
;fn N2 xSl d ks293 K ij t y esalsçokfgr fd ;k t k, rks,d y hVj t y esafd rusfey heksy N2 xSl foy ; gksxh\ ;fn
N2 d k vka
f'kd nkc 0.987 bar gSrFkk293 K ij N2 d sfy , gsujh fu;e fu;rkad (KH) d k eku 76.48 kbar gSA
Ans. 0.716 mmol.

Section (E) : Immiscible Liquids


[k.M (E) : v feJ.kh; nzo

Commit to memory :
If A and B are volatile immiscible liquids, then above their mixture.
* PT = PºA + PºB (PT = Total pressure above mixture of A and B, PºA = vapour pressure of A,
PºB = vapour pressure of B)

PAº nA
= n (nA and nB are moles of A and B in distillate)
PBº B

w A PA MA
 (wA and wB are masses of A and B in distillate, MA and MB are molar masses of A and B)
w B PB MB

Commit to memory :
;fn A rFkk B ok"i'khy vfeJ.kh; nzo gSrksbud k feJ.kA
* PT = PºA + PºB (PT = A rFkk B d sfeJ.k ij d q
y nkc, PºA = A d k ok"i nkc, PºB = B d k ok"i nkc)
PAº nA
= n (nA rFkk nB vklq
r esaA rFkk B d seksy )
PBº B

w A PA MA
 (wA rFkk wB vklq
r esaA rFkk B d snzO;eku gS] MA rFkk MB, A rFkk B d seksy j nzO;eku gSA)
w B PB MB

E-1. A mixture of an organic liquid A and water distilled under one atmospheric pressure at 99.2ºC. How many
grams of steam will be condensed to obtain 1.0 g of liquid A in the distillate ? (Vapour pressure of water at
99.2ºC is 739 mm Hg. Molecular weight of A = 123)
99.2ºC rFkk ,d ok;qe.My h; nkc ij ,d d kcZfud nzo A rFkk t y d sfeJ.k d ksvklfor fd ;k t krk gSA vklou esa
nzo A d s1 xzke izkIr d jusd sfy , Hkki d sfd rusxzke d ksla?kfur fd ;k t k;s\ (99.2ºC ij t y d k ok"inkc 739 mm
Hg rFkkA d k v.kq Hkkj = 123)
Ans. 5.15 g
E-2. Boiling point of a mixture of water and nitrobenzene is 99°C, the vapour pressure of water is 733 mm of Hg
and the atmospheric pressure is 760 mm of Hg. The molecular weight of nitrobenzene is 123. Find the ratio
of weights of the components of the distillate.
t y rFkk ukbVªkscsat hu d sfeJ.k d k DoFkukad 99°C gS] t y d k ok"i nkc 733 mm of Hg gSrFkk ok;qe.My h; nkc
760 mm Hg gS
A ukbVªkscsat hu d k v.kqHkkj 123 gSA vklfor fd , x;svo;oksad sHkkjksad k vuqikr Kkr d hft ,A
WW
Ans. WN = 3.973

Corporate Office: CG Tower, A-46 & 52, IPIA, Near City Mall, Jhalawar Road, Kota (Raj.)-324005
Website : www.resonance.ac.in | E-mail : contact@resonance.ac.in
Toll Free : 1800 200 2244 | 1800 258 5555 | CIN: U80302RJ2007PLC024029
ADV-SCP -4
Solution & Colligative Properties
Section (F) : Completely miscible liquids : Raoult’s law
[k.M (F) : iw.kZr% feJ.kh; nzo % jkWÅ YV fu;e

Commit to memory :
Statement of Raoult's law ( for volatile liq. mixture ): In solution of volatile liquids, the partial vapour
pressure of each component is directly proportional to its mole fraction in solution.
pA  xA
pA = xAPAº ....... (1)
where pA = Partial vapour pressure of component A, xA = Mole fraction of component ‘A’ in solution,
PAº = Vapour pressure of pure component ‘A’ at given temperature
Hence PT = xAPAº + xBPBº ....... (2)
(PT = Total pressure of vapour above solution, PBº = vapour pressure of pure component 'B' at given temperature.)
1 y A yB
Thus, P = o + o (for complete derivation, refer page no. 12 of sheet)
T PA PB
(yA = mole fraction of A in vapour phase above the solution and yB = mole fraction of B in vapour phase
above the solution)

Commit to memory :
jkWÅ YV fu;e d k d Fku (ok"i'khy nzo feJ.k d sfy ,): ok"i'khy nzoksad sfoy ;u esaizR;sd ?kVd d k vkaf'kd ok"i nkc
foy ;u esabld seksy izHkkt d slekuqikrh gksrk gSA
pA  xA
pA = xAPAº ....... (1)
t gk¡pA = ?kVd A d k vkaf'kd ok"i nkc, xA = foy ;u esa?kVd ‘A’ d k eksy izHkkt , PAº = fn;sx;srki ij 'kq) ?kVd
‘A’ d k ok"i nkc
vr% PT = xAPAº + xBPBº ....... (2)
(PT = foy ;u ij d qy ok"i nkc, PBº = fn;sx;srki ij 'kq) ?kVd 'B' d k ok"i nkc)
1 yA yB
blizd kj, P = o + o (iw.kZfopy u d sfy , 'khV d k ist ua- 12 ns[ksaA)
T PA PB
(yA = foy ;u ij ok"i iz
koLFkk esaA d k eksy izHkkt rFkk yB = foy ;u ij ok"i izkoLFkk esaB d k eksy izHkkt )

F-1. Two liquids A and B form an ideal solution. At 300 K, the vapour pressure of a solution containing 1 mole of
A and 3 moles of B is 550 mm of Hg. At the same temperature, if one mole of B is added to this solution, the
vapour pressure of the solution increases by 10 mm of Hg. Determine the vapour pressure of A and B in
their pure states.
nksnzo A rFkkB ,d vkn'kZfoy ;u cukrsgaSA 300 K rki ij A d s1 eksy rFkkB d s3 eksy ;qDr ,d foy ;u d kok"inkc
550 mm Hg gS
A leku rki ij B d k ,d eksy bl foy ;u esafey k;k t krk gS]rc foy ;u d k ok"i nkc 10 mm Hg
lsc< t krk gSA rc 'kq) voLFkkvksaessaA rFkk B d sok"i nkc d h x.kuk d hft ,A
Ans. pA0 = 400 mm of Hg, pB0 = 600 mm of Hg
F-2. Two liquids, A and B, form an ideal solution. At the specified temperature, the vapour pressure of pure A is
200 mm Hg while that of pure B is 75 mm Hg. If the vapour over the mixture consists of 50 mol percent A,
what is the mole percent A in the liquid ?
nksnz
o A rFkkB vkn'kZfoy;u cukrsgSA fd lhfuf'pr rki ij 'k)q A d kok"i nkc 200 mm Hg t cfd 'k)q B d kok"i nkc
75 mm Hg gS A ;fn feJ.k d sÅ ij ok"i es
a50 ekys iz
fr'kr A gS
]rksnzo esaA d sekys iz
fr'kr fd rusgS
a\
Ans. 27.3 mole %

Corporate Office: CG Tower, A-46 & 52, IPIA, Near City Mall, Jhalawar Road, Kota (Raj.)-324005
Website : www.resonance.ac.in | E-mail : contact@resonance.ac.in
Toll Free : 1800 200 2244 | 1800 258 5555 | CIN: U80302RJ2007PLC024029
ADV-SCP -5
Solution & Colligative Properties
F-3. Two solutions of A and B are available. The first is known to contain 1 mole of A and 3 moles of B and its
total vapour pressure is 1.0 atm. The second is known to contain 2 moles of A and 2 moles of B; its vapour
pressure is greater than 1 atm, but it is found that this total vapour pressure may be reduced to 1 atm by the
addition of 6 moles of C. The vapour pressure of pure C is 0.80 atm. Assuming ideal solutions and that all
these data refer to 25°C, calculate the vapour pressure of pure A and of pure B.
A o B d snksfoy ;u miy C/k gS
A çFke Kkr foy ;u esaA d k1 eksy o B d srhu eksy fLFkr gSrFkk bld k d qy ok"i nkc
1.0 atm gS
A f}rh; Kkr foy ;u esaA d s2 eksy o B d s2 eksy fLFkr gSrFkk bld k ok"i nkc 1 atm lsvf/kd gSA y sfd u
ik;k x;k fd C d s6 eksy fey kusij bld k d qy ok"i nkc 1 atm rd ?kV t krk gSA 'kq) C d k ok"i nkc 0.80 atm gSA
vkn'kZfoy ;u ekursgq;slHkh vk¡d Ms25°C lslEcfU/kr gSA 'kq) A o 'kq) B d k ok"i nkc ifjd fy r d hft , \
Ans. PB0 = 0.7 atm o PA0 = 1.9 atm

F-4. At 80oC, the vapour pressure of pure benzene is 753 mm Hg and of pure toluene 290 mm Hg. Calculate the
composition of a liquid in mole per cent which at 80oC is in equilibrium with the vapour containing 30 mole
per cent of benzene.
80oC ij 'kq
) csat hu o 'kq) VkWy qbZu d k ok"inkc Ø e'k%753 mm Hg o 290 mm Hg gSA nzo d slaxBu d h x.kuk eksy
izfr'kr esaKkr d hft , t ks80oC ij 30 eksy izfr'kr csUt hu ;qDr ok"i d slkFk lkE;koLFkk esagSA
Ans. 14.16 mole percent benzene (14.16 eks y izfr'kr csat hu)
F-5. Vapour pressure of C6H6 and C7H8 mixture at 50ºC is given P (mm Hg) = 180XB + 90, where XB is the mole
fraction of C6H6. A solution is prepared by mixing 936 g benzene and 736 g toluene and if vapours over this
solution are removed and condensed into liquid and again brought to the temperature 50ºC, what would be
mole fraction of C6H6 in the vapour state. (At. wt. of C = 12, H = 1)
50ºC ij C6H6 rFkkC7H8 feJ.kd k ok"i nkc]P (mm Hg) = 180XB + 90, fn;kx;kgS A t gk¡XB, C6H6 d keksy izHkkt
gSA ,d foy ;u ft ls936 g csat hu rFkk 736 g VkWy qbZu d ksfefJr d jd scuk;k x;k gSrFkk ;fn bl foy ;u ij ok"i
d ksgVk fn;k t k;srFkk blsnzo esala?kfur d j fn;k t k, rFkk ;g iqu% 50ºC rki ij igq¡p t krk gS] rc ok"i voLFkk
esaC6H6 d k eksy izHkkt D;k gksxk \ (C d k ijek.kqHkkj = 12, H = 1)
Ans. Y’B = 0.932.

Section (G) : Non-ideal Solutions


[k.M (G) : v ukn'kZfoy ;u

Commit to memory :
* +ve deviation –ve deviation
PT.exp > (xAPºA + xBPºB) PT.exp < (xAPºA + xBPºB)
(where PT.exp is experimental total pressure above mixture of volatile liquids A and B).
Hmix = +ve Hmix = –ve
Vmix = +ve Vmix = –ve
Smix = +ve Smix = +ve
Gmix = –ve Gmix = –ve

Commit to memory :
* +ve fopy u –ve fopy u
PT.exp > (xAPºA + xBPºB) PT.exp < (xAPºA + xBPºB)
(t gk¡PT.exp ok"i'khy nzoksaA rFkk B d sfeJ.k ij izk;ksfxd d qy nkc gSA).
HfeJ.k = +ve HfeJ.k = –ve
VfeJ.k = +ve VfeJ.k = –ve
SfeJ.k = +ve SfeJ.k = +ve
GfeJ.k = –ve GfeJ.k = –ve

Corporate Office: CG Tower, A-46 & 52, IPIA, Near City Mall, Jhalawar Road, Kota (Raj.)-324005
Website : www.resonance.ac.in | E-mail : contact@resonance.ac.in
Toll Free : 1800 200 2244 | 1800 258 5555 | CIN: U80302RJ2007PLC024029
ADV-SCP -6
Solution & Colligative Properties
G-1. A non ideal solution was prepared by mixing 30 ml chloroform and 50 ml acetone. Comment on volume of
mixture.
30 ml Dy ks
jksQ kWeZo 50 ml ,lhVksu d ksfefJr d jd s,d vu~& vkn'kZfoy ;u cuk;k x;kA feJ.k d k vk;ru Kkr
d hft ,A
Ans. V < 80ml.
G-2. Total vapour pressure of mixture of 1 mole of volatile component A (PºA = 100 mm Hg) and 3 mole of volatile
component B (PºB = 80 mm Hg) is 90 mm Hg. Find out nature of solution and sign of entropy of solution.
1 eks
y ok"i'khy ?kVd A (PºA = 100 mm Hg) o 3 eksy ok"i'khy ?kVd B (PºB = 80 mm Hg) d sfeJ.k d k d qy ok"i
nkc 90 mm Hg gSA foy ;u d h çd f̀r o foy ;u d h ,UVªkWih d k fpUg Kkr d hft ,A
Ans. There is positive deviation from Raoult’s law, S > 0.
;g jkWÅ YV fu;e ls/kukRed fopy u n'kkZrk gS] S > 0.
Section (H) : Degree of Ionisation/Dissociation for Weak Electrolytes
[k.M (H) : nqcZy oS| qr v i?kV~;ksad sv k;uu@fo;kst u d h ek=kk

Commit to memory :
* For dissociation / association i = 1 + (n – 1) 
(where n is total number of particles produced per solute particle after association / dissociation, i = vant
Hoff factor,  = degree of dissociation / association.
Moles of solute particles after association / dissociation of X Observed value of colligative property
i= Moles of X without association / dissociation
= Theoretical value of that colligative property

Theoretical molar mass of solute


i = Observed molar mass of solute

Commit to memory :
* fo;kst u @ laxq.ku d sfy , i = 1 + (n – 1) 
(t gk¡n laxq.ku @ fo;kst u d si'pkr izfr foy s; d .k lsmRiUu d .kksad h d qy la[;k gS, i = okUV gkWQ xq.kkad ,
 = fo;kst u @ laxq.ku d h ek=kkA
X la
xq.ku@fo; kst u d si'pkr ~foy s; d .kksad seksy v.kql a[; d xq.k/keZd k izsf{kd eku
i= l axq.ku@fo; kst u d svfrfjDr X d seksy = bl h v.kq
l a[; d xq.k/keZd k lS) kfUrd eku
foy s; d k lS) kfUrd eksy j nzO; eku
i=
foy s; d k izsf{kr eksy j nzO; eku

H-1. Complete the following table. (Use : i = 1 + (n – 1))

Dissociation / association
Solute Degree of dissociation / association n i
reaction
KCl 1

H2SO4 1
CH3COOH
0.2
(in water)
CH3COOH
0.5
(in benzene)
Urea
NaBr 0.8

A 3A  A3 1

Corporate Office: CG Tower, A-46 & 52, IPIA, Near City Mall, Jhalawar Road, Kota (Raj.)-324005
Website : www.resonance.ac.in | E-mail : contact@resonance.ac.in
Toll Free : 1800 200 2244 | 1800 258 5555 | CIN: U80302RJ2007PLC024029
ADV-SCP -7
Solution & Colligative Properties
fuEu lkj.kh d ksiw.kZd hft ,A (iz
;ksx d js: i = 1 + (n – 1))

foy s
; fo;ks
tu @ la
xq.ku v fHkfØ ;k fo;ks
tu @ la
xq.ku d h ek=kk n i
KCl 1

H2 SO 4 1
CH3COOH
0.2
(t y es
)a
CH3COOH
0.5
(cs
Ut hues
)a
;w
fj;k
NaBr 0.8

A 3A  A 3 1

Dissociation / association
Solute Degree of dissociation / association n i
Sol. reaction
KCl KCl  K+ + Cl– 1 2 2
H2SO4 H2SO4  2H+ + SO42– 1 3 3
CH3COOH + –
CH3COOH  H + CH3COO 0.2 2 1.2
(in water)
CH3COOH
2CH3COOH  (CH3COOH)2 0.5 0.5 0.75
(in benzene)
Urea No association or dissociation No association or dissociation Not defined 1
NaBr NaBr  Na+ + Br– 0.8 2 1.8
A 3A  A3 1 0.33 0.33

foy s
; fo;ks
tu @ la
xq.ku v fHkfØ ;k fo;ks
tu @ la
xq.ku d h ek=kk n i
KCl KCl  K + + Cl– 1 2 2
H2SO4 H2SO4  2H+ + SO42– 1 3 3
CH3COOH
CH3COOH  H+ + CH3COO– 0.2 2 1.2
(t y es
)a
CH3COOH
2CH3COOH  (CH3COOH)2 0.5 0.5 0.75
(cs
Ut hues
)a
;w
fj;k la
xq.ku @ fo;ks
t u ughagks
rkA la
xq.ku @ fo;ks
t u ughagks
rkA ifjHkkf"kr ughagksld rkA 1
NaBr + 0.8 2 1.8
NaBr  Na + Br –

A 3A  A 3 1 0.33 0.33

H-2. Calculate the percentage degree of dissociation of an electrolyte XY2 (Normal molar mass = 164) in water
if the observed molar mass by measuring elevation in boiling point is 65.6.
t y esa,d fo|qrvi?kV~; XY2 (lkekU; eksy j nzO;eku = 164) d h çfr'kr fo;kst u ek=kk ifjd fy r d hft ,A ;fn
DoFkukad esamUu;u }kjk ekfir çsf{kr eksy j nzO;eku 65.6 gSA
Ans. 75%

Corporate Office: CG Tower, A-46 & 52, IPIA, Near City Mall, Jhalawar Road, Kota (Raj.)-324005
Website : www.resonance.ac.in | E-mail : contact@resonance.ac.in
Toll Free : 1800 200 2244 | 1800 258 5555 | CIN: U80302RJ2007PLC024029
ADV-SCP -8
Solution & Colligative Properties
Section (I) : Relative lowering of vapour pressure
[k.M (I) : ok"i nkc d k v kisf{kd v oueu

Commit to memory :

Pº  PS Pº  Ps n
* RLVP = = xsolute ; Ps =
Pº N
(where Pº = vapour pressure of pure solvent, Ps = partial pressure of vapour above solution, n = dissolved
moles of solute, N = moles of solvent.)
Pº  Ps msolute(g) Msolvent
Ps = × (where m = given mass in solution, M = molar mass.)
Msolute msolvent(g)

Pº  Ps Msolvent
Ps = ( molality ) ×
1000
If solution is of single solute and it gets associated or dissociated;
Pº  Ps i.n Pº  Ps Msolvent
Ps = ; that is Ps = i × (molality) ×
N 1000

Commit to memory :

Pº  PS Pº  Ps n
* RLVP = = xfoy s; ; Ps =
Pº N
(t gk¡Pº = 'kq
) foy k;d d kok"i nkc, Ps = foy ;u ij ok"i d kvkaf'kd nkc, n = foy s; d sfo;ksft r eksy , N = foyk;d
d seksy )
Pº  Ps m foy s; (g) Mfoy k;d
Ps =
Mfoy s;
× m foy k;d (g) (t gk¡m = foy ;u esafn;k x;k nzO;eku, M = eksy j nzO;eku)

Pº  Ps M
Ps y y rk) × foy k;d
= (eks
1000
;fn foy ;u ,d y foy s; d k gSrFkk ;g laxqf.kr ;k fo;ksft r gksrk gS(
Pº  Ps i.n Pº  Ps M
Ps = ; vFkkZ
r~ P y y rk) × foy k;d
= i × (eks
N s 1000

I-1. Twenty grams of a solute are added to 100 g of water at 25ºC. The vapour pressure of pure water is 23.76
mmHg; the vapour pressure of the solution is 22.41 Torr.
(a) Calculate the molar mass of the solute.
(b) What mass of this solute is required in 100 g of water to reduce the vapour pressure to one-half the
value for pure water ?
25ºC ij 100 g t y esa20 g foy s; fey k;kt krk gSA 'kq) t y d k ok"i nkc 23.76 mmHg gSrFkkfoy ;u d kok"i nkc
22.41 Vks
j gSA
(a) foy s
; d seksy j nzO;eku d h x.kuk d jksA
(b) 'kq
) t y d sfy ;sok"i nkc d seku d ksvk/kk d jusd sfy ;s100 g t y esafoy s; d k fd ruk nzO;eku vko';d gS\
Ans. (a) 60 g/mol , (b) 333.3 g

Corporate Office: CG Tower, A-46 & 52, IPIA, Near City Mall, Jhalawar Road, Kota (Raj.)-324005
Website : www.resonance.ac.in | E-mail : contact@resonance.ac.in
Toll Free : 1800 200 2244 | 1800 258 5555 | CIN: U80302RJ2007PLC024029
ADV-SCP -9
Solution & Colligative Properties
I-2. The degree of dissociation of Ca(NO3)2 in a dilute aqueous solution containing 7 g salt per 100 g of water
at 100ºC is 70%. If the vapour pressure of water at 100ºC is 760 mm of Hg, calculate the vapour pressure
of the solution.
100ºC ij 7 g y o.k çfr 100 g
t y ;qDr ruqt y h; foy ;u esaCa(NO3)2 d sfo;kst u d h ek=kk 70% gSaA ;fn 100ºC
ij t y d k ok"i nkc 760 mm Hg gSA rc foy ;u d sok"i nkc d h x.kuk d hft ,A
Ans. 746.2 mm of Hg

I-3. Dry air was passed through bulbs containing a solution of 40 grams of non-volatile electrolytic solute in 360
grams of water, then through bulbs containing pure water at the same temperature and finally through a
tube in which pumice moistened with strong H2SO4 was kept. The water bulbs lost 0.0870 grams and the
sulphuric acid tube gained 2.175 grams. Calculate the molecular weight of solute.
,d cYc]ft lesa40 g vok"i'khy oS| qrvi?kV~; foy s; d ks360 g t y esa?kksy k t krk gS]esals'kq"d ok;qizokfgr d h t krh
gS]fQ j ;gh ok;qleku rki ij nwl jscYc ft lesa]'kq) t y Hkjk gSs]esalsizokfgr d jk;h t krh gSrFkk vUr esa,d V~;wc
esalsft lesaizcy H2SO4 d slkFk ,d I;wfel ¼,d iRFkj t kst y d ksvo'kksf"kr d jrk gS½ d ksj[kk x;k gS] lsizokfgr
d jk;h t krh gSA t y d scYc esa0.0870 xzke d k á l gksrk gSrFkk lY¶;wfjd vEy V~;wc esa2.175 g d h of̀) gksrh gSA
foy s; d k v.kqHkkj ifjd fy r d hft ,A
Ans. M = 48

Section (J) : Elevation of Boiling Point & Depression of Freezing Point


[k.M (J) : DoFkukad esamUu;u o fgekad esav oueu

Commit to memory :
* Tb = i × Kb × m (where Tb = elevation in boiling point of solution, Kb = ebullioscopic constant,
m = molality of single solute, Tb = boiling point of solvent (in K).
2 2
RTb M RTb
Kb = = (M = Molar mass of solvent)
1000  H vap 1000  L vap
Tf = i × Kf × m (where Tf = depression in freezing point of solution, Kf = cryoscopic constant,
Tf = freezing point of solvent (in K).
2 2
RTf M RTf
Kf = =
1000  Hfusion 1000  L fusion

Commit to memory :
* Tb = i × Kb × m (t gk¡Tb = foy ;u d sDoFkukad esamUu;u, Kb = DoFkukad ferh; fLFkjkad , m = ,d y foy s; d h
eksy y rk, Tb = foy k;d d k DoFkukad (K esa).
RTb2M RTb2
Kb = = (M = foy k;d d k eksy j nzO;eku)
1000  Hok"i 1000  L ok"i
Tf = i × Kf × m (t gk¡Tf = foy ;u d sfgekad esavoueu, Kf = fgekad ferh; fLFkjkad , Tf = foy k;d d k fgekad (K esa))
RTf 2 M RTf 2
Kf = = 1000  L
1000  Hlaxy u laxy u

J-1. (a) A solution containing 0.5 g of naphthalene in 50 g CCl4 yield a boiling point elevation of 0.4 K, while a
solution of 0.6 g of an unknown solute in the same mass of the solvent gives a boiling point elevation of
0.65 K. Find the molar mass of the unknown solute.
(b) The boiling point of a solution of 0.1 g of a substance in 16 g of ether was found to be 0.100ºC higher that
of pure ether. What is the molecular mass of the substance. Kb(ether) = 2.16 K kg mol–1.
(a) 50 g CCl4 esa0.5 g us¶Fksfy u mifLFkr gSft llsDoFkukad esamUu;u 0.4 K gS] t cfd ,d foy ;u ft lesablh

Corporate Office: CG Tower, A-46 & 52, IPIA, Near City Mall, Jhalawar Road, Kota (Raj.)-324005
Website : www.resonance.ac.in | E-mail : contact@resonance.ac.in
Toll Free : 1800 200 2244 | 1800 258 5555 | CIN: U80302RJ2007PLC024029
ADV-SCP - 10
Solution & Colligative Properties
foy k;d d sleku nzO;eku esa0.6 g vKkr foy s; mifLFkr gS] ft llsDoFkukad mUu;u 0.65 K izkIr gksrk gSA vKkr
foy s; d k eksy j nzO;eku Kkr d hft ,A
(b) ,d foy ;u] ft les a0.1 g foy s; 16 g bZFkj esafey kd j cuk;k t krk gS] d k DoFkukad ] 'kq) bZFkj d sDoFkukad ls
0.100ºC vf/kd ik;k x;kA inkFkZd k vkf.od nz O;eku D;k gS\ Kb(bZFkj) = 2.16 K kg mol–1.
Ans. (a) M = 94.52, (b) m = 135

J-2. The amount of benzene that will separate out (in grams) if a solution containing 7.32 g of triphenylmethane
in 1000 g of benzene is cooled to a temperature which is 0.2°C below the freezing point of benzene?
(Kf = 5.12 K-Kg/mol)
1000 g cs
Ut hu esa7.32 g VªkbQ sfuy esFksu ;qDr foy ;u d kscsUt hu d st eko fcUnqls0.2°C d e fd ;kt krk gS]rksiF̀kd
gqbZcsUt hu d k nzO;eku xzke esaKkr d hft ,A (Kf = 5.12 K-Kg/mol)
Ans. 232

J-3. The boiling point of a solution of 5 g of sulphur in 100 g of carbon disulphide is 0.474ºC above that of pure
solvent. Determine the molecular formula of sulphur in this solvent. The boiling point of pure carbon disulphide
is 47ºC and its heat of vaporisation is 84 calories per gram.
d kcZuMkblYQ kbM d s100 g esalYQ j d s5 g fey kd j cuk;sx;sfoy ;u d k DoFkukad 'kq) foy k;d ls0.474ºC vfèkd
gSA bl foy k;d esalYQ j d svkf.od lw=k d h x.kuk d hft ,A 'kq) d kcZuMkblYQ kbM d k DoFkukad 47ºC gSrFkk bld s
ok"iu d h Å "ek 84 d Sy kjh@xzke gSA
Ans. S8

J-4. Calculate the freezing point of a solution of a non-volatile solute in a unknown solvent of molar mass 30 g/
mole having mole fraction of solvent equal to 0.8. Given that latent heat of fusion of solid solvent = 2.7 kcal
mol–1, freezing point of solvent = 27°C and R = 2 cal mol–1 k–1.
30 g/mole eks
yj nzO;eku oky s,d vKkr foy k;d esavok"i'khy foy s; d s,d foy ;u d k fgekad Kkr d hft ,A ;fn
foy k;d d keksy izHkkt 0.8 gSA fn;kx;kgS]Bksl foy k;d d sxy u d hxqIr Å "ek= 2.7 kcal mol–1, foy k;d d kfgekad
= 27°C rFkk R = 2cal mol–1 k–1 gS A
Ans. 10.33°C

J-5. A 0.01 molal solution of ammonia freezes at – 0.02ºC. Calculate the van’t Hoff factor, i and the percentage
dissociation of ammonia in water. (K f (H2O ) ) = 1.86 deg molal–1.
veksfu;k d k 0.01 eksy y foy ;u – 0.02ºC ij t erk gSA okUVgkWQ xq.kkad (i) d h x.kuk d jksrFkk t y esaveksfu;k d k
çfr'kr fo;kst u Kkr d jksA(K f (H2O ) ) = 1.86 deg molal–1.
Ans. 1.075, 7.5.

Section (K) : Osmotic Pressure


[k.M (K) : ijklj.k nkc

Commit to memory :
n
*  = CRT = RT
T (n = Total moles of solute particles in solution, V = Total volume of solution in L)
V

Commit to memory :
n
*  = CRT = RT
T (n = foy ;u esafoy s; d .kksad sd qy eksy , V = foy ;u d k d qy vk;ru y hVj esa)
V

Corporate Office: CG Tower, A-46 & 52, IPIA, Near City Mall, Jhalawar Road, Kota (Raj.)-324005
Website : www.resonance.ac.in | E-mail : contact@resonance.ac.in
Toll Free : 1800 200 2244 | 1800 258 5555 | CIN: U80302RJ2007PLC024029
ADV-SCP - 11
Solution & Colligative Properties
K-1. (a) Predict the osmotic pressure order for the following(assume salts are 100% dissociated).
I 0.1 M urea II 0.1 M NaCl
III 0.1 M Na2SO4 IV 0.1 M Na3PO4
(b) If equal volumes of all these solutions are mixed then calculate the osmotic pressure of the net
resultant solution obtained at 300K.

(a) fuEufy f[kr d sijklj.k nkc d k Ø e crkb;s(ekuk y o.k 100% fo;ksft r gksrk gS)
I 0.1 M ;wfj;k II 0.1 M NaCl
III 0.1 M Na2SO4 IV 0.1 M Na3PO4
(b) ;fn bu lHkh foy ;uksad sleku vk;ru fefJr fd ;st k,sarks300K ij izkIr ifj.kkeh foy ;u d k ijklj.k nkc
ifjd fy r d hft ,A
Ans. (a) Urea < NaCl < Na2SO4 < Na3PO4 (b) 6.15 atm
K-2. A solution containing 3.00 g of calcium nitrate in 100 c.c. of solution had an osmotic pressure of 11.2
atmosphere at 12° C. Calculate the degree of ionisation of calcium nitrate at this dilution and temperature.
12ºC rki ij 100 c.c. foy ;u es
a3.00 g d SfY'k;e ukbVªsV ;qDr foy ;u d kijklj.knkc 11.2 atm gSA bl ruqrko rki
ij d SfY'k;e ukbVªsV d svk;uu d h ek=kk ifjd fy r d hft ,A
Ans.  = 0.81
K-3. 17.4% (w/v) K2SO4 solution at 27ºC is isotonic with 5.85% (w/v) NaCl solution at 27ºC. If NaCl is 100%
ionised, what is % ionisation of K2SO4 in aq. solution ?
27ºC ij 17.4% (Hkkj@vk;ru) K2SO4 foy ;u]5.85% (Hkkj@vk;ru) NaCl foy ;u d slkFk leijkljh gS A ;fn NaCl
d ks100% vk;uhd r̀ ekuk t k;as] rkst y h; foy ;u esaK2SO4 d k % vk;uu (ionisation) D;k gS\
Ans. 50% K2SO4
K-4. At 2ºC the osmotic pressure of a urea solution is found to be 500 mm of Hg. The solution is diluted and the
temperature is raised to 27ºC, when the osmotic pressure is found to be 109.09 mm of Hg. Determine the
extent of dilution.
2ºC ij ;w fj;k foy ;u d k ijklj.k nkc 500 mm Hg ik;k t krk gSA t c foy ;u d ksruqfd ;k t krk gSrFkk rki d ks
27ºC rd c<+k;k t krk gS] ml le; ijklj.k nkc 109.09 mm Hg ik;k t krk gSA foy ;u d sruqrk d h lhek Kkr
d hft ,A
Ans. Volume must have been made 5 times
mÙkj- vk;ru d ks5 xquk d j nsuk pkfg;sA

PART - II : ONLY ONE OPTION CORRECT TYPE


Hkkx - II : d soy ,d l gh fod Yi çd kj (ONLY ONE OPTION CORRECT TYPE)

Section (A) : Concentration terms (Revision of mole)


[k.M (A) : lkUnzrk in (eksy d k la'kks/ku)
A-1. Persons are medically considered to have lead poisoning if they have a concentration greater than 10
micrograms of lead per decilitre of blood. Concentration in parts per billion is :
t c O;fDr d sjDr esaizfr Msl h y hVj y sM d s10 ekbØ ksxzke lsvf/kd d h lkUnzrk gksrksmlsesfMd y t k¡p esat gjhy k
y sM fy , gq, ekuk t krk gSA Hkkx izfr fcfy ;u esalkUnzrk gS%
(A) 1000 (B*) 100 (C) 10 (D) 1

Section (B) : General Introduction & types of solution


[k.M (B) : lkekU; çLrkouk o foy ;u d sçd kj
B-1. Which statement best explains the meaning of the phrase “like dissolves like “ ?
(A) A Solute will easily dissolve a solute of similar mass
(B*) A solvent and solute with similar intermolecular forces will readily form a solution
(C) The only true solutions are formed when water dissolves a non-polar solute
(D) The only true solutions are formed when water dissolves a polar solute
Corporate Office: CG Tower, A-46 & 52, IPIA, Near City Mall, Jhalawar Road, Kota (Raj.)-324005
Website : www.resonance.ac.in | E-mail : contact@resonance.ac.in
Toll Free : 1800 200 2244 | 1800 258 5555 | CIN: U80302RJ2007PLC024029
ADV-SCP - 12
Solution & Colligative Properties
d gkor ^^leku&leku d ks?kksy rk gS** d sfy , d kSulk d Fku bld svFkZd k lgh o.kZu d jrk gS\
(A) ,d foy s
; vklkuh ls?kqy rk gSft lesafoy s; d k leku nzO;eku gksrk gSA
(B*) ,d foy k;d rFkk foy s ; d se/; leku vUrjkf.od cy gksusij ;g t Ynh lsfoy ;u cukrsgSA
(C) t c t y esav/kqzoh; foy s; d ks?kksy rsgS] rksd soy okLrfod foy ;u cursgSaA
(D) t c t y esa/kqzoh; foy s; d ks?kksy rsgS] rksd soy okLrfod foy ;u cursgSA
B-2. An ionic compound that attracts atmospheric water so strongly that a hydrate is formed is said to be :
(A) Dilute (B*) Hygroscopic (C) Immiscible (D) Miscible
,d vk;fud ;kSfxd t ksfd ok;qe.My h; t y lsizcy : i lsvkd f"kZr gksrkgS];g ,d gkbMªsV cukrkgS]ft lsd gkt krk
gS:
(A) ruq (B*) vknZ
zrkxzkgh (C) vfeJ.kh; (D) feJ.kh;

Section (C) : Vapour Pressure


[k.M (C) : ok"i nkc
C-1. A liquid is kept in a closed vessel. If a glass plate (negligible mass) with a small hole is kept on top of the
liquid surface, then the vapour pressure of the liquid in the vessel is :
(A) More than what would be if the glass plate were removed
(B*) Same as what would be if the glass plate were removed
(C) Less than what would be if the glass plate were removed
(D) Cannot be predicted
,d can ik=kesanzo d ksj[kkt krkgSA ;fn nzo lrg d sÅ ij d hvksj ,d NksVsfNnz;qDr Xy kl Iy sV d ks(nzO;eku ux.;)
j[kk t krk gS] rksik=k esanzo d k ok"i nkc %
(A) Xy kl Iy s
V d ksgVkusij t ksçkIr gksuk pkfg;smllsvf/kd gksxkA
(B*) Xy kl Iy s
V d ksgVkusij t ksçkIr gksuk pkfg;smld sleku gksxkA
(C) Xy kl Iy s
V d ksgVkusij t ksçkIr gksuk pkfg;smllsd e gksxkA
(D) Kkr ughafd ;k t k ld rk gS A
C-2. The vapour pressure of water depends upon :
(A) Surface area of container (B) Volume of container
(C*) Temperature (D) All
t y d k ok"i nkc fuHkZj d jrk gS%
(A) ik=k d si`
"Bh; {ks=kQ y ij (B) ik=k d svk;ru ij
(C*) rkieku ij (D) mijksDr lHkh ij
C-3. Among the following substances, the lowest vapour pressure is exerted by :
(A) Water (B*) Mercury (C) Acetone (D) Ethanol
fuEu inkFkksZesals] U;wure ok"i nkc fd ld k gS\
(A) t y (B*) ed Z
jh (C) ,lhVks
u (D) ,Fks
uksy
C-4. At higher altitudes, water boils at temperature < 100ºC because
(A) temperature of higher altitudes is low (B*) atmospheric pressure is low
(C) the proportion of heavy water increases (D) atmospheric pressure becomes more.
mPp LFkkuksaij] t y 100ºC lsd e rki ij mcy rk gS] D;ksafd
(A) mPp LFkkuksad k rkieku fuEu gksrk gSa (B*) ok;q
e.My h; nkc fuEu gksrk gS
(C) Hkkjh t y d k vuqikr c<+rk gSA (D) ok;q
e.My h; nkc vf/kd gkst krk gSA
Section (D) : Solutions of Solid and Gases in Liquids
[k.M (D) : nzo esaBksl rFkk xSl ksad k foy ;u
D-1. The solubility of gases in liquids :
(A) increases with increase in pressure and temperature
(B) decreases with increase in pressure and temperature
(C*) Increases with increase in pressure and decrease in temperature
(D) decreases with increase in pressure and increase in temperature
Corporate Office: CG Tower, A-46 & 52, IPIA, Near City Mall, Jhalawar Road, Kota (Raj.)-324005
Website : www.resonance.ac.in | E-mail : contact@resonance.ac.in
Toll Free : 1800 200 2244 | 1800 258 5555 | CIN: U80302RJ2007PLC024029
ADV-SCP - 13
Solution & Colligative Properties
nzoksesaxSl ksad h foy s;rk %
(A) nkc rFkk rkieku es aof̀) d slkFk c<+rh gSA
(B) nkc rFkk rkieku es aof̀) d slkFk ?kVrh gSA
(C*) nkc es aof̀) rFkk rkieku esad eh d slkFk c<+rh gSA
(D) nkc es aof̀) rFkk rkieku esaof̀) d slkFk ?kVrh gSA
D-2. Which of the following curves represents the Henry’s law ?
fuEu esalsd kSu lk oØ gsujh fu;e d ksiznf'kZr d jrk gSA

(A*) (B) (C) (D)

D-3. According to Henry’s law, the solubility of a gas in a given volume of liquid increases with increase in :
(A) Temperature (B*) Pressure (C) Both (A) and (B) (D) None of these
gsujh fu;e d svuql kj nzo d sfn, x;svk;ru esa,d xSl d h foy s;rk esafuEu esaof̀} d slkFk of̀} gksrh gS&
(A) rkieku (B*) nkc (C) (A) o (B) nks
uksa (D) buesalsd ksbZugha
D-4. Some of the following gases are soluble in water due to formation of their ions :
I : CO2 ; II : NH3 ; III : HCl ; IV : CH4 ; V : H2
Water insoluble gases can be :
fuEu esalsd qN xSl s] bud svk;u fuekZ.k d sd kj.k t y esafoy s;'khy gksrh gSa%
I : CO2 ; II : NH3 ; III : HCl ; IV : CH4 ; V : H2
t y esav?kqy u'khy xSl sgksld rh gS&
(A) I, IV , V (B) I, V (C) I, II, III (D*) IV, V
D-5. The solubility of N2(g) in water exposed to the atmosphere, when its partial pressure is 593 mm is
5.3 × 10–4 M. Its solubility at 760 mm and at the same temperature is :
okrkoj.k esa[kqy sj[kst y esaN2(g) d h foy ;srk 5.3 × 10–4 M gS] t c mld k vkaf'kd nkc 593 nm gSA leku rki ij
,oa760 mm ij bld h foy s;rk fuEu gS%
(A) 4.1 × 10–4 M (B*) 6.8 × 10–4 M (C) 1500 M (D) 2400 M

Section (E) : Immiscible Liquids


[k.M (E) : v feJ.kh; nzo
E-1. When a liquid that is immiscible with water was steam distilled at 95.2ºC at a total pressure of 748 torr, the
distillate contained 1.25 g of the liquid per gram of water. The vapour pressure of water is 648 torr at
95.2ºC, what is the molar mass of liquid?
(A) 7.975 g/mol (B) 166 g/mol (C*) 145.8 g/mol (D) None of these
t y d slkFk vfeJ.kh; nzo d k 95.2ºC o d qy nkc 748 Vksj ij Hkki vklou d jrsgSA vklfor (distillate) esat y d s
çfrxzke esa1.25 g nzo fLFkr gSA 95.2ºC ij t y d k ok"i nkc 648 Vksj gSA nzo d k eksy j nzO;eku Kkr d hft ,A
(A) 7.975 g/mol (B) 166 g/mol (C*) 145.8 g/mol (D) bues alsd ksbZugha
Section (F) : Completely miscible liquids : Raoult’s law
[k.M (F) : iw.kZr% feJ.kh; nzo % jkWÅ YV fu;e
F-1. For a binary ideal liquid solution, the total pressure of the solution is given as :
(A) Ptotal = PºA + (PºA – PºB) XB (B*) Ptotal = PºB + (PºA – PºB) XA
(C) Ptotal = PºB + (PºB – PºA) XA (D) Ptotal = PºB + (PºB – PºA) XB

Corporate Office: CG Tower, A-46 & 52, IPIA, Near City Mall, Jhalawar Road, Kota (Raj.)-324005
Website : www.resonance.ac.in | E-mail : contact@resonance.ac.in
Toll Free : 1800 200 2244 | 1800 258 5555 | CIN: U80302RJ2007PLC024029
ADV-SCP - 14
Solution & Colligative Properties
f}?kVd h; vkn'kZnzo foy ;u d sfy , foy ;u d k d qy nkc gS%
(A) Pd qy = PºA + (PºA – PºB) XB (B*) Pd qy = PºB + (PºA – PºB) XA
(C) Pd qy = PºB + (PºB – PºA) XA (D) Pd qy = PºB + (PºB – PºA) XB

F-2. An ideal solution contains two volatile liquids A (pº = 100 torr) and B (pº = 200 torr). If mixture contain 1 mole
of A and 4 mole of B then total vapour pressure of the distillate is:
,d vkn'kZfoy ;u nksok"i'khy nzo A (pº = 100 torr) o B (pº = 200 torr) j[krk gSA ;fn feJ.k esaA d k1 eksy o B d s
4 eks
y fLFkr gSrc vklfor (distillate) d k d qy ok"i nkc gS%
(A) 150 (B) 180 (C*) 188.88 (D) 198.88

F-3. At 323 K, the vapour pressure in millimeters of mercury of a methanol-ethanol solution is represented by
pA
the equation p = 120 XA + 140, where XA is the mole fraction of methanol. Then the value of xlim X is
A 1 A

323 K ij, ,d esFksukWy &,FksukWy foy ;u d h ed Zjh d k fey hehVj esaok"inkc lehd j.kp = 120 XA + 140 }kjk iznf'kZr
A p
fd ;k t krk gS, t gk¡XA esFksukWy d k eksy çHkkt gSA rc xlim d k eku gS&
A 1 X A

(A) 250 mm (B) 140 mm (C*) 260 mm (D) 20 mm


F-4. Given at 350 K pA° = 300 torr and pB° = 800 torr, the composition of the mixture having a normal boiling
point of 350 K is :
350 K ij fn;k x;k gSpA° = 300 Vks
j vkSj pB° = 800 Vksj gSA 350 K d slkekU; DoFkukad d sfeJ.k d k la?kBu gSA
(A*) XA = 0.08 (B) XA = 0.06 (C) XA = 0.04 (D) XA = 0.02
F-5. Two liquids A and B have PºA and PºB in the ratio of 1 : 3 and the ratio of number of moles of A and B in liquid
phase are 1 : 3 then mole fraction of ‘A’ in vapour phase in equilibrium with the solution is equal to :
nksnzo A o B d sPºA o PºB d kvuqikr 1 : 3 gSrFkknzo çkoLFkkesaA o B d seksy ksad hla[;kd kvuqikr 1 : 3 gSrc foy ;u
d slkFk lkE; esaok"i voLFkk esa‘A’ d h eksy fHkUu gS%
(A*) 0.1 (B) 0.2 (C) 0.5 (D) 1.0

F-6. The vapour pressure of two pure liquids A and B, that form an ideal solution are 100 and 900 torr respectively
at temperature T. This liquid solution of A and B is composed of 1 mole of A and 1 mole of B. What will be
the pressure, when 1 mole of mixture has been vapourized ?
(A) 800 torr (B) 500 torr (C*) 300 torr (D) None of these
rkieku T ij nks'kq) nzo A o B t ksvkn'kZfoy ;u cukrsgS] d sok"i nkc Ø e'k%100 o 900 Vksj gSA A o B d k ;g nzo
foy ;u A d k1 eksy o B d k1 eksy j[krk gSA t c 1 eksy feJ.k ok"ihd r̀ gksrk gS] rc nkc gS&
(A) 800 torr (B) 500 torr (C*) 300 torr (D) buesalsd ksbZugha
Section (G) : Non-ideal Solutions
[k.M (G) : v ukn'kZfoy ;u
G-1. The vapour pressure of the solution of two liquids A(pº = 80 mm) and B(pº = 120 mm) is found to be 100
mm when xA = 0.4. The result shows that
(A) solution exhibits ideal behaviour (B) solution shows positive deviations
(C*) solution shows negative deviations
(D) solution will show positive deviations for lower concentration and negative deviations for higher
concentrations.
nksnzoksaA(pº = 80 mm) o B(pº = 120 mm) d sfoy ;u d k ok"i nkc 100 mm ik;k x;k] t c xA = 0.4 gSA ifj.kke
n'kkZrk gSfd &
(A) foy ;u vkn'kZO;ogkj iz
nf'kZr d jrk gSA (B) foy ;u /kukRed fopy u n'kkZrk gSA
(C*) foy ;u _ .kkRed fopy u n'kkZrk gSA
(D) foy ;u fuEu lkUnz
rk d sfy , /kukRed rFkk mPp lkUnzrk d sfy , _ .kkRed fopy u n'kkZrk gSaA

Corporate Office: CG Tower, A-46 & 52, IPIA, Near City Mall, Jhalawar Road, Kota (Raj.)-324005
Website : www.resonance.ac.in | E-mail : contact@resonance.ac.in
Toll Free : 1800 200 2244 | 1800 258 5555 | CIN: U80302RJ2007PLC024029
ADV-SCP - 15
Solution & Colligative Properties
G-2. Consider a binary mixture of volatile liquids. If at XA = 0.4 the vapour pressure of solution is 580 torr then the
mixture could be (pAº = 300 torr, pB° = 800 torr) :
ok"i'khy nzoksd sf}?kVd h; feJ.k ij fopkj d hft ,A ;fn XA = 0.4 ij foy ;u d k ok"i nkc 580 Vksj gks] rc feJ.k
gksld rk gS(pAº = 300 torr, pB° = 800 torr) %
(A*) CHCl3 – CH3COCH3 (B) C6H5Cl – C6H5Br
(C) C6H6 – C6H5CH3 (D) nC6H14 – nC7H16

G-3. A maxima or minima obtained in the temperature composition curve of a mixture of two liquids indicates
(A*) an azeotropic mixture (B) an eutectic formation
(C) that the liquids are immiscible with one another
(D) that the liquids are partially miscible at the maximum or minimum
nksnzoksad sfeJ.k d srkieku laxBu oØ esa,d mfPp"B vFkok fufEu"B bafxr d jrk gS&
(A*) ,d fLFkj DokFkh feJ.k gSaA (B) ,d xy u Ø kafrd feJ.k d k fuekZ.k gSaA
(C) nzo ,d nwl jsd slkFk vfeJ.kh; gSA (D) mfPp"B vFkok fufEu"V ij nzo vkaf'kd : i lsfeJ.kh; gSaA
G-4. A solution of sulphuric acid in water exhibits :
(A*) Negative deviations from Raoult’s law (B) Positive deviations from Raoult’s law
(C) Ideal properties (D) The applicability of Henry’s law
t y esalY¶;wfjd vEy d k foy ;u iznf'kZr d jrk gSA
(A*) jkW
Å YV fu;e ls_ .kkRed fopy u (B) jkW
Å YV fu;e ls/kukRed fopy u
(C) vkn'kZxq.k/keZ (D) gsujh fu;e d svuqç;ksx
G-5. When KCl dissolves in water (assume endothermic dissolution), then :
t c KCl d kst y esa?kksy k t krk gS¼ekuk fd Å "ek'kks"kh foy k;d u gS½] rc %
(A) H = + ve, S = + ve, G = + ve (B) H = + ve, S = – ve, G = – ve
(C*) H = + ve, S = + ve, G = – ve (D) H = – ve, S = – ve, G = + ve

G-6. The dissolving process is exothermic when :


(A*) The energy released in solvation exceeds the energy used in breaking up solute-solute and solvent-
solvent interactions.
(B) The energy used in solvation exceeds the energy released in breaking up solute-solute and solvent-
solvent interactions.
(C) The energy released in solvation is about the same as the energy used in breaking up solute-solute and
solvent-solvent interactions.
(D) The energy used in solvation is about the same as the energy used in breaking up solute-solute and
solvent-solvent interactions.
foy k;d u fof/k Å "ek{ksih gksrh gSt c %
(A*) foy k;d ;kst u (solvation) esamRlft Zr Å t kZfoy s;&foy s; rFkk foy k;d &foy k;d vU;ksU; fØ ;k d ksrksM +usesa
iz;qDr Å t kZlsvf/kd gksrh gSA
(B) foy k;d ;kst u esaiz;qDr Å t kZfoy s;&foy s; rFkk foy k;d &foy k;d vU;ksU; fØ ;k d ksrksM +usesamRlft Zr gq;h Å t kZ
lsvf/kd gksrh gSA
(C) foy k;d ;kst u esamRlft Zr Å t kZfoy s;&foy s; rFkk foy k;d &foy k;d vU;ksU; fØ ;k d ksrksM +usesaiz;qDr Å t kZd s
y xHkx leku gksrh gSA
(D) foy k;d ;kst u esaiz;qDr Å t kZfoy s;&foy s; rFkk foy k;d &foy k;d vU;ksU; fØ ;k d ksrksM +usesaiz;qDr Å t kZd s
y xHkx cjkcj gksrh gSA
G-7. Which of the following is less than zero for ideal solutions ?
(A) Hmix (B) Vmix (C*) Gmix (D) Smix
fuEu esalsd kSu vkn'kZfoy ;uksad sfy , 'kwU; lsd e gS\
(A) HfeJ.k (B) VfeJ.k (C*) GfeJ.k (D) SfeJ.k

Corporate Office: CG Tower, A-46 & 52, IPIA, Near City Mall, Jhalawar Road, Kota (Raj.)-324005
Website : www.resonance.ac.in | E-mail : contact@resonance.ac.in
Toll Free : 1800 200 2244 | 1800 258 5555 | CIN: U80302RJ2007PLC024029
ADV-SCP - 16
Solution & Colligative Properties
Section (H) : Degree of Ionisation/Dissociation for Weak Electrolytes
[k.M (H) : nqcZy oS| qr v i?kV~;ksad sv k;uu@fo;kst u d h ek=kk
H-1. One mole of a solute A is dissolved in a given volume of solvent. The association of the solute take place
as follows: nA An
If  is the degree of association of A, the van’t Hoff factor i is expressed as:
foy s; A d s1 eksy d ksfoy k;d d sfn;sx;svk;ru esa?kksy rsgSA foy s; d k laxq.ku nA And svuq
l kj gksrk gSA
;fn A d slaxq.ku d h ek=kk gSrc okUV gkWQ xq.kkad (i) d ksçnf'kZr d jrsgSA

1–  
 n
(A) i  1 –  (B) i  1  (C*) i = 1
(D) i = 1
n

H-2. The degree of dissociation of an electrolyte is  and its van’t Hoff factor is i. The number of ions obtained
by complete dissociation of 1 molecule of the electrolyte is :
,d oS| qr vi?kV~; d sfy , fo;kst u d h ek=kk  gSo bld k okUVgkWQ xq.kkad i gSA oS| qr vi?kV; d s,d v.kqd siw.kZ
fo;kst u }kjk izkIr vk;uksad h la[;k fuEu gS%
i  –1 i–1 i  1 
(A*) (B) i –  – 1 (C) (D)
  1– 

H-3. If Mnormal is the normal molecular mass and  is the degree of ionization of K3[Fe(CN)6], then the abnormal
molecular mass of the complex in the solution will be :
(A) Mnormal (1 + 2)–1 (B*) Mnormal (1 + 3)–1 (C) Mnormal (1 + )–1 (D) equal to Mnormal
;fn MlkekU; lkekU; vkf.od nzO;eku gSrFkk]K3[Fe(CN)6] d svk;uu d h ek=kk gSrc foy ;u esalad qy d k vlkekU;
vkf.od nzO;eku D;k gksxk \
(A) MlkekU; (1 + 2)–1 (B*) MlkekU; (1 + 3)–1 (C) MlkekU; (1 + )–1 (D) MlkekU; d scjkcj
H-4. A complex containing K+, Pt(IV) and Cl¯ is 100% ionised giving i = 3. Thus, complex is :
,d lad qy K+, Pt(IV) rFkkCl¯ ;qDr 100% vk;fur gksd j i = 3 nsrk gSrkslad qy gksxk %
(A) K2[PtCl4] (B*) K2[PtCl6] (C) K3[PtCl5] (D) K[PtCl3]

Section (I) : Relative lowering of vapour pressure


[k.M (I) : ok"i nkc d k v kisf{kd v oueu
I-1. If P0 and P are the vapour pressures of a solvent and its solution respectively and N1 and N2 are the mole
fractions of the solvent and non-volatile solute respectively, then correct relation is :
;fn ,d foy k;d ,oabld sfoy ;u d sok"i&nkc Ø e'k%P0 ,oaP gSarFkk foy k;d ,oavok"i'khy foy s; d h eksy fHkUu
Ø e'k% N1 o N2 gSa] rkslgh laca/k fuEu gS%
(A) P = P0N2 (B*) P = P0N1 (C) P0 = PN1 (D) P = P0(N1/N2)

I-2. Relative decrease in vapour pressure of an aqueous NaCl is 0.167. Number of moles of NaCl present in
180g of H2O is :
t y h; NaCl d sok"i nkc esavkisf{kd voueu 0.167 gSrks180g H2O esaNaCl d smifLFkr eksy ksad h la[;k gksxh :
(A) 2 mol (B*) 1 mol (C) 3 mol (D) 4 mol

I-3. The vapour pressure of pure benzene, C6H6 at 50°C is 268 Torr. How many moles of non-volatile solute per
mol of benzene is required to prepare a solution of benzene having a vapour pressure of 167 Torr at 50°C?
50°C ij 'kq
) csUt hu C6H6 d k ok"i nkc 268 Torr gSA csUt hu d k foy ;u cukusd sfy , csUt hu d sizfr eksy d sfy ,
fd rusvok"i'khy foy s; d seksy ksad h vko';d rk gksrh gSA 50°C ij csUt hu d sfoy ;u d k ok"inkc 167 torr gS?
(A) 0.377 (B*) 0.605 (C) 0.623 (D) 0.395
I-4. If relative decrease in vapour pressure is 0.4 for a solution containing 1 mol NaCl in 3 mol H2O, NaCl is ....
% ionised.
3 eksy H2O esa1 eksy NaCl foy ;u d sfy , ;fn ok"i nkc esavkisf{kd d eh 0.4 gSrksNaCl .... % vk;fur gksrk gSA
(A) 60% (B) 50% (C*) 100% (D) 40%
Corporate Office: CG Tower, A-46 & 52, IPIA, Near City Mall, Jhalawar Road, Kota (Raj.)-324005
Website : www.resonance.ac.in | E-mail : contact@resonance.ac.in
Toll Free : 1800 200 2244 | 1800 258 5555 | CIN: U80302RJ2007PLC024029
ADV-SCP - 17
Solution & Colligative Properties
I-5. The vapour pressure of a solution of a non-volatile solute B in a solvent A is 95% of the vapour pressure of
the solvent at the same temperature. If the molecular weight of the solvent is 0.3 times the molecular
weight of the solute, what is the ratio of weight of solvent to solute.
(A) 0.15 (B*) 5.7 (C) 0.2 (D) none of these
,d foy k;d A esa,d vok"i'khy foy s; B d kfoy ;u d sok"i nkc d keku mlh rki ij foy k;d d sok"i nkc d k95%
gSA ;fn foy k;d d k v.kqHkkj] foy s; d sv.kqHkkj d k 0.3 xquk gS] rksfoy k;d o foy s; d sHkkj d k vuqikr Kkr d jksA
(A) 0.15 (B*) 5.7 (C) 0.2 (D) bues alsd ksbZugha
I-6. Relative decrease in vapour pressure of an aqueous solution containing 2 moles [Cu(NH3)3Cl]Cl in 3 moles
H2O is 0.50. On reaction with AgNO3, this solution will form(assuming no change in degree of ionisation of
substance on adding AgNO3)
(A*) 1 mol AgCl (B) 0.25 mol AgCl (C) 0.5 mol AgCl (D) 0.40 mol AgCl
,d t y h; foy ;u esa3 eksy H2O esa2 eksy [Cu(NH3)3 Cl] Cl fo|eku gSA bl foy ;u d sok"inkc esavkisf{kd d eh0.50
gSA AgNO3 d slkFk foy ;u d h vfHkfØ ;k d jusij ;g fuEu esalsD;k cuk;sxk (AgNO3 fey kusij inkFkZd svk;uu
d h ek=kk esaifjorZu ughaekusaA) %
(A*) 1 eksy AgCl (B) 0.25 eks
y AgCl (C) 0.5 eks
y AgCl (D) 0.40 eks
y AgCl
Section (J) : Elevation of Boiling Point & Depression of Freezing Point
[k.M (J) : DoFkukad esamUu;u o fgekad esav oueu
J-1. At a constant temperature, S will be maximum for which of the following processes :
(A*) Vaporisation of a pure solvent
(B) Vaporisation of solvent from a solution containing nonvolatile and nonelectrolytic solute in it
(C) Vaporisation of solvent from a solution containing nonvolatile but electrolytic solute in it
(D) Entropy change will be same in all the above cases
fu;r rkieku ij] fuEu esalsfd l izØ e d sfy , S vf/kd re gksxk \
(A*) 'kq
) foy k;d d k ok"ihd j.k
(B) vok"i'khy o fo|q r vuvi?kV~; foy s; ;qDr ,d foy ;u lsfoy k;d d k ok"ihd j.k
(C) vok"i'khy ] ijUrqfo|q
r vi?kV~; foy s; ;qDr foy ;u lsfoy k;d d k ok"ihd j.k
(D) mijks
Dr lHkh fLFkfr;ksaesa,UVªkWih ifjorZu leku jgsxkA
J-2. A solute’S’ undergoes a reversible trimerization when dissolved in a certain solvent. The boiling point
elevation of its 0.1 molal solution was found to be identical to the boiling point elevation in case of a 0.08
molal solution of a solute which neither undergoes association nor dissociation. To what percent had the
solute ‘S’ undergone trimerization?
,d foy s; ’S’ esamRØ e.kh; f=ky d hd j.kgkst krkgSA t c mlsfd lhfoy k;d esa?kksy kt krkgSA bld s0.1 eksy y foy ;u
d sDoFkukad mUu;u d k eku],d foy s; ¼ft ld k laxq.ku ;k fo;kst u ughagksrk gSa½ d s0.08 eksy y foy ;u d sDoFkukad
mUu;u d seku d sleku ik;k x;kA foy s; ‘S’ d sf=ky d hd j.k d h izfr'krrk Kkr d hft ,A
(A*) 30% (B) 40% (C) 50% (D) 60%

J-3. A complex of iron and cyanide ions is 100% ionised at 1m (molal). If its elevation in b.p. is 2.08. Then the
complex is (Kb = 0.52° mol–1 kg) :
vk;ju rFkk lk;ukbM vk;uksad k ,d lad qy 1m (eksy y ) ij 100% vk;fur gksrk gSA ;fn bld sDoFkukad esamUu;u
2.08 gS
A rc lad qy gksxk (Kb = 0.52° mol–1 kg) %
(A*) K3[Fe(CN)6] (B) Fe(CN)2 (C) K4[Fe(CN)6] (D) Fe(CN)4

J-4. PtCl4.6H2O can exist as a hydrated complex 1 molal aq. solution has depression in freezing point of 3.72°.
Assume 100% ionisation and Kf(H2O) = 1.86° mol–1 kg, then complex is -
PtCl4.6H2O ,d t y ;ksft r lad qy d s: i esagksrk gSaA 1 eksy y t y h; foy ;u d sfgekad esavoueu 3.72° gSA ;g
ekud j fd 100% vk;uu gS,oaKf(H2O) = 1.86° eksy –1 kg, gSa] rkslad qy fuEu gSa%
(A) [Pt(H2O)6]Cl4 (B) [Pt(H2O)4Cl2]Cl2 . 2H2O
(C*) [Pt(H2O)3Cl3]Cl . 3H2O (D) [Pt(H2O)2Cl4] . 4H2O

Corporate Office: CG Tower, A-46 & 52, IPIA, Near City Mall, Jhalawar Road, Kota (Raj.)-324005
Website : www.resonance.ac.in | E-mail : contact@resonance.ac.in
Toll Free : 1800 200 2244 | 1800 258 5555 | CIN: U80302RJ2007PLC024029
ADV-SCP - 18
Solution & Colligative Properties
J-5. How many moles of sucrose should be dissolved in 500 g of water so as to get a solution which has a
difference of 104°C between boiling point and freezing point. (Kf = 1.86 K Kg mol–1, Kb = 0.52 K Kg mol–1)
500 g t y esalwØ kst d sfd ruseksy ?kksy st kuspkfg,]rkfd ,sl kfoy ;u izkIr fd ;kt kld sa]ft ld sDoFkukad o fgekad
esa104ºC d k vUrj gks? (Kf = 1.86 K Kg mol–1, Kb = 0.52 K Kg mol–1)
(A) 1.68 (B) 3.36 (C) 8.40 (D*) 0.840

J-6. Which of the following has been arranged in order of decreasing freezing point?
(A*) 0.05 M KNO3 > 0.04 M CaCl2 > 0.140 M sugar > 0.075 M CuSO4
(B) 0.04 M BaCl2 > 0.140 M sucrose > 0.075 M CuSO4 > 0.05 M KNO3
(C) 0.075 M CuSO4 > 0.140 M sucrose > 0.04 M BaCl2 > 0.05 M KNO3
(D) 0.075 M CuSO4 > 0.05 M NaNO3 > 0.140 M sucrose > 0.04 M BaCl2
fuEu esalsd kSulk fod Yi fgekad d s?kVrsgq, Ø e esaO;ofLFkr gS?
(A*) 0.05 M KNO3 > 0.04 M CaCl2 > 0.140 M 'kd Z
jk> 0.075 M CuSO4
(B) 0.04 M BaCl2 > 0.140 M lw
Ø kst > 0.075 M CuSO4 > 0.05 M KNO3
(C) 0.075 M CuSO4 > 0.140 M lw
Ø kst > 0.04 M BaCl2 > 0.05 M KNO3
(D) 0.075 M CuSO4 > 0.05 M NaNO3 > 0.140 M lw
Ø kst > 0.04 M BaCl2
J-7. A solution of x moles of sucrose in 100 grams of water freezes at 0.2ºC. As ice separates the freezing
point goes down to – 0.25ºC. How many grams of ice would have separated?
lwØ kst d sx eksy d k foy ;u 100 xzke t y esa0.2ºC ij t erk gSA t c cQ Zd ksvy x d j y srsgSaA fgekad – 0.25ºC
rd d e gkst krk gSA cQ Zd sfd rusxzke d ksvy x fd ;k t krk gS\
(A) 18 grams (B*) 20 grams (C) 25 grams (D) 23 grams
J-8. Sea water is found to contain 5.85 % NaCl and 9.50% MgCl2 by weight of solution. Calculate its normal
boiling point assuming 80% ionisation for NaCl and 50% ionisation of MgCl2 (Kb(H2O) = 0.51 kgmol–1 K).
leqnzt y esafoy ;u d sHkkj d k 5.85% NaCl rFkk 9.50% MgCl2 ik;k x;kA bld k lkekU; DoFkukad Kkr d hft ,A
ekuk fd NaCl d k 80% vk;uu o MgCl2 d k 50% vk;uu gksrk gSA [Kb(H2O) = 0.51 kg mol–1 K].
(A) Tb = 101.9°C (B*) Tb = 102.3°C (C) Tb = 108.5°C (D) Tb = 110.3°C
J-9. The freezing point of aqueous solution that contains 3% urea, 7.45% KCl and 9% of glucose is (given Kf of
water = 1.86 and asume molarity = molality).
t y h; foy ;u d k fgekad D;k gksxk t ks;wfj;k d k 3%] KCl d k 7.45% vkSj Xy wd kst d k 9% j[krk gSA (fn;k x;k gS%
t y d k Kf = 1.86 rFkk ekukfd eksy jrk = eksy y rk½
(A) 290 K (B) 285.5 K (C*) 267.42 K (D) 250 K

Section (K) : Osmotic Pressure


[k.M (K) : ijklj.k nkc
K-1. Select correct statement :
(A) Osmosis produced by semipermeable membrane.
(B) Desalination of sea-water is done by reverse osmosis
(C*) Both are correct statements (D) None is correct statement
lgh d Fku d k p;u d hft , \
(A) ijklj.k v) Z ikjxE; f>Yy h d s}kjk mRiUu gksrk gSA
(B) leqnzh t y d k foy o.khd j.k foijhr ijklj.k }kjk gksrk gSA
(C*) nks
uksalgh d Fku gSaA
(D) d ks
bZHkh d Fku lgh ughagSA
K-2. The relationship between osmotic pressure at 273 K when 10 g glucose (P1) ,10 g urea (P2) and 10 g
sucrose (P3) are dissolved in 250 mL of water is :
273 K ij ijklj.k nkc d se/; lEcU/k D;k gS
]t c 10 g Xy wd kst (P1) 10 g ;wfj;k(P2) rFkk10 g lwØ kst (P3) d ks250
mL t y es a?kksy k t krk gS\
(A) P1 > P2 > P3 (B) P3 > P1 > P2 (C*) P2 > P1 > P3 (D) P2 > P3 > P1

Corporate Office: CG Tower, A-46 & 52, IPIA, Near City Mall, Jhalawar Road, Kota (Raj.)-324005
Website : www.resonance.ac.in | E-mail : contact@resonance.ac.in
Toll Free : 1800 200 2244 | 1800 258 5555 | CIN: U80302RJ2007PLC024029
ADV-SCP - 19
Solution & Colligative Properties
K-3. Osmotic pressure of blood is 7.40 atm at 27°C. Number of moles of glucose to be used per litre for an
intravenous injection that is to have the same osmotic pressure as blood is :
27°C ij jDr d k ijklj.k nkc 7.40 atm gS aA vUr%f'kjh; bat sD'ku d sfy , izfr y hVj Xy wd kst d seksy ksad h la[;k t ks
fd jDr d sleku gh ijklj.k nkc mRiUu d jrh gS] fuEu gS%
(A*) 0.3 (B) 0.2 (C) 0.1 (D) 0.4

K-4. The total concentration of dissolved particles inside red blood cells is approximately 0.30 M and the membrane
surrounding the cells is semipermeable. What would the osmotic pressure (in atmosphere) inside the cells
become if the cells were removed from the blood plasma and placed in pure water at 298 K ?
y ky jDr d ksf'kd kvksad svUnj ?kqfy r d .kksad h d qy lkUnzrk y xHkx 0.30 M gSrFkk d ksf'kd k d spkjksvksj d h f>Yy h
v) ZikjxE; gSA ;fn d ksf'kd k d ksjDr Iy kTek lsvy x fd ;k x;k rFkk 298 K ij 'kq) t y esaj[kk x;k gS]rksd ksf'kd k
d svUnj ijklj.k nkc ¼ok;qe.My h; esa½ D;k gksxk \
(A*) 7.34 atm (B) 1.78 atm (C) 2.34 atm (D) 0.74 atm

K-5. Osmotic pressure of a solution of glucose is 1.20 atm and that of a solution of cane sugar is 2.5 atm. The
osmotic pressure of the mixture containing equal volumes of the two solutions will be
Xy wd kst d s,d foy ;u d k ijklj.k nkc 1.20 atm rFkk xUusd h 'kd Zjk d s,d foy ;u d k ijklj.k nkc 2.5 atm gSaA
nksuksafoy ;uksad sleku vk;ru ;qDr feJ.k d k ijklj.k nkc gksxk &
(A) 2.5 atm (B) 3.7 atm (C*) 1.85 atm (D) 1.3 atm.
K-6. Consider following cases :
I : 2M CH3COOH solution in benzene at 27°C where there is dimer formation to the extent of 100%
II : 0.5 M KCl aq. solution at 27°C, which ionises 100%
Which is/are true statements(s) :
(A*) both are isotonic (B) I is hypertonic (C) II is hypotonic (D) none is correct
fuEu ifjfLFkfr;ksad k voy ksd u d hft ,A
I : 27ºC ij csat hu eas2 M CH3COOH foy ;u gS;gk¡f}y d d k 100% fuekZ.k gksrk gSA
II : 27ºC ij 0.5 M KCl d k t y h; foy ;u] t ks100% vk;fur gksrk gSA
fuEu easd kSulk@d kSulsd Fku lgh gS@ gSa\
(A*) nksuksaleijkljh gS (B) I vfrijkljh gSa (C) II vYiijkljh gS (D) d ks bZHkh lgh ughagS

PART - III : MATCH THE COLUMN


Hkkx - III : LrEHk l qesfy r d hft , (MATCH THE COLUMN )
1_. Column – I Column – II
If at a particular temperature, the density of 18 M H2SO4 is 1.8 g cm–3. Then :
(A) Molality (p) 0.1
(B) % concentration by wt. of solute (q) 0.9
(C) mole fraction of H2SO4 (r) 500
(D) mole fraction of H2O (s) 98
LrEHk– I LrEHk– II
;fn ,d fo'ks"k rki ij 18 M H2SO4 d k ?kuRo 1.8 g cm–3 gS] rc %
(A) eksy y rk (p) 0.1
(B) Hkkjkuq
l kj foy s; d h % lkUnzrk (q) 0.9
(C) H2SO4 d k eks y izHkkt (r) 500
(D) H2O d k eks y izHkkt (s) 98
Ans. (A)  (r) ; (B)  (s) ; (C)  (q) ; (D)  (p)

Corporate Office: CG Tower, A-46 & 52, IPIA, Near City Mall, Jhalawar Road, Kota (Raj.)-324005
Website : www.resonance.ac.in | E-mail : contact@resonance.ac.in
Toll Free : 1800 200 2244 | 1800 258 5555 | CIN: U80302RJ2007PLC024029
ADV-SCP - 20
Solution & Colligative Properties
2. Column – I Column – II
(A) Acetone + CHCl3 (p) Smix. > 0
(B) Ethanol + Water (q) Vmix. > 0
(C) C2H5Br + C2H5I (r) Hmix. < 0
(D) Acetone + Benzene (s) Maximum boiling azeotropes
(t) Minimum boiling azeotropes
LrEHk- I LrEHk- II
(A) ,lhVksu + CHCl3 (p) SfeJ.k > 0
(B) ,Fks
ukWy + t y (q) VfeJ.k > 0
(C) C2H5Br + C2H5I (r) HfeJ.k < 0
(D) ,lhVks
u + csat hu (s) vf/kd re fLFkjDokaFkh
(t) U;w
ure fLFkjDokaFkh
Ans. (A) – (p, s, r); (B) – (p, q, t); (C) – (p); (D) – (p, q, t)

3. Column – I Column – II


Assuming all the solutes are non volatile and all solutions are ideal and neglect the hydrolysis of cation and
anion.
(A) 10 ml 0.1 M NaOH aqueous solution is added (p) Osmotic pressure of solution increases
to 10 ml 0.1 M HCl aqueous solution
(B) 10 ml 0.1 M NaOH aqueous solution is added (q) Vapour pressure of solution increases
to 10 ml 0.1 M CH3COOH aqueous solution
(C) 10 ml 0.1 M HCl aqueous solution is added (r) Boiling point of solution increases
to 10 ml 0.1 M NH3 aqueous solution
(D) 10 ml 0.1 M HCl aqueous solution is added (s) Freezing point of solution increases
to 10 ml 0.1 M KOH aqueous solution
l Hkh foy s;ksad ksv ok"i'khy rFkk foy ;uksad ksv kn'kZekusarFkk /kuk;u] _ .kk;u d k t y v i?kVu ux.; ekusaA
LrEHk – I LrEHk – II
(A) 0.1 M NaOH d s10 fey h- t y h; foy ;u d ks (p) foy ;u d k ijkl j.k nkc c<+rk gSA
0.1 M HCl d s10 fey h- t y h; foy ;u es afey kusij
(B) 0.1 M NaOH d s10 fey h- t y h; foy ;u d ks (q) foy ;u d k ok"i nkc c<+rk gSA
0.1 M CH3COOH d s10 fey h- t y h; foy ;u es afey kusij
(C) 0.1 M HCl d s10 fey h- t y h; foy ;u d ks (r) foy ;u d k DoFkuka
d c<+rk gSA
0.1 M NH3 d s10 fey h- t y h; foy ;u es afey kusij
(D) 0.1 M HCl d s10 fey h- t y h; foy ;u d ks (s) foy ;u d k fgeka
d c<+rk gSA
0.1 M KOH d s10 fey h- t y h; foy ;u es afey kusij
Ans. (A) – q, s ; (B) – q, s ; (C) – q, s ; (D) – q, s

Corporate Office: CG Tower, A-46 & 52, IPIA, Near City Mall, Jhalawar Road, Kota (Raj.)-324005
Website : www.resonance.ac.in | E-mail : contact@resonance.ac.in
Toll Free : 1800 200 2244 | 1800 258 5555 | CIN: U80302RJ2007PLC024029
ADV-SCP - 21
Solution & Colligative Properties

 Marked Questions may have for Revision Questions.


 fpfUgr iz
'u nksgjkus;ksX; iz'u gSA

PART - I : ONLY ONE OPTION CORRECT TYPE


Hkkx-I : d soy ,d l gh fod Yi çd kj (ONLY ONE OPTION CORRECT TYPE)

Section (A) : Concentration terms (Revision of mole)


[k.M (A) : lkUnzrk in (eksy d k la'kks/ku)
1. All of the water in a 0.20 M solution of NaCl was evaporated and a 0.150 mol of NaCl was obtained. What
was the original volume of the sample ?
0.20 M NaCl d sfoy ;u d k lEiw.kZt y okf"ir d jusij 0.150 eksy NaCl d sizkIr gksrsgSrksizkn'kZd k okLrfod
vk;ru D;k gksxk \
(A) 30 mL (B) 333 mL (C*) 750 mL (D) 1000 mL

2. A 20.0 mL sample of CuSO4 solution was evaporated to dryness, leaving 0.967 g of residue. What was the
molarity of the original solution ? (Cu = 63.5)
CuSO4 foy ;u d k 20.0 mL izkn'kZd ks'kq"d d jusd sfy , okf"ir fd ;k x;kA 0.967 g vof'k"V 'ks"k jg t krk gS] rks
okLrfod foy ;u d h eksy jrk D;k gksxh \ (Cu = 63.5)
(A) 48.4 M (B) 0.0207 M (C) 0.0484 M (D*) 0.303 M

3. The vapour pressure of water at 20°C is 17.54 mmHg. What will be
the vapour pressure of the water in the apparatus shown after the
piston is lowered, decreasing the volume of the gas above the liquid
to one half of its initial volume (assume temperature constant).
(A) 8.77 mmHg
(B*) 17.54 mmHg
(C) 35.08 mmHg
(D) between 8.77 and 17.54 mmHg
20°C ij t y d k ok"i nkc 17.54 mmHg gS ]fn[kk;sx;smid j.k esafiLVu
d ksuhpsd jusd sckn nzo ij xSl d kvk;ru bld sizkjfEHkd vk;ru d kvkèkk
d e gkst krkgSrksmid j.kesat y d kok"inkc gksxkA (ekukrkieku fLFkj gS).
(A) 8.77 mmHg
(B*) 17.54 mmHg
(C) 35.08 mmHg
(D) 8.77 rFkk 17.54 mmHg d se/;

4. A sample of air is saturated with benzene (vapor pressure = 100 mm Hg at 298 K) at 298K, 750mm Hg
pressure. If it is isothermally compressed to one third of its initial volume, the final pressure of the system
is
298 K, 750 mm Hg ij ok;qd s,d çkn'kZd kscs at hu (298K ij ok"i nkc = 100mm Hg) d slkFk larÌr fd ;k t krk
gSA ;fn ;g bld sizkjafHkd vk;ru d k ,d frgkbZlerkih; : i lslaihfM+r fd ;k t krk gS] rksfud k; d k vfUre nkc
fuEu gS%
(A) 2250 torr (B) 2150 torr (C*) 2050 torr (D) 1950 torr

5. Pressure cooker reduces cooking time because


(A) the heat is more evenly distributed inside the cooker.
(B) a large flame is used.
(C*) boiling point of water is elevated.
(D) whole matter is converted into steam.

Corporate Office: CG Tower, A-46 & 52, IPIA, Near City Mall, Jhalawar Road, Kota (Raj.)-324005
Website : www.resonance.ac.in | E-mail : contact@resonance.ac.in
Toll Free : 1800 200 2244 | 1800 258 5555 | CIN: U80302RJ2007PLC024029
ADV-SCP - 22
Solution & Colligative Properties
izs'kj d qd j esa[kkuk id kusesale; d e y xrk gSa] D;ksafd
(A) d qd j d svUnj Å "ek vf/kd leku : i lsforfjr gksrh gSA
(B) ,d cM+ h Toky k d ke esay h t krh gSA
(C*) t y d k DoFkuka d mUu;u gksrk gSA
(D) lkjk inkFkZ ] Hkki esacny t krk gSA
6. Water and chlorobenzene are immiscible liquids. Their mixture boils at 89ºC under a reduced pressure of
7.7 × 104 Pa. The vapour pressure of pure water at 89ºC is 7 × 104 Pa. Weight percent of chlorobenzene
in the distillate is:
t y o Dy ksjkscsfUt u vfeJ.kh; nzo gSaA 7.7 × 104 Pa d s,d lkekU; nkc d svUrxZr 89ºC ij bud k feJ.k mcy rk gSA
89°C ij 'kq) t y d k ok"inkc 7 × 104 Pa gSA vklqr esaDy ksjkscsfUt u d k Hkkj izfr'kr fuEu gS&
(A) 50 (B) 60 (C) 78.3 (D*) 38.46

7. If two liquids A (PºA =100 torr) and B (PºB = 200 torr) are completely immiscible with each other, each one
will behave independently of the other, are present in a closed vessel. The total vapour pressure of the
system will be: (A) less than 100 torr (B) less than 200 torr
(C) between 100 to 200 torr (D*) 300 torr
;fn nksnzo A(PºA =100 torr) o B (PºB = 200 torr) ,d nwl jsd slkFkiw.kZ: i lsvfeJ.kh; gSaA izR;sd ,d nwl jsd slkis{k
Lora=k : i lsO;ogkj d jrsgSo ,d cUn ik=k esamifLFkr gksarc ra=k d k d qy ok"i nkc gksxk &
(A) 100 torr lsd e gks
xk (B) 200 torr lsd e gks xk
(C) 100 o 200 torr d schp gks
xk (D*) 300 torr

Section (F) : Completely miscible liquids : Raoult’s law


[k.M (F) : iw.kZr% feJ.kh; nzo % jkWÅ YV fu;e
8. Consider two liquids A & B having pure vapour pressures PA & PB forming an ideal solution. The plot of

1 1
XA v/s YA (where XA and YA are the mole fraction of liquid A in liquid and vapour phase respectively) is

linear with slope and Y intercepts respectively :

PºA (PAº  PBº ) PºA (PBº  PAº )


º
PB (PAº  PBº )
º
PB (PBº  PAº )
(A) º and (B*) º and (C) and (D) and
PB PBº PB PBº PºA PBº PºA PBº

1 1
ekuk nksnzo A vkSj B ft ud k 'kq) ok"i nkc PA & PB gSa] vkn'kZfoy ;u cukrsgSA XA dk YA d slkFk vkjs[k(;gk¡
XA vkS
j YA nzo A d sØ e'k%nzo voLFkk vkSj ok"i voLFkk esaeksy izHkkt gSa½ <y ku (slope) d slkFk js[kh; vkSj Y vUr%
[k.M Ø e'k% fuEu gS%
PºA (PAº  PBº ) PºA (PBº  PAº )
º
PB (PAº  PBº )
º
PB (PBº  PAº )
(A) º
PB
rFkk PBº
(B*) º
PB
rFkk PBº
(C)
PºA
rFkk PBº
(D)
PºA
rFkk PBº
benzene
2200
9. Assuming the formation of an ideal solution, determine
1800
the boiling point of a mixture containing 1560 g Vapour 1350 chlorobenzene
benzene (molar mass = 78) and 1125 g Pressure
1000
chlorobenzene (molar mass = 112.5) using the 540
following against an external pressure of 1000 Torr. 400
300
200
90 100 110 120
t(cº)
(A) 90ºC (B*) 100ºC (C) 110º (D) 120ºC

Corporate Office: CG Tower, A-46 & 52, IPIA, Near City Mall, Jhalawar Road, Kota (Raj.)-324005
Website : www.resonance.ac.in | E-mail : contact@resonance.ac.in
Toll Free : 1800 200 2244 | 1800 258 5555 | CIN: U80302RJ2007PLC024029
ADV-SCP - 23
Solution & Colligative Properties

vkn'kZfoy ;u ekursgq, ,d feJ.k d k DoFkukad Kkr d jks cast hu


2200
ft lesa1560 g csat hu (eksy j nzO;eku = 78) vkSj 1125 g
1800
Dy ksjkscsat hu (eksy j nzO;eku = 112.5) gSrFkk 1000 Vksj ok"i nkc 1350 Dyksjkscsat hu
d k ckg; nkc ç;qDr fd ;k t krk gSA 1000
540
(A) 90ºC (B*) 100ºC 400
300
(C) 110ºC (D) 120ºC 200
90 100 110 120
Section (G) : Non-ideal Solutions t(cº)
[k.M (G) : v ukn'kZfoy ;u
10. Given P-x curve for a non-ideal liquid mixture (Fig.). Identify the correct T-x curve for the same mixture.

(A) (B*) (C) (D)

vukn'kZnzfor feJ.k d sfy , P-x oØ fn;k x;k gSA leku feJ.k d sfy , lgh T-x oØ d ksigpkfu;sA

800
nkc (mm Hg)

600
400
200

0 0.2 0.4 0.6 0.8 1.0


eks
y iz
Hkkt
DoFkukad

DoFkukad
DoFkuka
d

(A) (B*) (C) (D)


0 0.2 0.4 0.6 0.8 1.0 0 0.2 0.4 0.6 0.8 1.0 0 0.2 0.4 0.6 0.8 1.0
eks
y iz
Hkkt eks
y iz
Hkkt eks
y iz
Hkkt

Corporate Office: CG Tower, A-46 & 52, IPIA, Near City Mall, Jhalawar Road, Kota (Raj.)-324005
Website : www.resonance.ac.in | E-mail : contact@resonance.ac.in
Toll Free : 1800 200 2244 | 1800 258 5555 | CIN: U80302RJ2007PLC024029
ADV-SCP - 24
Solution & Colligative Properties

11. If vapour pressures of pure liquids ‘A’ & ‘B’ are 300 and 800 torr respectively at 25°C. When these two
liquids are mixed at this temperature to form a solution in which mole percentage of ‘B’ is 92, then the total
vapour pressure is observed to be 0.95 atm. Which of the following is true for this solution.
25ºC ij ;fn 'kq) nzO;ks‘A’ o ‘B’ d sok"i nkc Ø e'k%300 vkSj 800 Vksj gSaA t c bu nksnzoksd ksbl rki ij ,d foy ;u
cukusd sfy ;sfefJr fd ;k t krk gS] ft lesaB d k eksy izfr'kr 92 gSA rc d qy ok"i nkc 0.95 atm izkIr gksrk gSA bl
foy ;u d sfy ;sfuEu esalsd kSulk lR; gSA
(A) Vmix > 0 (B*) Hmix < 0 (C) Vmix = 0 (D) Hmix = 0

Section (H) : Degree of Ionisation/Dissociation for Weak Electrolytes


[k.M (H) : nqcZy oS| qr v i?kV~;ksad sv k;uu@fo;kst u d h ek=kk
12. Barium ions, CN¯ and Co2+ form an ionic complex. If that complex is supposed to be 75% ionised in water
with vant Hoff factor ‘ i ’ equal to four, then the coordination number of Co2+ in the complex can be :
(A) Six (B*) Five (C) Four (D) Six and Four both
csfj;e vk;u] CN¯ vkSj Co2+ ,d vk;fud lad qy cukrsgSA ;fn ekuk ;g lad qy t y esa75% vk;fur gksrk gS] lkFk
gh okUVgkWQ xq.kkad ‘ i ’ pkj d scjkcj gks] rc lad qy esaCo2+ d h leUo; la[;k gksxh &
(A) N% (B*) ik¡
p (C) pkj (D) N% rFkk pkj nks
uksa
Section (I) : Relative lowering of vapour pressure
[k.M (I) : ok"i nkc d k v kisf{kd v oueu
13. In the following aqueous solutions.
(A) 1 m sucrose (B) 1 m potassium ferricyanide and (C) 1 m potassium sulphate
maximum value of vapour pressure of solution is that of :
(A*) A (B) B (C) C (D) equal
fuEu t y h; foy ;uksaesa
(A) 1 m lw Ø kst (B) 1 m iks
Vsf'k;e Q sjksl k;ukbM rFkk (C) 1 m iks
Vsf'k;e lYQ sV
fuEu d sfoy ;u d sfy , ok"i nkc d k vf/kd re eku gksxk &
(A*) A (B) B (C) C (D) cjkcj

14. Dry air is slowly passed through three solutions of


different concentrations, c1, c2 and c3 ; each containing
(non volatile) NaCl as solute and water as solvent, as
shown in the Fig. If the vessel 2 gains weight and the
vessel 3 loses weight, then
(A) c2 > c3 (B) c1 < c2
(C) c1 < c3 (D*) Both (A) and (B)

rhu fofHkUu lkUnzrkvksac1, c2 rFkkc3 d srhu foy ;uksaesals'kq"d


ok;qd ks/khjs& /khjsizokfgr fd ;k t krk gS]izR;sd esa(vok"i'khy )
NaCl d ksfoy s ; rFkk t y d ksfoy k;d d s: i esaiz;qDr fd ;k
x;k gS] t Sl k fd fp=k esaiznf'kZr fd ;k x;k gSA ;fn ik=k& 2 esa
Hkkj esaof̀) gksrh gSrFkk ik=k& 3 esaHkkj esad eh gksrh gS] rks
Vessel-1 Vessel-2 Vessel-3
(A) c2 > c3 (B) c1 < c2 ik=k-1 ik=k-2 ik=k-3
(C) c1 < c3 (D*) (A) rFkk (B) nks
uksa

Corporate Office: CG Tower, A-46 & 52, IPIA, Near City Mall, Jhalawar Road, Kota (Raj.)-324005
Website : www.resonance.ac.in | E-mail : contact@resonance.ac.in
Toll Free : 1800 200 2244 | 1800 258 5555 | CIN: U80302RJ2007PLC024029
ADV-SCP - 25
Solution & Colligative Properties
Section (J) : Elevation of Boiling Point & Depression of Freezing Point
[k.M (J) : DoFkukad esamUu;u o fgekad esav oueu
15. When only a little quantity of HgCl2(s) is added to excess KI(aq) to obtain a clear solution, which of the
following is true for this solution? (no volume change on mixing).
The reaction is 4KI(aq.) + HgCl2(s) — K2[HgI4] (aq.) + 2KCl (aq.)
(A) Its boiling and freezing points remain same (B*) Its boiling point is lowered
(C) Its vapour pressure become lower (D) Its boiling point is raised
(E) Its freezing point is lowered.
t c ,d lqLi"V foy ;u izkIr d jusd sfy , vkf/kD; KI (t y h;) d slkFk HgCl2(s) d h FkksM +h ek=kk fey k;h t krh gSrks
bl foy ;u d sfy , buesalsd kSulk d Fku lR; gS\ ¼fefJr d jusij d ksbZvk;ru ifjorZu ughagksrk gSA½
4KI(aq.) + HgCl2(s) — K2[HgI4] (aq.) + 2KCl (aq.)
(A) bld k DoFkuka
d rFkk fgekad leku jgrk gS (B*) bld k DoFkuka
dfuEu gkst krk gS
(C) bld k ok"i nkc d e gkst krk gS (D) bld sDoFkuka
d esaof̀) gksrh gS
(E) bld k fgeka
d fuEu gkst krk gSA
16. x mole of KCI and y mole of BaCl2 are both dissolved in 1 kg of water. Given that x + y = 0.1 and Kf for water
is 1.85 K/molal, what is the observed range of Tf, if the ratio of x to y is varied ?
(A*) 0.37º to 0.555º (B) 0.185º to 0.93º (C) 0.56º to 0.93º (D) 0.37º to 0.93º
x eks
y KCI o y eksy BaCl2 nksukas1 kg t y esa?kksy sx,A fn;k gSx + y = 0.1 rFkk t y d sfy , Kf = 1.85 K/molal gSA
;fn x o y d k vuqikr ifjofrZr gksrk gS] rksTf d h izsf{kr ijkl D;k gksxh \
(A*) 0.37º ls0.555º (B) 0.185º ls0.93º (C) 0.56º ls0.93º (D) 0.37º ls0.93º

17. For a solution of 0.849 g of mercurous chloride in 50 g of HgCl2() the freezing point depression is 1.24ºC.
Kf for HgCl2 is 34.3. What is the state of mercurous chloride in HgCl2 ? (Hg – 200, Cl – 35.5)
(A*) as Hg2Cl2 molecules (B) as HgCl molecules
+ –
(C) as Hg and Cl ions (D) as Hg22+ and Cl– ions
HgCl2() d s50 g es
a0.849 g d seD;wZjl Dy ksjkbM foy ;u d sfy ;sfgekad voueu 1.24ºC gSA HgCl2 d sfy ;sKf 34.3
gSA HgCl2 esaeD;wZjl Dy ksjkbM d h voLFkk D;k gksxhA (Hg – 200, Cl – 35.5)
(A*) Hg2Cl2 t S
l sv.kqv ksad sleku (B) HgCl t Sl sv.kqv ksad sleku
+ 2+
(C) Hg rFkk Cl t S

l svk;uksad sleku (D) Hg2 rFkk Cl– t S l svk;uksad sleku
Section (K) : Osmotic Pressure
[k.M (K) : ijklj.k nkc
18. FeCl3 on reaction with K4[Fe(CN)6] in aq. solution gives blue colour. These
are separated by a semipermeable membrane PQ as shown. Due to
osmosis there is-
(A) blue colour formation in side X
(B) blue colour formation in side Y
(C) blue colour formation in both of the sides X and Y
(D) no blue colour formation
FeCl3, K4[Fe(CN)6] d slkFk t y h; foy ;u es avfHkfØ ;k d j uhy k jax nsrk gSaA
uhpsn'kkZ;svuql kj bld ksv) ZikjxE; f>Yy h PQ }kjk iF̀kd fd ;k t krk gSaA
ijklj.k d sd kj.k ;gk¡&
(A) X d h vksj uhy sjax d k fuekZ.k gksrk gSA
(B) Y d h vksj uhy sjax d k fuekZ.k gksrk gSA
(C) nks
uksaX o Y d h vksj uhy sjax d k fuekZ.k gksrk gSA
(D*) uhy sjax d k fuekZ.k ughagksrk gSA

Corporate Office: CG Tower, A-46 & 52, IPIA, Near City Mall, Jhalawar Road, Kota (Raj.)-324005
Website : www.resonance.ac.in | E-mail : contact@resonance.ac.in
Toll Free : 1800 200 2244 | 1800 258 5555 | CIN: U80302RJ2007PLC024029
ADV-SCP - 26
Solution & Colligative Properties
19. Two beakers, one containing 20 ml of a 0.05 M aqueous solution of a non volatile, non electrolyte and the
other, the same volume of 0.03 M aqueous solution of NaCl, are placed side by side in a closed enclosure.
What are the volumes in the two beakers when equilibrium is attained ? Volume of the solution in the first
and second beaker are respectively.
(A) 21.8 ml and 18.2 mL (B*) 18.2 mL and 21.8 mL
(C) 20 mL and 20 mL (D) 17.1 mL and 22.9 mL
nkschd j gS],d esa0.05 M t y h; foy ;u d k 20 ml vok"i'khy ]oS| qr vuvi?kV~; gSvkSj nwl jsesamlh leku vk;ru
esaNaCl d st y h; foy ;u d k 0.03 M gSA ft Ugsacan ik=k esa,d d sikl ,d j[kk t krk gSA t c lkE; izkIr gksrk gS]rks
nksuksachd jksad svk;ru D;k gksxsa\ izFke rFkk f}rh; chd j d sfoy ;uksad k vk;ru Ø e'k% gSa&
(A) 21.8 ml vkS j 18.2 mL (B*) 18.2 mL vkS j 21.8 mL
(C) 20 mL vkS j 20 mL (D) 17.1 mL vkS j 22.9 mL
20. The vapor pressures of chlorobenzene and water at different temperatures are

At what temperature will Cl steam-distillation under a total pressure of 800 mmHg?
fofHkUu rki ij Dy ksjkscsUt hu rFkk t y d k ok"i nkc fuEu gS&

800 mmHg d sd q
y nkc d svUrZxr Cl Hkki&vklou fd l rki ij gksxk \
(A)  95ºC (B*)  92ºC (C)  94ºC (D)  89ºC

PART - II : SINGLE AND DOUBLE VALUE INTEGER TYPE


Hkkx - II : ,d y ,oaf}&iw.kk±d eku izd kj ¼SINGLE AND DOUBLE VALUE INTEGER TYPE½
Section (A) : Concentration terms (Revision of mole)
[k.M (A) : lkUnzrk in (eksy d k la'kks/ku)
1. What volume of 98% sulphuric acid (in ml) should be mixed with water to obtain 200 mL of 15% solution of
sulphuric acid by weight ? Given density of H2O = 1.00 g cm–3, sulphuric acid (98%) = 1.88 g cm–3 and
sulphuric acid (15%) = 1.12 g cm–3.
98% H2SO4 foy ;u d kfd ruk vk;ru]t y es afey k;k t k, fd 15% H2SO4 d k200 mL foy ;u Hkkj d svuql kj izkIr
gksld s\
H2O d k ?kuRo = 1.00 g cm–3 , 98% H2SO4 d k ?kuRo = 1.88 g cm–3 , 15% H2SO4 d k ?kuRo = 1.12 g cm–3.
Ans. 18
2. The vapour pressure of fluorobenzene at t°C is given by the equation
1250
log p (mm Hg) = 7.0 –
t  220
Calculate the boiling point of the liquid in ºC if the external (applied) pressure is 5.26% more than required
for normal boiling point. (log 2 = 0.3)
t°C ij ¶y ks
jkscsUt hu d k ok"i nkc fuEu lehd j.k }kjk fn;k t krk gSA
1250
log p (mm Hg) = 7.0 –
t  220
ºC es
anzo d k DoFkukad Kkr d hft ,];fn ¼iz;qDr½ckâ; nkc lkekU; DoFkukad d sfy , vko';d ckâ; nkc d h rqy uk esa
5.26% vf/kd gS A (log 2 = 0.3)
Ans. 85

Corporate Office: CG Tower, A-46 & 52, IPIA, Near City Mall, Jhalawar Road, Kota (Raj.)-324005
Website : www.resonance.ac.in | E-mail : contact@resonance.ac.in
Toll Free : 1800 200 2244 | 1800 258 5555 | CIN: U80302RJ2007PLC024029
ADV-SCP - 27
Solution & Colligative Properties
3. At 300 K, 40 mL of O3 (g) dissolves in 100 g of water at 1.0 atm. What mass of ozone (in gram) dissolved
in 1600 g of water at a pressure of 4.0 atm at 300 K ?
300 K ij]1 atm ij 100 g t y es a40 mL O3 (g) d ks?kksy k t krk gSA 300 K o 4.0 atm d snkc ij t y d s1600 g esa
vkst ksu d k fd ruk nzO;eku ¼xzke esa½ ?kqy sxk \
Ans. 5

4. Calculate the concentration of CO2 in a soft drink that is bottled with a partial pressure of CO2 of 4 atm over
the liquid at 25ºC. The Henry’s law constant for CO2 in water at 250C is 3.1 × 10–2 mol/litre–atm. Write
answer after multiplying by 100.
,d eǹqis; (soft drink) esaCO2 d h lkUnzrk d h x.kuk d jks] t ks25ºC ij nzo d sÅ ij CO2 d s4 atm d svkaf'kd nkc
d slkFk cksry esagSaA t y esaCO2 d sfy , gsujh fu;e fLFkjkad 3.1 × 10–2 mol/litre–atm gSA
Ans. 12 mol/litre.

5. An ideal aqueous solution containing liquid A(M.Wt. = 128) 64% by weight has a vapour pressure of 145
mm Hg. If the vapour pressure of A is x mm of Hg and that of water is 155 mm Hg at the same temperature.
Then find x/5. The solutions is ideal.
,d vkn'kZt y h; foy ;u] t ksnzo A (v.kqHkkj = 128) 64% Hkkj@Hkkj j[krk gS] d sfy , ok"i nkc 145 mm Hg gSA ;fn
leku rki ij A d k ok"i nkc x mm Hg rFkk t y d k 155 mm Hg gS] rksX/5 Kkr d hft ,A foy ;u vkn'kZgSA
Ans. 21
6. A and B form ideal solutions; at 50ºC, PAº is half PBº. A solution containing 0.2 mole of A and 0.8 mole of B
has a normal bolling point of 50ºC. Find 18 × PBº. (PBº is in atm)
50ºC ij A rFkkB vkn'kZfoy ;u cukrsgS ]PAº d k eku PBº d k vk/kk gSA ,d foy ;u]t ks0.2 eksy A o 0.8 eksy B j[krk
gS] d sfy , lkekU; DoFkukad 50ºC gSA 18 × PBº Kkr d hft ,A (PBº, atm esagSA)
Ans. 20
7. The vapour pressure of pure liquid A at 300 K is 577 Torr and that of pure liquid B is 390 Torr. These two
compounds form ideal liquid and gaseous mixtures. Consider the equilibrium composition of a mixture in
which the mole fraction of A in the vapour is 0.35. Find the mole % of A in liquid.
300 K rki ij 'kq
) nzo A d k ok"i nkc 577 Vksj rFkk 'kq) nzo B d k ok"i nkc 390 Vksj gSaA ;g nksuksa;kSfxd ]vkn'kZnzo
rFkk xSl h; feJ.k cukrsgSaA feJ.k d slkE; laxBu esaok"i voLFkk esaA d k eksy izHkkt 0.35 gSA nzo esaA d k eksy %
Kkr d hft ,A
Ans. 27
8. Two liquids A and B are miscible over the whole range of composition and may be treated as ideal (obeying
Raoult’s law.) At 350 K the vapour pressure of pure A is 24.0 kPa and of pure B is 12.0 kPa. A mixture of
60% A and 40% B is distilled at this temperature. A small amount of the distillate is collected and and
redistilled at 350 K; what is the mole percent of B in the second distillate ?
laxBu d siw.kZijkl d sfy , nksnzo A rFkkB feJ.kh; gSrFkk vkn'kZ: i esamipkfjr fd , t krsgSa(jkWÅ YV fu;e d k
iky u d jrsgS)A 350 K ij 'kq) A d k ok"i nkc 24.0 kPa rFkk 'kq) B d k ok"i nkc 12.0 kPa gSA ,d feJ.k ft lesa
60% A o 40% B gS ]d ksbl rki ij vklfor fd ;k t krk gSA vklfor gqbZinkFkZd h vYi ek=kk d ks,d f=kr fd ;k t krk
gSrFkk mls350 K ij iqu% vklfor fd ;k t krk gS] f}rh; vklou esaB d k eksy izfr'kr D;k gS\
Ans. 14

9. How many facts related to CHCl3 + ethylmethylketone solution are correct ?


(a) It shows negative derivation. (b) It forms maximum boiling azeotropic mixture
(c) S > 0 (d) G < 0
(e) Components can be separated by fractional distillation.
CHCl3 + ,fFky es
fFky d hVksu foy ;u lslEcfU/kr lgh rF; fd rusgSa\
(a) ;g _ .kkRed fopy u n'kkZ rk gSA (b) ;g vf/kd re DoFkukad fLFkjDokaFkh feJ.k gSA
(c) S > 0 (d) G < 0
(e) ?kVd ks
ad ksizHkkt h vklou }kjk iF̀kd fd ;k t k ld rk gSA
Ans. 4 (A,B,C,D)

Corporate Office: CG Tower, A-46 & 52, IPIA, Near City Mall, Jhalawar Road, Kota (Raj.)-324005
Website : www.resonance.ac.in | E-mail : contact@resonance.ac.in
Toll Free : 1800 200 2244 | 1800 258 5555 | CIN: U80302RJ2007PLC024029
ADV-SCP - 28
Solution & Colligative Properties
10. How many of the following solutions show negative deviation from Raoult's Law ?
Liquid A + Liquid B
(i) (CH3)2CO + CS2
(ii) CCl4 + C6 H6
(iii) CCl4 + CHCl3
(iv) H2 O + C2H5OH
(v) (C2H5)2O + (CH3)2CO
(vi) CH3COOH + C5H5N (pyridine)
(vii) C6H5NH2 + (CH3)2CO
(viii) C6H5Cl + C6H5Br
(ix) Cyclohexane + Ethanol
fuEu esalsfd rusfoy ;u jkWÅ YV fu;e ls_ .kkRed fopy u n'kkZrsgS\
nzo A + nzo B
(i) (CH3)2CO + CS2
(ii) CCl4 + C6 H6
(iii) CCl4 + CHCl3
(iv) H2 O + C2H5OH
(v) (C2H5)2O + (CH3)2CO
(vi) CH3COOH + C5H5N (fijhMhu)
(vii) C6H5NH2 + (CH3)2CO
(viii) C6H5Cl + C6H5Br
(ix) lkbDy ksgsDlsu + ,FksukWy
Ans. 2

11. Determine i (vant-Hoff factor) for a tribasic acid H3A. Assuming first dissociation to be 100%, second
dissociation 50%, third dissociation 20%. (Round off your answer to nearest integer).
,d f=k{kkfj; vEy H3A d sfy , ok.VgkWQ xq.kkad i d h x.kuk d jksA izFke fo;kst u d ks100% f}rh; fo;kst u d ks50%
rFkk rr̀h; fo;kst u d ks20% ekursgq,A ¼viuk mÙkj fud Vre iw.kk±d esnhft ,A½
Ans. 3

12. 0.1 mole XY2 is dissolved in 2L water, where it ionizes to give X2+ and Y22–. Observed osmotic pressure is 3
atm. Molar mass of X is 24 and Y is 32. Find Mobserved + 2i (where Mobserved is observed molar mass of XY2)
(Use R = 1/12 L-atm/mol.K and temperature is 87°C)
0.1 eks
y XY2 d ks2L t y esa?kksy k t krk gS] t gk¡;g vk;fur gksd j X2+ rFkk Y22– nsrk gSA izsf{kr ijklj.k nkc 3 atm
gSA X d k eksy j nzO;eku 24 rFkk Y d k 32 gSA Mizsf{kr + 2i Kkr d hft ,A (t gk¡Mizsf{kr, XY2 d k izsf{kr eksy j nzO;eku gSA)
(iz;ksx d jsa] R = 1/12 L-atm/mol.K rFkk rki = 87°C)
Ans. 48

13. Dry air was successively passed through a solution of 5g solute in 80 g water and then through pure water.
The loss in weight of solution was 2.5 g and that of pure water was 0.04 g. What is mol. wt of solute ?
'kq"d ok;qd ks80 g t y esa5g foy s; d s,d foy ;u esalso fQ j 'kq) t y esalsØ ekxr : i lsizokfgr fd ;k t krk gSA
foy ;u d sHkkj esaá l 2.5 g Fkk o 'kq) t y d sfoy ;u d sHkkj esá l 0.04 g FkkA foy s; d k v.kqHkkj D;k gS\
Ans. 70
14. How many grams of sucrose (C12H22O11) must be dissolved in 90 g of water to produce a solution over
which the relative humidity is 80%? Assume the solution is ideal. Give your answer after dividing by 10.
,d foy ;u d kscukusd sfy , 90 g t y esalwØ kst C12H22O11 d sfd rusxzke ?kksy st kuspkfg;s] rkfd ml foy ;u ij
vkisf{kd vknzZrk 80% gks\ eku y hft , fd foy ;u vkn'kZgSaA viuk mÙkj 10 lsHkkx nsusd si'pkr~nhft ,A
Ans. 43

15. 1.22 g of a monobasic acid is dissolved in 100 g of benzene. Boiling point of solution increases by 0.13ºC
with respect to pure benzene. Find the molecular mass of acid in benzene solvent (in u). Report your
answer after dividing it by 100 and Round it off to nearest integer. (Kb of benzene = 2.6 K kg mol–1).

Corporate Office: CG Tower, A-46 & 52, IPIA, Near City Mall, Jhalawar Road, Kota (Raj.)-324005
Website : www.resonance.ac.in | E-mail : contact@resonance.ac.in
Toll Free : 1800 200 2244 | 1800 258 5555 | CIN: U80302RJ2007PLC024029
ADV-SCP - 29
Solution & Colligative Properties
1.22 ,d y {kkjh; vEy d ks100 g csUt hu esa?kksy kt krkgSrc bl foy ;u d kDoFkukad 'kq) csUt hu ls0.13ºC c<+t krk
gSA csUt hu foy k;d esavEy d kvkf.od nzO;eku (u esa) Kkr d hft ,A viukmÙkj 100 lsHkkx nsd j rFkkfud VorhZiw.kkZd
esanhft ,A (csUt hu d k Kb = 2.6 K kg mol–1).
Ans. 2

16. 1 g of a monobasic acid dissolved in 200 g of water lowers the freezing point by 0.186ºC. On the other hand
when 1 g of the same acid is dissolved in water so as to make the solution 200 mL, this solution requires
K  kg
125 mL of 0.1 M NaOH for complete neutralization. Calculate % dissociation of acid ? (Kf = 1.86 )
mol

200 g t y esa1 g ,d {kkjh; vEy d ks?kksy usij bld k fgekad 0.186ºC lsd e gkst krk gSA mlh vEy d s1 g d kst y
esa?kksy usij 200 mL foy ;u curk gSA ;g foy ;u 0.1 M NaOH d s125 mL d ksiw.kZr;k mnklhu d jusd sfy ,
K  kg
vko';d gSrc vEy d sfy , izfr'kr fo;kst u Kkr d jksA (Kf = 1.86 mol
)
Ans. 60

17._ At 27ºC, a 1.2% solution ( wt./vol.) of glucose is isotonic with 4.0 g/litre of solution of solute X. Find the
molar mass of X, if the molar mass of glucose is 180. (R = 0.082 L atm mol–1 K–1, Molar mass of glucose
= 180 g/mole)
27ºC ij],d Xy w
d kst d k1.2% ( wt./vol.) foy ;u]foy s; X d s4.0 g/litre foy ;u d slkFkleijkljhgSA ;fn Xy wd kst
d k eksy j nzO;eku 180 gS]rksX d k eksy j nzO;eku Kkr d jksA (R = 0.082 L atm mol–1 K–1, Xy wd kst d k eksy j nzO;eku
= 180 g/mole)
Ans. 60

18. 10 g of solute A and 20 g of solute B both are dissolved in 500 ml. of water. The solution has the same
osmotic pressure as 6.67 g of A and 30 g of B are dissolved in the same volume of water at the same
temperature. If the ratio of molar masses of A and B is x/y, find x + y.
foy ;u A d s10 g o foy s; B d s20 g d ks500 ml t y esa?kksy k t krk gSA bl foy ;u d k ijklj.kh nkc ml foy ;u
d sleku gSft lesaleku rki ij t y d sleku vk;ru esaA d s6.67 g o B d s30 g ?kksy st krsgSA A o B d seksy j
nzO;ekuksad k vuqikr x/y gS] rc x + y Kkr d hft ,A
Ans. 4

PART - III : ONE OR MORE THAN ONE OPTIONS CORRECT TYPE


Hkkx - III : ,d ;k ,d l sv f/kd l gh fod Yi çd kj (ONE OR MORE THAN ONE OPTION CORRECT TYPE)
Section (A) : Concentration terms (Revision of mole)
[k.M (A) : lkUnzrk in (eksy d k la'kks/ku)
1. We have 100 mL of 0.1 M KCl solution. To make it 0.2 M,
(A*) evaporate 50 mL water (B*) evaporate 50 mL solution
(C) add 0.1 mol KCl (D*) add 0.01 mol KCl
gekjsikl 0.1 M KCl foy ;u d k 100 mL gSA bls0.2 M cukusd sfy , &
(A) 50 mL t y okf"ir fd ;k t krk gS (B) 50 mL foy ;u okf"ir fd ;k t krk gS
(C) 0.1 eks
y KCl d ksfey k;k t krk gS (D*) 0.01 eks
y KCl d ksfey k;k t krk gS
2. Which of the following concentration factors can be calculated if the mole fraction and density of an aqueous
solution of HCl are known ?
(A*) Molality (B*) Molarity (C*) Percent by mass (D*) Normality
fuEu esalsd kSulk@d kSulslkUnzrk inksad h x.kuk d h t k ld rh gSA ;fn t y h; HCl foy ;u d sfy , eksy fHkUu rFkk
?kuRo Kkr gks\
(A*) eks
y y rk (B*) eks
y jrk (C*) iz
fr'kr ¼Hkkj ls½ (D*) ukW
eZy rk

Corporate Office: CG Tower, A-46 & 52, IPIA, Near City Mall, Jhalawar Road, Kota (Raj.)-324005
Website : www.resonance.ac.in | E-mail : contact@resonance.ac.in
Toll Free : 1800 200 2244 | 1800 258 5555 | CIN: U80302RJ2007PLC024029
ADV-SCP - 30
Solution & Colligative Properties
Section (C) : Vapour Pressure
[k.M (C) : ok"i nkc
3. The vapour pressure of a dilute solution of a solute is influenced by :
(A*) Temperature of solution (B*) Mole fraction of solute
(C) M.pt. of solute (D*) Degree of dissociation of solute
,d foy s; d sruqfoy ;u d k ok"inkc fuEu }kjk izHkkfor gksrk gSA
(A*) foy ;u d k rkieku (B*) foy s
; d k eksy izHkkt
(C) foy s
; d k xy ukad (D*) foy s
; d sfo;kst u d h ek=kk
Section (D) : Solutions of Solid and Gases in Liquids
[k.M (D) : nzo esaBksl rFkk xSl ksad k foy ;u
4. According to Henry’s law, the partial pressure of gas (Pg) is directly proportional to mole fraction of gas in
dissolved state , i.e., Pgas = KH . Xgas where KH is Henry’s constant. Which are correct ?
(A*) KH is characteristic constant for a given gas–solvent system
(B*) Higher is the value of KH , lower is solubility of gas for a given partial pressure of gas
(C*) KH has temperature dependence
(D*) KH increases with temperature
gsujh fu;e d svuql kj xSl d k vkaf'kd nkc (Pg), foy s; voLFkk esamifLFkr xSl d seksy izHkkt d slekuqikrh gksrk gS
vFkkZr~ PxSl  = KH . XxSl t gk¡KH gSujh fu;rkad gSA d kSulsd Fku lgh gSa?
(A*) KH , fn;sx;sxS l &foy k;d ra=k d sfy , vfHky k{kf.kd fu;rkad gSA
(B*) xSl d sfn;sx;svkaf'kd nkc d sfy , KH d k mPp eku] xSl d h d e foy s;rk d ksn'kkZrk gSA
(C*) KH rki ij fuHkZ j d jrk gSA
(D*) rki es aof̀) d slkFk KH esaof̀) gksrh gSA
5. Select correct statements :
(A*) Gases which have high value of van der Waals constant ‘a’ are easily liquefied
(B) Easily liquefiable gases are water soluble
(C*) Gases which forms ions in a solvent are soluble in that solvent
(D) Under same conditions, NH3 has low solubility in water than that of CO2 .
lgh d Fkuksad k p;u d hft , %
(A*) xS
l sft ud sfy , ok.Mjoky fu;rkad (a) d k mPp eku gS] vklkuh lsnzohd r̀ gksrh gSA
(B) vklkuh lsnz ohd r̀ xSl ] t y esa?kqy u'khy gksrh gSA
(C*) ,d foy k;d es axSl sat ksvk;u fuekZ.k d jrh gSog foy k;d esafoy s;'khy gksrh gSA
(D) leku ifjfLFkfr es aNH3, CO2 lst y esad e ?kqy u'khy gSA
Section (E) : Immiscible Liquids
[k.M (E) : v feJ.kh; nzo
6. Two liquids X and Y are perfectly immiscible. If X and Y have molecular masses in ratio 1 : 2, the total
vapour pressure of a mixture of X and Y prepared in weight ratio 2 : 3 should be (Px0 = 400 torr, Py0 = 200
torr)
nksnzo X o Y lEiw.kZrjg lsvfeJ.kh; gS;fn X o Y d sfy , vkf.od nzO;eku d k vuqikr 1 : 2 gksrksHkkj vuqikr
2 : 3 es
acuk;sx;sX o Y d s,d feJ.k d k d qy ok"i nkc Kkr d hft ,A (Px0 = 400 torr, Py0 = 200 torr)
(A*) 600 torr (B) 400 torr (C) 800 torr (D) 1000 torr
Ans. 600 torr

Corporate Office: CG Tower, A-46 & 52, IPIA, Near City Mall, Jhalawar Road, Kota (Raj.)-324005
Website : www.resonance.ac.in | E-mail : contact@resonance.ac.in
Toll Free : 1800 200 2244 | 1800 258 5555 | CIN: U80302RJ2007PLC024029
ADV-SCP - 31
Solution & Colligative Properties
Section (F) : Completely miscible liquids : Raoult’s law
[k.M (F) : iw.kZr% feJ.kh; nzo % jkWÅ YV fu;e
7. Which of the following is/are correct for an ideal binary solution of two volatile liquids (eg. benzene &
toluene) ?
(A*) Its vapor is always richer in the more volatile component (compared to the liquid).
(B*) The liquid will gradually become richer in the less volatile component if such a mixture is boiled
(distilled).
(C) The PT (ie. the total pressure) above the solution will be the sum of the vapor pressures of the two pure
components.
(D) The boiling point of the solution will be less than the boiling points of the two components.
nksok"i'khy nzokas(mnk- csat hu rFkk VkWy qbZu) d svkn'kZf}vaxh foy ;u d sfy ;sfuEu esalsd kSulk@d kSulsd Fku lR;
gS@ gSa\
(A*) bld h ok"i vf/kd ok"i'khy vo;o ¼nz o d h rqy uk esa½ esages'kk T;knk gksxhA
(B*) d e ok"i'khy vo;o es anzo lkekU;r% vf/kd gksxk ;fn bl rjg d sfeJ.k d ksmcky k ¼vklfor½ t k;saA
(C) mijksDr foy ;u d k PT (d qy nkc), nks'kq) vo;o d sok"i nkcksad k ;ksx gksxkA
(D) foy ;u d k DoFkukad ] nksuksavo;oksad sDoFkukad lsd e gksxkA
8. Which is/are true about ideal solutions ?
(A*) The volume change on mixing is zero (B*) The enthalpy of mixing is zero
(C) The entropy of mixing is zero (D) The enthalpy of mixing is negative
vkn'kZfoy ;u d sckjsesad kSulk@d kSulsd Fku lgh gS@ gSa\
(A*) feJ.k d k vk;ru es aifjorZu 'kwU; gSA (B*) feJ.k d h ,UFkS
Yih 'kwU; gSA
(C) feJ.k d h ,UVª
kWih 'kwU; gSA (D) feJ.k d h ,UFkS
Yih _ .kkRed gSA
9. At 40ºC, vapour pressure in Torr of methanol and ethanol solution is P = 119x + 135 where x is the mole
fraction of methanol. Hence
(A) vapour pressure of pure methanol is 119 Torr
(B*) vapour pressure of pure ethanol is 135 Torr
(C) vapour pressure of equimolar mixture of each is 127 Torr
(D) mixture is completely immiscible
40ºC rki ij es Fksuksy rFkk ,Fksuksy foy ;u d k ok"inkc Vksj esaP = 119x + 135 gSA t gk¡x esFksuksy d k eksy izHkkt gSA
vr% &
(A) 'kq
) esFksuksy d k ok"inkc 119 Vksj gSA (B*) 'kq
),Fksuksy d k ok"inkc 135 Vksj gSA
(C) izR;sd d sleeksy j feJ.k d k ok"inkc 127 Vksj gSA (D) feJ.k iw
.kZr;k vfeJ.kh; gSA
Section (G) : Non-ideal Solutions
[k.M (G) : v ukn'kZfoy ;u
10. The diagram given below represents boiling point composition
diagram of solution of components A and B, which is/are incorrect
among the following?
(A*) The solution shows negative deviation
(B*) A-B-interactions are stronger than A-A and B-B
(C*) The solution is ideal solution
(D) The solution shows positive deviation.
uhpsfn;k x;k js[kkfp=k vo;o A rFkk B d sfoy ;u d sDoFkukad la?kBu
d sjs[kkfp=k d ksiznf'kZr d jrk gSa] buesalsxy r d Fku gSa\
(A*) foy ;u _ .kkRed fopy u n'kkZ rk gS
(B*) A-B-vU;ks U; fØ ;k A-A ,oaB-B lsvf/kd izcy gSa
(C*) foy ;u ,d vkn'kZfoy ;u gS
(D) foy ;u /kukRed fopy u n'kkZ rk gSA
Corporate Office: CG Tower, A-46 & 52, IPIA, Near City Mall, Jhalawar Road, Kota (Raj.)-324005
Website : www.resonance.ac.in | E-mail : contact@resonance.ac.in
Toll Free : 1800 200 2244 | 1800 258 5555 | CIN: U80302RJ2007PLC024029
ADV-SCP - 32
Solution & Colligative Properties
11. Which is/are correct statement(s) ?
(A*) When mixture is more volatile than both pure components, there is positive deviation from Raoult’s law
(B*) When mixture is less volatile than both pure components, there is negative deviation from Raoult’s law
(C) Ethanol and water form ideal solution
(D) CHCl3 and water form ideal solution
d kSulk@d kSulsd Fku lgh gS@ gSa\
(A*) t c feJ.k nksuksa'kq) vo;oksalsvf/kd ok"i'khy gksrk gS] rc ;g jkWÅ YV fu;e ls/kukRed fopy u n'kkZrk gSA
(B*) t c feJ.k nksuksa'kq) vo;oksalsd e ok"i'khy gksrk gSrc ;g jkWÅ YV fu;e ls_ .kkRed fopy u n'kkZrk gSA
(C) ,Fks
uksy rFkk t y vkn'kZfoy ;u cukrsgSA
(D) CHCl3 rFkk t y vkn'kZfoy ;u cukrsgS A
12. At 35°C, the vapour pressure of CS2 is 512 mmHg, and of acetone is 344 mmHg. A solution of CS2 and
acetone in which the mole fraction of CS2 is 0.25, has a total vapour pressure of 600mmHg. Which of the
following statements is/are correct ?
(A) A mixture of 100 mL of acetone and 100 mL of CS2 has a volume of 200 mL
(B*) When acetone and CS2 are mixed at 35°C, heat must be absorbed in order to produce a solution at
35°C.
(C) Process of mixing is exothermic
(D) Entropy of mixing is zero
35°C ij CS2 d k ok"i nkc 512 mmHg rFkk ,lhVks u d k 344 mmHg gSA CS2 rFkk ,lhVksu d sfoy ;u esaCS2 d k
eksy izHkkt 0.25 gSrFkk d qy ok"i nkc 600mmHg gSA fuEu esalsd kSulk@d kSulsd Fku lgh gS\
(A) ,lhVks u d s100 mL rFkk CS2 d s100 ml d k feJ.k 200 mL vk;ru j[krk gSA
(B*) t c 35°C ij ,lhVks u rFkk CS2 d ksfefJr d jrsgaSrc 35°C ij foy ;u mRiUu d jusd sØ e esaÅ "ek vo'kksf"kr
gksuh pkfg,A
(C) feJ.k d h fØ ;k m"ek{ks ih gksrh gSA
(D) feJ.k d h ,UVªksih 'kwU; gksrh gSA
Section (H) : Degree of Ionisation/Dissociation for Weak Electrolytes
[k.M (H) : nqcZy oS| qr v i?kV~;ksad sv k;uu@fo;kst u d h ek=kk
13. For the given electrolyte AxBy, the degree of dissociation '' can be given as
i 1 1 i
(A*)  = x  y  1 (B*) i = ( 1– )+ x + y (C*)  = 1  x  y (D) None

fn;sx;sfo|qr vi?kV~; AxBy d sfy ;sfo;kst u d h ek=kk '' nh t k ld rh gS%


i 1 1 i
(A*)  = x  y  1 (B*) i = ( 1– )+ x + y (C*)  = 1  x  y (D) d ks
bZugha

14. In which of the following pairs of solutions will the values of the vant Hoff factor be the same?
(A) 0.05 M K4 [Fe(CN)6] and 0.10 M FeSO4
(B*) 0.10 M K4[Fe(CN)6] and 0.05 M FeSO4 (NH4)2SO4. 6H2O
(C) 0.20 M NaCl and 0.10 M BaCl2
(D*) 0.05 M FeSO4 (NH4)2SO4 . 6H2O and 0.02 M KCl . MgCl2 . 6H2O
okUVgkWQ xq.kkad d k eku fuEu esalsfd u foy ;uksad s;qXe d sfy , leku gksxk ?
(A) 0.05 M K4 [Fe(CN)6] rFkk0.10 M FeSO4
(B*) 0.10 M K4[Fe(CN)6] rFkk0.05 M FeSO4 (NH4)2SO4. 6H2O
(C) 0.20 M NaCl rFkk0.10 M BaCl2
(D*) 0.05 M FeSO4 (NH4)2SO4 . 6H2O rFkk0.02 M KCl . MgCl2 . 6H2O

Corporate Office: CG Tower, A-46 & 52, IPIA, Near City Mall, Jhalawar Road, Kota (Raj.)-324005
Website : www.resonance.ac.in | E-mail : contact@resonance.ac.in
Toll Free : 1800 200 2244 | 1800 258 5555 | CIN: U80302RJ2007PLC024029
ADV-SCP - 33
Solution & Colligative Properties
Section (I) : Relative lowering of vapour pressure
[k.M (I) : ok"i nkc d k v kisf{kd v oueu
15. 2 g of non-volatile hydrocarbon solute dissolved in 100 g of hypothetical organic solvent (molar mass = 50)
was found to lower vapour pressure from 75.50 to 75 mm of Hg at 20°C. Given that hydrocarbon contains
96% of C. Then which of the following are true ?
(A*) molecular wt of solute = 150 (B*) molecular formula = C12H6
(C) molecular wt. of solute = 132 (D) none of these
,d 100 g d kYifud d kcZfud foy k;d (eksy j nzO;eku = 50) esafo;ksft r 2 g vok"i'khy gkbMªksd kcZu foy s; d k ok"i
nkc 20ºC ij 75 ls75.50 mm Hg rd d e ik;k t krk gSA fn;k gSfd gkbMªksd kcZu esa96% d kcZu gSrc fuEu esals
d kSulk@d kSulsd Fku lR; gS\
(A*) foy s
; d k v.kqHkkj = 150 (B*) v.kq
l w=k = C12H6
(C) foy s
; d k v.kqHkkj = 132 (D) buesalsd ksbZugha
Section (J) : Elevation of Boiling Point & Depression of Freezing Point
[k.M (J) : DoFkukad esamUu;u o fgekad esav oueu
16. In the depression of freezing point experiment , it is found that the :
(A*) Vapour pressure of the solution is less than that of pure solvent
(B) Vapour pressure of the solution is more than that of pure solvent
(C) Only solute molecules solidify at the freezing point
(D*) Only solvent molecules solidify at the freezing point
fgekad d svoueu d siz;ksx esa] ;g ik;k x;k fd %
(A*) foy ;u d k ok"i nkc] 'kq
) foy k;d d h rqy uk esad e gksrk gSA
(B) foy ;u d k ok"i nkc 'kq
) foy k;d d h rqy uk esavf/kd gksrk gSaA
(C) fgeka
d ij d soy foy s; v.kqBksl voLFkk esaizkIr gksrsgSaA
(D*) fgeka
d ij d soy foy k;d v.kqBksl voLFkk esaizkIr gksrsgSaA
17. For a dilute solution having molality m of a given solute in a solvent of mol.wt. M , b.pt. Tb and heat of
  Tb 
vaporisation per mole H ;   m  is equal to :
 m0
(A*) Molal elevation constant of solvent
RTb2 M
(B*)  H ; where M in kg vap H and R in J mol–1
vap

RTb2 M
(C)  S ; where M in kg ; vapS and R in J mol–1
vap

RTb2 M
(D*) 1000  H ; where M in g ; R and vapH expressed in same unit of heat.
vap

M v.kq
Hkkj d sfoy k;d esafn;sx;sfoy s; d h m eksy y rk oky sruqfoy ;u d sfy , DoFkukad Tb gSrFkk ok"iu d h Å "ek
  Tb 
izfr eksy H ;   fuEu d scjkcj gS&
  m m0
(A*) foy k;d d seksy y mUu;u fLFkjkad d s
RTb2 M
(B*)  H ; t gk¡M, kg es
a; vap H rFkkR t wy izfr eksy esa
vap

RTb2 M
(C)  S ; t gk¡M, kg es
a; vapS rFkkR t wy izfr eksy esa
vap

RTb2 M
(D*) 1000  H ; t gk¡M, g es
a; R o vapH d ksÅ "ek d h leku bd kbZesaO;Dr fd ;k t krk gSA
vap

Corporate Office: CG Tower, A-46 & 52, IPIA, Near City Mall, Jhalawar Road, Kota (Raj.)-324005
Website : www.resonance.ac.in | E-mail : contact@resonance.ac.in
Toll Free : 1800 200 2244 | 1800 258 5555 | CIN: U80302RJ2007PLC024029
ADV-SCP - 34
Solution & Colligative Properties
Section (K) : Osmotic Pressure
[k.M (K) : ijklj.k nkc
18. Consider following solutions :
I : 1 M aqueous glucose solution II : 1M aqueous sodium chloride solution
III : 1M aqueous ammonium phosphate solution IV : 1M benzoic acid in benzene
Select correct statements for the above solutions :
(A) All are isotonic solutions (B*) III is hypertonic of I, II and IV
(C*) IV is hypotonic of I, II and III (D*) II is hypotonic of III but hypertonic of I and IV
fuEu foy ;uksaij fopkj d hft , %
I : Xy w
d kst d k 1 M t y h; foy ;u II : lks
fM;e Dy ksjkbM d k 1M t y h; foy ;u
III : veksfu;e Q kWLQ sV d k 1M t y h; foy ;u IV : csUt hu esa1M csUt kWbd vEy
mDr foy ;uksad sfy , lgh d Fkuksad k pquko d hft ,A
(A) lHkh leijkljh foy ;u gS A
(B*) III foy ;u]I, II rFkk IV d h rqy uk esavfrijkljh gSA
(C*) IV foy ;u] I, II rFkk III d h rq
y uk esavYiijkljh gSA
(D*) II foy ;u] III d h rqy uk esavYi ijkljh gS] y sfd u I rFkk IV d h rqy uk esavfrijkljh gSA
19. Which facts are true when we use van’t Hoff equation PV = nST for osmotic pressure P of dilute solutions?
(A*) The equation is identical to that of ideal gas equation
(B*) The solute particles in solution are analogous to the gas molecules and the solvent is analagous to the
empty space between the gas molecules
(C*) Solute molecules are dispersed in the solvent the way the gas molecules are dispersed in empty
space
(D) The equation is not identical to that of ideal gas equation
t c ge ruqfoy ;u d sijklj.knkc P d sfy , ok.V gkWQ lehd j.kPV = nST d kmi;ksx d jrsgSrc d kSulsrF; lR;
gS\
(A*) lehd j.k] vkn'kZxS l lehd j.k d sleku gSA
(B*) foy ;u esafoy s; d sd .k] xSl v.kqv ksad sleku rFkk foy k;d ] xSl v.kqv ksad se/; fjDr LFkku d sleku gSA
(C*) foy s
; d sd .k foy k;d esamlh izd kj forfjr jgrsgS] ft l izd kj xSl d sv.kqfjDr LFkkuksaesaforfjr jgrsgSA
(D) lehd j.k] vkn'kZxS l lehd j.k d sleku ughagSA

PART - IV : COMPREHENSIONS
Hkkx - IV : v uqPN sn (COMPREHENSIONS)
Read the following passage carefully and answer the questions.
fuEu v uqPN sn d ks/;kuiwoZd if<+;srFkk iz'uksad smÙkj nhft ,A
Comprehension # 1
PA = x A PA & PB = xBPB
PT = x A PA + xBPB (Note : PA > PB ; A is more volatile than B)

Vapour pressure of mixtures of Benzene (C6H6 ) & toluene (C7H8) at 50ºC are given by PM = 179 XB + 92,
where XB is mole fraction of C6H6.

Corporate Office: CG Tower, A-46 & 52, IPIA, Near City Mall, Jhalawar Road, Kota (Raj.)-324005
Website : www.resonance.ac.in | E-mail : contact@resonance.ac.in
Toll Free : 1800 200 2244 | 1800 258 5555 | CIN: U80302RJ2007PLC024029
ADV-SCP - 35
Solution & Colligative Properties
v uqPN sn # 1
PA = x A PA & PB = xBPB

PT = x A PA + xBPB (uks


V : PA > PB ; A, B lsvf/kd ok"i'khy gSA)

50ºC ij csUt hu (C6H6 ) rFkk VkWy qbZu (C7H8) d sfeJ.k d k ok"i nkc PM = 179 XB + 92 }kjk n'kkZ;k x;k gSA t gk¡XB,
C6H6 d k eks
y izHkkt gSA
1. What is vapour pressure of pure liquids ?
(A) PB = 92 mm, PT = 179 mm (B*) PB = 271 mm, PT = 92 mm
(C) PB = 180 mm, PT = 91 mm (D) none of these
'kq) nzoksad k ok"i nkc D;k gS\
(A) PB = 92 mm, PT = 179 mm (B*) PB = 179 mm, PT = 92 mm
(C) PB = 180 mm, PT = 91 mm (D) bues
alsd ksbZugha
2. Vapour pressure of liquid mixture obtained by mixing 936 g C6H6 & 736 g tolene is :
936 g C6H6 rFkk 736 g VkWy qbZu d ksfey kuslsizkIr nzo feJ.k d k ok"i nkc gS&
(A) 300 mm Hg (B) 250 mm Hg (C*) 199.4 mm Hg (D) 180.6 mm Hg

Comprehension # 2
Answer the questions (given below) which are based on the
following diagram.
Consider some facts about the above phase diagram :
Vapour pressure diagram for real solutions of two liquids A and B
that exhibit a positive deviation from Raoult’s law. The vapour
pressure of both A and B are greater than predicted by Raoult’s
law. The dashed lines represented the plots for ideal solutions.

v uqPN sn # 2
fuEu vkjs[k ij vk/kkfjr iz'uksa¼uhpsfn;sx;s½ d smÙkj nhft ,A
mDr voLFkk vkjs[k d slanHkZesad qN rF;ksaij fopkj d hft ,A
A rFkk B nksnz oksad sokLrfod foy ;u d sfy , ok"inkc vkjs[k fn;k
gS] t ksfd jkWÅ YV fu;e ls/kukRed fopy u n'kkZrk gSA A rFkk B
nksuksad k ok"inkc]jkWÅ YV fu;e d svuql kj nkc d h rqy uk esavf/kd
gSA vkjs[k esafcUnqor~js[kk;sa]vkn'kZfoy ;u d svkjs[k d ksn'kkZrh gSA

Corporate Office: CG Tower, A-46 & 52, IPIA, Near City Mall, Jhalawar Road, Kota (Raj.)-324005
Website : www.resonance.ac.in | E-mail : contact@resonance.ac.in
Toll Free : 1800 200 2244 | 1800 258 5555 | CIN: U80302RJ2007PLC024029
ADV-SCP - 36
Solution & Colligative Properties
3. A : This is observed when A...B attractions are greater than average of A...A and B...B attraction:
B : Hmix = +ve, Vmix = +ve
C : Boiling point is smaller than expected such that vaporisation is increased
D : Mixture can form azeotropic mixture
Select correct facts
(A) A, B, C (B*) B, C, D (C) A, C, D (D) A, B, C, D
A : t c A....B d k vkd "kZ
.k] A....A rFkk B....B vkd "kZ.k d svkSl r lsvf/kd gksrk gSA rc ;g izsf{kr fd ;k t krk gSA
B : HfeJ.k = +ve, VfeJ.k = +ve
C : feJ.k d k DoFkuka
d vuqeku lsd e vkrk gSbl izd kj ok"ihd j.k esaof̀) gksrh gSA
D : feJ.k] fLFkj Doka
Fkh feJ.k cuk ld rk gSA
lgh rF;ksad k pquko d hft ,A
(A) A, B, C (B*) B, C, D (C) A, C, D (D) A, B, C, D

4. Total vapour pressure of mixture of 1 mol of volatile component A (p°A = 100 mm Hg) and 3 mol of volatile
component B (p°B = 60 mm Hg) is 75 mm. For such case :
(A) There is positive deviation from Raoult’s law
(B) Boiling point has been lowered
(C) Force of attraction between A and B is smaller than that between A and A or between B and B.
(D*) All the above statements are correct.
1 eks
y ok"i'khy ?kVd A (p°A = 100 mm Hg) rFkk 3 eksy ok"i'khy ?kVd B (p°B = 60 mm Hg) d sfeJ.k d kd qy ok"i
nkc 75 mm gSA bl izd kj d h ifjfLFkfr;ksad sfy , &
(A) ;gk¡jkW
Å YV fu;e ls/kukRed fopy u gksrk gSA
(B) DoFkuka
d esad eh vkrh gSA
(C) A rFkk B d se/; vkd "kZ
.k cy ] A rFkk A vFkok B rFkk B d se/; vkd "kZ.k cy d h rqy uk esad e gksrk gSA
(D*) mDr lHkh d Fku lgh ga SA
Comprehension # 3
Addition of non–volatile solute to a solvent always increases the colligative properties such as osmotic
pressure, P, Tb and Tf . All these colligative properties are directly proportional to molality if solutions
are dilute. The increases in colligative properties on addition of non–volatile solute is due to increase in
number of solute particles.
v uqPN sn # 3
,d foy k;d esa]vok"i'khy foy s; inkFkZfey kusij lnSo v.kql a[;d xq.kksat Sl sfd ijklj.k nkc] P, Tb rFkkTf esa
of̀) gksrh gSA ;fn foy ;u ruqgksrc ;g lHkh v.kql a[;d xq.k] eksy y rk d slekuqikrh gksrsgSA vok"i'khy foy s; d ks
fey kusij v.kql a[;d xq.kksaesaof̀) foy s; d .kksad h la[;k esaof̀) d sd kj.k gksrh gSA
5. For different aqueous solutions of 0.1 M NaCI, 0.1 M urea , 0.1 M Na2SO4 and 0.1 M Na3PO4 solution at 270
C, the correct statements are : (Consider the solutions to be dilute)
1. The order of osmotic pressure is, NaCl > Na2SO4 > Na3PO4 > urea
Tb
2.  = K × ST for urea solution
b
3. Addition of salt on ice increases its melting point
4. Addition of salt on ice brings in melting of ice earlier
27°C rki ij fofHkUu t y h; foy ;uks a0.1 M NaCI, 0.1 M ;wfj;k, 0.1 M Na2SO4 rFkk0.1 M Na3PO4 d sfy , lgh
d Fku gaS& ¼foy ;u d ksruqekusa½
1. ijklj.k nkc d k Ø e gSNaCl > Na2SO4 > Na3PO4 > ;w
fj;k
Tb
2.  = K × ST ;w
fj;k foy ;u d sfy ,
b

3. cQ Zij y o.k fey kusij bld sxy ukad esaof̀) gksrh gSA
4. cQ Zij y o.k d ksfey kusij cQ Zd kst Ynh fi?ky k ns
rk gS¼iwoZesay h xbZcQ Zd h rqy uk esa½
(A) 2 , 3 ,4 (B*) 2, 4 (C) 1 , 2 , 3 (D) 3 , 4
Corporate Office: CG Tower, A-46 & 52, IPIA, Near City Mall, Jhalawar Road, Kota (Raj.)-324005
Website : www.resonance.ac.in | E-mail : contact@resonance.ac.in
Toll Free : 1800 200 2244 | 1800 258 5555 | CIN: U80302RJ2007PLC024029
ADV-SCP - 37
Solution & Colligative Properties
6. 1 g mixture of glucose and urea present in 250 mL aqueous solution shows the osmotic pressure of 0.74
atm at 270 C. Assuming solution to be dilute , which are correct ?
1. Percentage of urea in mixture is 17.6.
2. Relative lowering in vapour pressure of this solution is 5.41 × 10–4.
3. The solution will boil at 100.015ºC , if Kb of water is 0.5 K molality–1.
4. If glucose is replaced by same amount of sucrose, the solution will show higher osmotic pressure at
27ºC.
5. If glucose is replaced by same amount of NaCl, the solution will show lower osmotic pressure at 270C.
250 mL t y h; foy ;u es aXy wd kst rFkk ;wfj;k d k 1g feJ.k mifLFkr gS]t ksfd 27ºC rki ij ijklj.k nkc 0.74 atm
n'kkZrk gSA ekuk fd foy ;u ruqgSA d kSulsd Fku lgh gS\
1. feJ.k es a;wfj;k d k izfr'kr 17.6 gSA
2. bl foy ;u d sfy , ok"i nkc es avkisf{kd d eh 5.41 × 10–4 gSA
3. foy ;u 100.015ºC, rki ij mcy rk gS;fn t y d k Kb 0.5 K eks y y rk–1 gSA
4. ;fn Xy wd kst d ksleku ek=kk esalwØ ksl lsizfrLFkkfir d j fn;k t k, rc 27ºC rki ij foy ;u mPp ijklj.k nkc
n'kkZ;sxkA
5. ;fn Xy wd kst d ksleku ek=kk esaNaCl lsizfrLFkkfir fd ;k t k, rc 27ºC ij foy ;u d e ijklj.k nkc n'kkZ;sxkA
(A*) 1 , 2 , 3 (B) 1 , 2 , 3 , 5 (C) 2 , 4 , 5 (D) 1 , 4 , 5

Corporate Office: CG Tower, A-46 & 52, IPIA, Near City Mall, Jhalawar Road, Kota (Raj.)-324005
Website : www.resonance.ac.in | E-mail : contact@resonance.ac.in
Toll Free : 1800 200 2244 | 1800 258 5555 | CIN: U80302RJ2007PLC024029
ADV-SCP - 38
Solution & Colligative Properties

* Marked Questions may have more than one correct option.


* fpfUgr iz
'u ,d l sv f/kd l gh fod Yi oky siz'u gS-

PART - I : JEE (ADVANCED) / IIT-JEE PROBLEMS (PREVIOUS YEARS)


Hkkx - I : JEE (ADVANCED) / IIT-JEE ¼fiN y so"kksZ½ d siz'u
1. In the depression of freezing point experiment, it is found that : [JEE 1999, 3/80]
I. The vapour pressure of the solution is less than that of pure solvent.
II. The vapour pressure of the solution is more than that of pure solvent.
III. Only solute molecules solidify at the freezing point.
IV. Only solvent molecules solidify at the freezing point.
fgekad d svoueu iz;ksx esa] ;g ik;k x;k fd % [JEE 1999, 3/80]
I. foy ;u d k ok"i nkc 'kq
) foy k;d lsd e gSA
II. foy ;u d k ok"i nkc 'kq
) foy k;d lsvf/kd gSA
III. d s
oy foy s; d sv.kqgh fgekad ij Bksl esaifjofrZr gksrsgSA
IV. d soy foy k;d d sv.kqgh fgekad ij Bksl esaifjofrZr gksrsgSA
(A) I, II (B) II, III (C*) I, IV (D) I, II, III.
2. The van’t Hoff factor for 0.1 M Ba(NO3)2 solution is 2.74. The degree of dissociation is :[JEE 1999, 3/80]
0.1 M Ba(NO3)2 foy ;u d sfy , okUVgkW
Q xq.kkad 2.74 gSA fo;kst u d h ek=kk gS% [JEE 1999, 3/80]
(A) 91.3% (B*) 87% (C) 100% (D) 74%
3. To 500 cm3 of water, 3.0 × 10–3 kg of acetic acid is added. If 23% of acetic acid is dissociated, what will be
the depression in freezing point? kf and density of water are 1.86 K kgmole–1 and 0.997 g cm–3 respectively
[JEE 2000, 3/35]
500 cm3 t y esa, 3.0 × 10–3 kg ,flfVd vEy Mky k t krk gSA ;fn 23% ,flfVd vEy fo;ksft r gksrk gS]rksfgekad esa
fd ruk voueu gksxk\ kf o t y d k ?kuRo Ø e'k% 1.86 K kg mole–1 o 0.997 g cm–3 gaS: [JEE 2000, 3/35]
(A) 0.186 K (B*) 0.228 K (C) 0.372 K (D) 0.556 K
4. During depression of freezing point in a solution, the following are in equilibrium : [JEE 2003, 3/84]
(A*) Liquid solvent-solid solvent (B) Liquid solvent-solid solute
(B) Liquid solute-solid solute (D) Liquid solute-solid solvent
foy ;u esafgekad d svoueu d snkSjku] fuEu lkE;koLFkk esagksrsgSa% [JEE 2003, 3/84]
(A*) nz
o foy k;d &Bksl foy k;d (B) nzo foy k;d &Bksl foy s;
(B) nz
o foy s;&Bksl foy s; (D) nzo foy s;&Bksl foy k;d
5. A 0.004 M solution of Na2SO4 is isotonic with 0.010 M solution of glucose at same temperature. The
apparent percentage dissociation of Na2SO4 is : [JEE 2004, 3/84]
leku rki ij Na2SO4 d k 0.004 M foy ;u Xy wd kst d s0.010 M foy ;u d slkFk leijkljh gSA Na2SO4 d k y xHkx
izfr'kr fo;kst u fuEu gS% [JEE 2004, 3/84]
(A) 25% (B) 50% (C*) 75% (D) 85%
6. 1.22 g of benzoic acid is dissolved in 100 g of acetone and 100 g of benzene separately. Boiling point of the
solution in acetone increases by 0.17ºC, while that of, in the benzene increases by 0.13ºC ; Kb for acetone
and benzene is 1.7 K kg mol–1 and 2.6 K kg mol–1. Find molecular weight of benzoic acid in two cases and
justify your answer. [JEE 2004, 4/60]
1.22 g cs at ksbd vEy d ksiF̀kd : i ls100 g ,lhVksu rFkk 100 g csat hu esafey k;k t krk gSA ,lhVksu esafoy ;u d k
DoFkukad 0.17ºC c<+t krkgS ]at cfd csta hu es
a0.13ºC rd DoFkukda c<+ rkgSA ,lhVkus rFkkcs
ta hu d sfy, Kb Ø e'k%1.7 K
kg mol rFkk2.6 K kg mol gS
–1 –1
A nksuksaifjfLFkfr;ksaesacsat kWbd vEy d kvkf.od Hkkj Kkr d jrsgq,]viusmÙkj d ksLi"V
d hft ,A [JEE 2004, 4/60]
Ans. 122, 224

Corporate Office: CG Tower, A-46 & 52, IPIA, Near City Mall, Jhalawar Road, Kota (Raj.)-324005
Website : www.resonance.ac.in | E-mail : contact@resonance.ac.in
Toll Free : 1800 200 2244 | 1800 258 5555 | CIN: U80302RJ2007PLC024029
ADV-SCP - 39
Solution & Colligative Properties
7. The elevation in boiling point of a solution of 13.44 g of CuCl2 in 1kg of water using the following information,
will be (Molecular weight of CuCl2 = 134.4 and Kb = 0.52 K molal–1) : [JEE 2005, 3/84]
1 kg t y esa13.44 g CuCl2 d sfoy ;u d sDoFkukad esamUu;u fuEu lwp uk d k mi;ksx d jrsgq,] gksxk(CuCl2 d k
v.kqHkkj = 134.4 o Kb = 0.52 K molal–1) : [JEE 2005, 3/84]
(A*) 0.16 (B) 0.05 (C) 0.1 (D) 0.2

8. When 20 g of naphthoic acid (C11H8O2) is dissolved in 50 g of benzene (Kf = 1.72 K kg mol–1), a freezing
point depression of 2 K is observed. The van’t Hoff factor (i) is : [JEE-2007, 3/162]
t c 20 g us¶FkkWbd vEy (C11H8O2) d ks50 g csat hu eas?kksy k t krk gS(Kf = 1.72 K kg mol ), rc fgekad esavoueu
–1

2 K ik;k t krk gS
A okUVgkWQ xq.kkad (i) gksxk % [JEE-2007, 3/162]
(A*) 0.5 (B) 1 (C) 2 (D) 3

Comprehension #
Properties such as boiling point, freezing point and vapour pressure of a pure solvent change when solute
molecules are added to get homogeneous solution. These are called colligative properties. Application of
colligative properties are very useful in day-to-day life. One of its example is the use of ethylene glycol and
water mixture as anti-freezing liquid in the radiator of automobiles
A solution M is prepared by mixing ethanol and water. The mole fraction of ethanol in the mixture is 0.9
Given : Freezing point depression constant of water (Kfwater) = 1.86 K kg mol–1
Freezing point depression constant of ethanol (Kfethanol) = 2.0 K kg mol–1
Boiling point elevation constant of water (Kbwater) = 0.52 K kg mol–1
Boiling point elevation constant of ethanol (Kbethanol) = 1.2 K kg mol–1
Standard freezing point of water = 273 K
Standard freezing point of ethanol = 155.7 K
Standard boiling point of water = 373 K
Standard boiling point of ethanol = 351.5 K
Vapour pressure of pure water = 32.8 mm Hg
Vapour pressure of pure ethanol = 40 mm Hg
Molecular weight of water = 18 g mol–1
Molecular weight of ethanol = 46 g mol–1
In answering the following questions, consider the solution to be ideal dilute solutions and solutes to be
non-volatile and non-dissociative.
v uqPN sn #
'kq) foy k;d d sDoFkukad ]fgekad rFkkok"i nkc t Sl sxq.k/keZifjofrZr gkst krsgS]t c foy s;hv.kqv ksad ksfey kd j lekaxh
foy ;u cuk;k t krk gSA bud ksv.kql a[;d xq.k/keZ(colligative properties) d grsgSaA v.kql a[;d xq.k/keZd k vuqiz;ksx
lkoZt fud t hou esacgqr mi;ksxh gSA bld k ,d mnkgj.k ,Fkhy hu Xy kbd kWy rFkk t y d sfeJ.k d k iz;ksx izfr fgeu
nzo (anti-freezing liquid) d s: i esavkWVkseksckby ksad sjsfM;sVj esafd ;k t krk gSA
,FksukWy rFkk t y d k ,d feJ.k M cuk;k x;k gSA feJ.k esa,FksukWy d k eksy fHkUu 0.9 gSA
iznÙk: t y d k fgekad voueu fLFkjkad (Kft y ) = 1.86 K kg mol–1
,FksukWy d k fgekad voueu fLFkjkad (Kf,FksukWy ) = 2.0 K kg mol–1
t y d k DoFkukad mUu;u fLFkjkad (Kbt y ) = 0.52 K kg mol–1
,FksukWy d k DoFkukad mUu;u fLFkjkad (Kb,FksukWy ) = 1.2 K kg mol–1
t y d k ekud fgekad = 273 K
,FksukWy d k ekud fgekad = 155.7 K
t y d k ekud DoFkukad = 373 K
,FksukWy d k ekud DoFkukad = 351.5 K
'kq) t y d k ok"i nkc = 32.8 mm Hg
'kq) ,FksukWy d k ok"i nkc = 40 mm Hg
t y d k v.kqHkkj = 18 g mol–1
,FksukWy d k v.kqHkkj = 46 g mol–1
iz'uksad k mÙkj nsrsle; foy ;uksad ksvkn'kZruqfoy ;u rFkk foy s;ksad ksvok"i'khy rFkk vfo;ksft r ekusaA

Corporate Office: CG Tower, A-46 & 52, IPIA, Near City Mall, Jhalawar Road, Kota (Raj.)-324005
Website : www.resonance.ac.in | E-mail : contact@resonance.ac.in
Toll Free : 1800 200 2244 | 1800 258 5555 | CIN: U80302RJ2007PLC024029
ADV-SCP - 40
Solution & Colligative Properties
9. The freezing point of the solution M is [JEE 2008, 3/163]
foy ;u M d k fgekad gS& [JEE 2008, 3/163]
(A) 268.7 K (B) 268.5 K (C) 234.2 K (D*) 150.9 K

10. The vapour pressure of the solution M is [JEE 2008, 3/163]


foy ;u M d k ok"i nkc gS [JEE 2008, 3/163]
(A) 39.3 mm Hg (B*) 36.0 mm Hg (C) 29.5 mm Hg (D) 28.8 mm Hg

11. Water is added to the solution M such that the mole fraction of water in the solution becomes 0.9. The
boiling point of this solution is [JEE 2008, 3/163]
foy ;u M esafey k;k x;k t y ft ld smijkUr foy ;u esat y d k eksy fHkUu 0.9 gkst krk gSA bl foy ;u d k DoFkukad
gS% [JEE 2008, 3/163]
(A) 380.4 K (B*) 376.2 K (C) 375.5 K (D) 354.7 K

12. The Henry's law constant for the solubility of N2 gas in water at 298 K is 1.0 × 105 atm. The mole fraction of
N2 in air is 0.8. The number of moles of N2 from air dissolved in 10 moles of water of 298 K and 5 atm
pressure is : [JEE 2009, 3/160]
N2 xSl d h t y esa?kqy u'khy rk (solubility) d sfy , gsujh fu;e fLFkjkad (Henry's law constant) d k eku 298 K ij
1.0 × 105 atm gSA ok;qesaN2 d k eksy izHkkt (mole fraction) 0.8 gSA 10 eksy t y esa298 K vkSj 5 ok;qe.My h; nkc
ij ok;qesamifLFkr N2 d sfo;ksft r (dissolved) eksy ksad h la[;k gS% [JEE 2009, 3/160]
(A*) 4 × 10–4 (B) 4.0 × 10–5 (C) 5.0 × 10–4 (D) 4.0 × 10–6

13. The freezing point (in ºC) of a solution containing 0.1 g of K3[Fe(CN)6] (Mol. Wt. 329) in 100 g of water
(Kf = 1.86 K kg mol–1) is : [JEE 2011, 3/180]
100 g t y (Kf = 1.86 K kg mol–1) es
a0.1 g K3[Fe(CN)6] (v.kqHkkj = 329) d s?kqy uslscusfoy ;u d k fgekad (ºC esa)
gS% [JEE 2011, 3/180]
(A*) – 2.3 × 10–2 (B) – 5.7 × 10–2 (C) – 5.7 × 10–3 (D) – 1.2 × 10–2

14. For a dilute solution containing 2.5 g of a non- volatile non- electrolyte solute in 100 g of water, the elevation
in boiling point at 1 atm pressure is 2ºC. Assuming concentration of solute is much lower than the
concentration of solvent, the vapour pressure (mm of Hg) of the solution is (take Kb = 0.76 K kg mol–1)
[IIT 2012, 3/136]
,d vok"i'khy fo|qr vuvi?kV; (non-volatile non-electrolyte) foy s; d s 2.5 g d ks100 g t y esa?kksy uslscusruq
foy ;u d sfy ,]t y d sDoFkukad (boiling point) esamUu;u, 1 atm ij 2ºC gksrk gSA foy s; d h lkUnzrk d ksfoy k;d d h
lkUnzrk lscgqr d e ekursgq
, foy;u d k ok"i nkc (mm Hg) fuEu gS( Kb = 0.76 K kg mol–1 ys a
)
[IIT 2012, 3/136]
(A*) 724 (B) 740 (C) 736 (D)718
15.* Benzene and naphthalene form an ideal solution at room temperature. For this process, the true statement(s)
is (are) : [JEE(Advanced) 2013, 4/120]
(A) G is positive (B*) Ssystem is positive
(C*) Ssurroundings = 0 (D*) H = 0
csUt hu vkSj us¶Fksy hu lk/kkj.k rkieku ij ,d vkn'kZfoy ;u cukrsgSaA bl izØ e d sfy ;slgh d Fku gS¼gSa½
[JEE(Advanced) 2013, 4/120]
(A) G /kukRed gSA (B*) S¼fud k;½ /kukRed gSA (C*) S¼ifjos'k½ = 0 (D*) H = 0

16. MX2 dissociates into M 2+ and X– ions in an aqueous solution, with a degree of dissociation () of 0.5.
The ratio of the observed depression of freezing point of the aqueous solution to the value of the
depression of freezing point in the absence of ionic dissociation is
[JEE(Advanced) 2014, 3/120]
MX2 ,d t y h; foy ;u es a0.5 d h ,d fo;kst u ek=kk (degree of dissociation)  d sl kFk M 2+ rFkk X– esa
fo;ksft r gksrk gSA ik;sx;st y h; foy ;u d sfgekad v oueu (depression of freezing point) rFkk v k;fud
fo;kst u (depression) d h v uqifLFkfr esafgekad v oueu d k v uqikr gS% [JEE(Advanced) 2014, 3/120]
Ans. 2

Corporate Office: CG Tower, A-46 & 52, IPIA, Near City Mall, Jhalawar Road, Kota (Raj.)-324005
Website : www.resonance.ac.in | E-mail : contact@resonance.ac.in
Toll Free : 1800 200 2244 | 1800 258 5555 | CIN: U80302RJ2007PLC024029
ADV-SCP - 41
Solution & Colligative Properties
17. If the freezing point of a 0.01 molal aqueous solution of a cobalt(III) chloride-ammonia complex (which
behaves as a strong electrolyte) is –0.0558ºC, the number of chloride(s) in the coordination sphere of the
complex is [Kf of water = 1.86 K kg mol–1] [JEE(Advanced) 2015 4/168]
;fn ,d d ksckWYV (III) Dy ksjkbM&veksfu;k lad qy ¼t ksizcy oS/;qr vi?kV~; (strong electrolyte) d h rjg O;ogkj d jrk
gS½ d s0.01 eksy y t y h; foy ;u d k fgekad –0.0558ºC gS] rc bl lad qy d sleUo; e.My esaDy ksjkbM@Dy ksjkbMksa
d h la[;k gS%[Kf ¼t y ½ = 1.86 K kg mol–1] [JEE(Advanced) 2015, 4/168]
Ans. (1)

PART - II : JEE (MAIN) / AIEEE PROBLEMS (PREVIOUS YEARS)


Hkkx - II : JEE (MAIN) / AIEEE ¼fiN y so"kksZ½ d siz'u
1. 6.02 × 1020 molecules of urea are present in 100 ml of its solution. The concentration of urea solution is :
;wfj;k d s6.02 × 1020 v.kq100 ml foy ;u esamifLFkr gSaA ;wfj;k foy ;u d h lkUnzrk gS% [AIEEE-2004]
(1) 0.001 M (2*) 0.01 M (3) 0.02 M (4) 0.1 M.

2. Which one of the following aqueous solutions will exhibit highest boiling point ? [AIEEE-2004]
(1*) 0.01 M Na2SO4 (2) 0.01 M KNO3 (3) 0.015 M urea (4) 0.015 M glucose
fuEu esalsfd l t y h; foy ;u d k lokZf/kd DoFkukad gksxk \ [AIEEE-2004] [Arihant]
(1*) 0.01 M Na2SO4 (2) 0.01 M KNO3 (3) 0.015 M ;w
fj;k (4) 0.015 M Xy w
d kst
3. Equimolar solutions in the same solvent have : [AIEEE-2005] [Arihant]
(1) same boiling point but different freezing point (2) same freezing point but different boiling point
(3*) same boiling and same freezing points (4) differnet boiling and freezing points
leku foy k;d esaleeksy j foy ;u d sfy ;slgh gksxk & [AIEEE-2005] [Arihant]
(1) leku DoFkukad y sfd u fHkUu fgekad (2) leku fgekad y sfd u fHkUu DoFkukad
(3*) leku DoFkukad rFkk leku fgekad (4) fHkUu DoFkukad rFkk fgekad
4. Two solutions of a substance (non electrolyte) are mixed in the following manner. 480 ml of 1.5 M first
solution + 520 mL of 1.2 M second solution. What is the molarity of the final mixture ?
[AIEEE-2005] [Arihant]
,d inkFkZ¼fo|qr vuvi?kV~;½ d snksfoy ;uksad ks1.5 M izFke foy ;u d s480 ml + 1.2 M f}rh; foy ;u d s520 mL
d svuql kj fefJr fd ;k t krk gSA vfUre feJ.k d h eksy jrk D;k gS\ [AIEEE-2005] [Arihant]
(1) 1.20 M (2) 1.50 M (3*) 1.344 M (4) 2.70 M

5. Benzene and toluene form nearly ideal solutions. At 20ºC, the vapour pressure of benzene is 75 torr and
that of toluene is 22 torr. The partial vapour pressure of benzene at 20 ºC for a solution containing 78 g of
benzene and 46 g of toluene in torr is : [AIEEE-2005] [Arihant]
csat hu rFkk VkWy qbZu y xHkx vkn'kZfoy ;u cukrsgSaA 20ºC ij]cast hu vkSj VkWy qbZu d k ok"i nkc Ø e'k%75 Vksj vkSj 22
Vksj gSA csat hu d s78 g rFkk VkWy qbZu d s46 g ;qDr foy ;u d sfy , 20º C ij csat hu d k vkaf'kd ok"i nkc torr esafuEu
gS% [AIEEE-2005] [Arihant]
(1*) 50 (2) 25 (3) 37.5 (4) 53.5

6. If  is the degree of dissociation of Na2SO4, the vant Hoff’s factor (i) used for calculating the molecular
mass is : [AIEEE-2005] [Arihant]
;fn Na2SO4 d sfo;kst u d hek=kk gS]rksvkf.od nzO;eku ifjd fy r d jusd sfy , ç;qDr okWUV gkWQ xq.kkad (i) d keku
D;k gksxk \ [AIEEE-2005] [Arihant]
(1) 1 +  (2) 1 –  (3*) 1 + 2 (4) 1 – 2.

7. Density of a 2.05 M solution of acetic acid in water is 1.02 g/mL. The molality of the solution is
[AIEEE-2006] [Arihant]
t y esa,flfVd vEy d s2.05 M foy ;u d k ?kuRo 1.02 g/mL gSA foy ;u d h eksy y rk fuEu gS%
[AIEEE-2006] [Arihant]
(1) 3.28 mol Kg–1 (2*) 2.28 mol Kg–1 (3) 0.44 mol Kg–1 (4) 1.14 mol Kg–1
Corporate Office: CG Tower, A-46 & 52, IPIA, Near City Mall, Jhalawar Road, Kota (Raj.)-324005
Website : www.resonance.ac.in | E-mail : contact@resonance.ac.in
Toll Free : 1800 200 2244 | 1800 258 5555 | CIN: U80302RJ2007PLC024029
ADV-SCP - 42
Solution & Colligative Properties
8. A mixture of ethyl alcohol and propyl alcohol has a vapour pressure of 290 mm at 300 K. The vapour
pressure of propyl alcohol is 200 mm. If the mole fraction of ethyl alcohol is 0.6, its vapour pressure (in mm)
at the same temperature will be : [AIEEE-2007, 3/120] [Arihant]
300 K ij ,fFky ,Yd ksgkWy vkSj izksfiy ,Yd ksgkWy d sfeJ.k d k ok"i nkc 290 feehgSA t cfd izksfiy ,Yd ksgkWy d k ok"i
nkc 200 feeh gSA ;fn ,fFky ,Yd ksgkWy d k eksy izHkkt 0.6 gks] rksleku rki ij bld k ok"i nkc ¼mm esa½ gksxk%
[AIEEE-2007, 3/120] [Arihant]
(1) 700 (2) 360 (3*) 350 (4) 300

9. A 5.25% solution of a substance is isotonic with a 1.5% solution of urea (molar mass = 60g mol–1) in the
same solvent. If the densities of both the solutions are assumed to be equal to 1.0 g cm–3, molar mass of
the substance will be [AIEEE-2007, 3/120] [Arihant]
leku foy k;d esafd lh inkFkZd k 5.25% foy ;u] ;wfj;k (eksy j nzO;eku = 60g mol–1) d s1.5% foy ;u d slkFk
leijkljh gSA ;fn nksuksad k ?kuRo leku gSat ksfd 1.0 g cm–3 gS] rksinkFkZd k eksy j nzO;eku gksxkA
[AIEEE-2007, 3/120] [Arihant]
(1) 105.0 g mol –1
(2*) 210.0 g mol –1
(3) 90.0 g mol –1
(4) 15.0 g mol–1

10. The vapour pressure of water at 20ºC is 17.5 mm Hg. If 18 g of glucose (C6H12O6) is added to 178.2 g of
water at 20°C, the vapour pressure of the resulting solution will be : [AIEEE-2008, 3/105] [SMD]
20ºC ij t y d k ok"i nkc 17.5 mm Hg gS A ;fn 20ºC ij 178.2 g t y esa18 g Xy wd kst (C6H12O6) fey k;k t krk
gS] rksizkIr foy ;u d k ok"i nkc gksxk & [AIEEE-2008, 3/105] [SMD]
(1) 15.750 mm Hg (2) 16.500 mm Hg (3*) 17.325 mm Hg (4) 17.675 mm Hg

11. At 80ºC , the vapour pressure of pure liquid 'A' is 520 mm Hg and that of pure liquid 'B' is 1000 mm Hg. If
a mixture solution of 'A' and 'B' boils at 80º C and 1 atm pressure, the amount of 'A' in the mixture is
(1 atm = 760 mm Hg) [AIEEE-2008, 3/105] [SMD]
(1) 34 mol percent (2) 48 mol percent (3*) 50 mol percent (4) 52 mol percent
80º C ij 'kq) nzo ‘A’ d k ok"i nkc 520 mm Hg vkSj 'kq) nzo ‘B’ d k ok"i nkc 1000 mm Hg gSA ;fn 'A’ vkSj ‘B’
d k fefJr foy ;u 80º C vkSj 1 atm nkc ij mcy rk gS] rksfeJ.k esa‘A’ d h ek=kk gS(1 atm = 760 mm Hg)
[AIEEE-2008, 3/105] [SMD]
(1) 34 eksy izfr'kr (2) 48 eksy izfr'kr (3*) 50 eksy izfr'kr (4) 52 eksy izfr'kr
12. A binary liquid solution is prepared by mixing n-heptane and ethanol. Which one of the following statement
is correct regarding the behaviour of the solution ? [AIEEE-2009, 4/144] [SMD]
(1*) The solution is non-ideal, showing +ve deviation from Raoult’s Law.
(2) The solution in non-ideal, showing –ve deviation from Raoult’s Law.
(3) n-heptane shows +ve deviation while ethanol shows –ve deviation from Raoult’s Law.
(4) The solution formed is an ideal solution.
n-gsIVsu vkSj ,FksukWy d ksfey kd j f}vaxh&nzo foy ;u cuk;kt krkgSA foy ;u d sO;ogkj lslacfU/kr fuEu d Fkuksaesad kSu
lR; gS? [AIEEE-2009, 4/144] [SMD]
(1*) foy ;u vukn'kZgSvkSj jkWÅ YV fu;e ls/kukRed fopy u n'kkZrk gSA
(2) foy ;u vukn'kZgSvkSj jkWÅ YV fu;e ls_ .kkRed fopy u n'kkZrk gSA
(3) jkW
Å YV fu;e lsn-gsIVsu /kukRed t cfd ,FkSukWy _ .kkRed fopy u n'kkZrk gSA
(4) fufeZ
r foy ;u ,d vkn'kZfoy ;u gSA
13. Two liquids X and Y form an ideal solution. At 300K, vapour pressure of the solution containing 1 mol of X
and 3 mol of Y is 550 mmHg. At the same temperature, if 1 mol of Y is further added to this solution, vapour
pressure of the solution increases by 10 mmHg. Vapour pressure (in mmHg) of X and Y in their pure states
will be, respectively : [AIEEE-2009, 8/144]
(1) 300 and 400 (2*) 400 and 600 (3) 500 and 600 (4) 200 and 300
nksnzo X vkSj Y ,d vkn'kZfoy ;u cukrsgSaA 300K ij X d s1 eksy vkSj Y d s3 eksy fey kuslscusgq, foy ;u d k ok"i
nkc 550 mmHg gSA mlh rki ij ;fn ml foy ;u esaY d k 1 eksy vksj fey k fn;k t krk gS] rksfoy ;u d k ok"i nkc
10 mmHg lsc<+t krk gS
A 'kq) voLFkk esaX vkSj Y d k ok"i nkc (mmHg esa) Ø e'k%gksxk & [AIEEE-2009, 8/144]
(1) 300 vkS
j 400 (2*) 400 vkS
j 600 (3) 500 vkS
j 600 (4) 200 vkS
j 300
Corporate Office: CG Tower, A-46 & 52, IPIA, Near City Mall, Jhalawar Road, Kota (Raj.)-324005
Website : www.resonance.ac.in | E-mail : contact@resonance.ac.in
Toll Free : 1800 200 2244 | 1800 258 5555 | CIN: U80302RJ2007PLC024029
ADV-SCP - 43
Solution & Colligative Properties
14. If sodium sulphate is considered to be completely dissociated into cations and anions in aqueous solution,
the change in freezing point of water (Tf), when 0.01 mole of sodium sulphate is dissolved in 1 kg of water,
is (Kf = 1.86 K kg mol–1) [AIEEE-2010, 4/144] [SMD]
;fn ;g eku fy ;k t k, fd lksfM;e lYQ sV iw.kZ: i lst y h; foy ;u esa/kuk;uksarFkk _ .kk;uksaesafo;ksft r gkst krk
gS] rks t y d s fgekad esa ifjorZu (Tf) D;k gksxk t c 1 kg t y esa 0.01 eksy lksfM;e lYQ sV ?kksy k x;k gks\
(Kf = 1.86 K kg mol–1) [AIEEE-2010, 4/144] [SMD]
(1) 0.0372 K (2*) 0.0558 K (3) 0.0744 K (4) 0.0186 K
15. On mixing, heptane and octane form an ideal solution. At 373 K, the vapour pressures of the two liquid
components (heptane and octane) are 105 kPa and 45 kPa respectively. Vapour pressure of the solution
obtained by mixing 25.0 g of heptane and 35 g of octane will be (molar mass of heptane = 100 g mol–1 and
of octane = 114 g mol–1) [AIEEE-2010, 4/144] [SMD]
gsIVsu rFkk vkWDVsu fey kusij ,d vkn'kZfoy ;u cukrk gSaA 373 K ij, nksnzo vo;oksa¼gsIVsu rFkk vkWDVsu½ d sok"i nkc
Ø e'k%105 kPa rFkk 45 kPa gSaA 25.0 g gsIVsu rFkk 35 g vkWDVsu d ksfefJr d juslsizkIr foy ;u d k ok"i nkc fuEu
gksxk (eksy j nzO;eku( gsIVsu = 100 g mol–1 rFkk vkWDVsu = 114 g mol–1) [AIEEE-2010, 4/144] [SMD]
(1*) 72.0 kPa (2) 36.1 kPa (3) 96.2 kPa (4) 144.5 kPa
16. Kf for water is 1.86 K kg mol–1. If your automobile radiator holds 1.0 kg of water, how may grams of ethylene
glycol (C2H6O2) must you add to get the freezing point of the solution lowered to –2.8ºC ?
[AIEEE 2012, 4/120]
t y d sfy , Kf d k eku 1.86 K kg mol–1 gSA ;fn vki d svkWVkseksckby jsfM;sVj esa1.0 kg t y Hkjk gks] rksfoy ;u d s
fgekad d ks–2.8ºC rd d e d jusd sfy , ,Fkkby hu Xy kbd ksy (C2H6O2) d sfd rusxzke vki d ksfey kusgksaxs\
[AIEEE 2012, 4/120]
(1) 72 g (2*) 93 g (3) 39 g (4) 27 g
17. Consider separate solution of 0.500 M C2H5OH(aq), 0.100 M Mg3(PO4)2(aq), 0.250 M KBr(aq) and
0.125 M Na3PO4(aq) at 25C. Which statement is true about these solution, assuming all salts to be strong
electrolytes ? [AIEEE 2014, 4/120]
(1*) They all have the same osmotic pressure.
(2) 0.100 M Mg3(PO4)2(aq) has the highest osmotic pressure.
(3) 0.125 M Na3PO4(aq) has the highest osmotic pressure.
(4) 0.500 M C2H5OH(aq) has the highest osmotic pressure.
0.500 M C2H5OH(t y h;), 0.100 M Mg3(PO4)2(t y h;) 0.250 M KBr (t y h;) vkS j 0.125 M Na3PO4(t y h;) d s
iF̀kd foy ;uksad k25C ij fopkj d hft ,A lHkh y o.kksad ksizcy oS| qrvi?kV~; ekursgq, fuEu d Fkuksaesalsd kSu&lk
d Fku lR; gS\ [AIEEE 2014, 4/120]
(1*) bu lc d sfy ;sijklj.k nkc d seku leku gks xk
(2) 0.100 M Mg3(PO4)2(t y h;) d k ijklj.k nkc mPpre gks xkA
(3) 0.125 M Na3PO4(t y h;) d k ijklj.k nkc mPpre gks xkA
(4) 0.500 M C2H5OH(t y h;) d k ijklj.k nkc mPpre gks xkA
18. The vapour pressure of acetone at 20ºC is 185 torr. When 1.2 g of a non-volatile substance was dissolved
in 100 g of acetone at 20ºC, its vapour pressure was 183 torr. The molar mass (g mol–1) of the substance
is: [JEE(Main) 2015, 4/120]
20ºC ij ,s flVksu d h ok"i nkc 185 torr gSA t c 20ºC ij] 1.2 g vok"i'khy inkFkZd ks100 g ,sflVksu esa?kksy k x;k]
rc ok"i nkc 183 torr gksx;kA bl inkFkZd k eksy j nzO;eku (g mol–1 esa) gSA [JEE(Main) 2015, 4/120]
(1) 32 (2*) 64 (3) 128 (4) 488

Corporate Office: CG Tower, A-46 & 52, IPIA, Near City Mall, Jhalawar Road, Kota (Raj.)-324005
Website : www.resonance.ac.in | E-mail : contact@resonance.ac.in
Toll Free : 1800 200 2244 | 1800 258 5555 | CIN: U80302RJ2007PLC024029
ADV-SCP - 44
Solution & Colligative Properties

 Marked Questions may have for Revision Questions.


 fpfUgr iz
'u nksgjkus;ksX; iz'u gSA

This Section is not meant for classroom discussion. It is being given to promote self-study
and self testing amongst the Resonance students.

Hkkx - I : PRACTICE TEST-1 (IIT-JEE (MAIN Pattern))


Max. Marks : 120 Max. Time : 1 Hr.

Important Instructions
egÙoiw.kZfunsZ'k %
1. The test is of 1 hour duration.
ijh{kkd hvof/k1 ?kaVsgSA
2. The Test Booklet consists of 30 questions. The maximum marks are 120.
bl ijh{kk iqfLrd k esa30 iz'u gSA vf/kd re vad 120 gSA
3. Each question is allotted 4 (four) marks for correct response.
lHkh iz'uksad svad leku gSA izR;sd iz'u d slgh mÙkj d sfy , 4 ¼pkj½ vad fu/kkZfjr fd ;sx;sgSA
4. Candidates will be awarded marks as stated above in Instructions No. 3 for correct response of each question.
¼ (one fourth) marks will be deducted for indicating incorrect response of each question. No deduction from the
total score will be made if no response is indicated for an item in the answer sheet.
vH;kfFkZ;ksad ksizR;sd lghmÙkj d sfy , mijksDr funs
Z'ku la[;k3 d sfuns
Z'kkuql kj ekDlZfn;st k,a
xsA izR;sd iz'u d sxy r mÙkj
d sfy ;s¼ okaHkkx fy ;kt k;sxkA ;fn mÙkj iqfLrd kesafd lhiz'u d kmÙkj ughafn;kx;kgksrksd qy izkIrkad lsd ksbZd VkSrh
ughafd t k;sxhA
5. There is only one correct response for each question. Filling up more than one response in any question will be
treated as wrong response and marks for wrong response will be deducted accordingly as per instructions 4
above.
çR;sd iz'u d k d soy ,d gh lgh mÙkj gSA ,d lsvf/kd mÙkj nsusij mlsxy r mÙkj ekuk t k;sxk vkSj mijksDr funsZ'k 4
d svuql kj vad d kV fy ;st k;saxsA
1. Mole fraction of C3H5(OH)3 in a solution of 36 g of water and 46 g of glycerine is :
t y d s36 g ,oe~fXy ljhu~d s46 g d s,d foy ;u esaC3H5(OH)3 d k eksy izHkkTk fuEu gS%
(1) 0.46 (2) 0.36 (3*) 0.20 (4) 0.40

2. Colligative properties have many practical uses, some of them may be :
I : Melting of snow by salt
II : Desalination of sea water
III : Determination of molar mass
IV : Determination of melting point and boiling point of solvent
Actual practical uses are :
v.kql a[;d xq.k/keZd sd bZizk;kSfxd mi;ksx gksrsgSmuesalsd qN gksld rsgSA
I : y o.k }kjk cQ Zd k xy u
II : leqnzh t y d k foy o.khd j.k
III : eks
y j nzO;eku d h x.kuk
IV : foy k;d d sxy uka d rFkk DoFkukad d h x.kuk
okLrfod izk;ks fxd mi;ks x gSA
(1) I, II (2) III, IV (3*) I, II, III (4) II, III, IV

Corporate Office: CG Tower, A-46 & 52, IPIA, Near City Mall, Jhalawar Road, Kota (Raj.)-324005
Website : www.resonance.ac.in | E-mail : contact@resonance.ac.in
Toll Free : 1800 200 2244 | 1800 258 5555 | CIN: U80302RJ2007PLC024029
ADV-SCP - 45
Solution & Colligative Properties
3. Select correct statement(s) :
(1) When solid CaCl2 is added to liquid water, the boiling temperature rises
(2) When solid CaCl2 is added to ice at 0°C, the freezing temperature falls
(3*) Both (1) and (2)
(4) None of the above
lghd Fku d kspqfu;s
(1) t c Bksl CaCl2 d ksnzo t y esafey krsgaSrksDoFkukad esaof̀) gksrh gSA
(2) 0°C ij t c cQ Zes aBksl CaCl2 fey k;k t krk gSfgekad rkieku esad eh gksrh gSA
(3*) (1) rFkk(2) nks
uks
a
(4) bues alsd ksbZugha
4. If pKa = – log Ka = 4, and Ka = Cx2 then Van’t Hoff factor for weak monobasic acid when C = 0.01 M is :
;fn pKa = – log Ka = 4, rFkkKa = Cx2 rksnqcZy ,y d {kkjh; vEy d sfy , okUV gkWQ xq.kkad gksxk t c C = 0.01 M gSA
(1) 1.01 (2) 1.02 (3*) 1.10 (4) 1.20

5. Consider following terms (m = molality) :


Tb
I : mKb ; II : mKbi III : IV : Kb
i
Terms which can be expressed in degree (temperature) are

fuEu inksad kslfEefy r d jrsgq, (m = eksy y rk) :


Tb
I : mKb ; II : mKbi III : IV : Kb
i
og in t ksfd (rki) d ksfMxzh esaiznf'kZr d jrk gSA
(1) III, IV (2) I, II (3*) I, II, IIII (4) I, III

6. Elevation in b.p. of an aqueous urea solution is 0.52°. (Kb = 0.52° mol–1 kg) Hence, mole-fraction of urea in
this solution is :
;wfj;kd st y h; foy ;u d kDoFkukad es
amUu;u 0.52° gS
A (Kb = 0.52° mol–1 kg) vr%bl foy;u esa;wfj;kd keksy izHkkt
gS
A
(1) 0.982 (2) 0.567 (3) 0.943 (4*) 0.018

7. Insulin (C2H10O5)n is dissolved in a suitable solvent and the osmotic pressure  of the solution of various
concentration (in kg/m 3) is measured at 20ºC. The slope of a plot of  against c is found to be
8.134 × 10–3 (SI units) The molecular weight of the insulin (in kg/mol) is :
bUlw
fyu (C2H10O5)n d ks,d mi;q Dr foyk;d es a?kks
y kt krkgSvkS
j foy;u d kijklj.knkc  ) fofHkUUklkUnz
rkvks
a (kg/
m3) es
a20ºC ij ekikx;k gSA oØ d k<+ky , c d sfo: ) 8.314 × 10–3 (S units) ik;k x;kgSA bUlwfy u d kv.kqHkkj
D;k gksxkA ( fd y ksxzke/eksy esa) \
(1) 4.8 × 105 (2) 9 × 105 (3*) 293 × 103 (4) 8.314 × 105

8. What is the normal boiling point of the solution represented by the phase diagram ?

Corporate Office: CG Tower, A-46 & 52, IPIA, Near City Mall, Jhalawar Road, Kota (Raj.)-324005
Website : www.resonance.ac.in | E-mail : contact@resonance.ac.in
Toll Free : 1800 200 2244 | 1800 258 5555 | CIN: U80302RJ2007PLC024029
ADV-SCP - 46
Solution & Colligative Properties
izkoLFkk vkjs[k }kjk iznf'kZr foy ;u d k lkekU; DoFkukad D;k gksrk gS\

P, 1 ok;qe.My h;
ok;qe.My h;
k; d
foy

A B C D

T°C
(1) A (2) B (3) C (4*) D
9. An aqueous solution of a solute AB has b.p. of 101.08°C (AB is 100% ionised at boiling point of the solution)
and freezes at – 1.80°C. Hence, AB (Kb / Kf = 0.3)
(1) is 100% ionised at the f.p. of the solution (2*) behaves as non-electrolyte at the f.p. of the solution
(3) forms dimer (4) none of the above
foy s; AB d st y h; foy ;u d k DoFkukad 101.08°C gSA (foy ;u d sDoFkukad ij AB, 100% vk;fur gksrk gS) rFkk
– 1.80°C ij t erk gS A vr% AB (Kb / Kf = 0.3)
(1) foy ;u d sfgeka
d fcUnqij 100% vk;fur gksrk gSA
(2*) foy ;u d sfgeka
d fcUnqij vuvi?kV~; d h rjg O;ogkj iznf'kZr d jrk gSA
(3) f}y d cukrk gSA
(4) mijksDr esalsd ksbZugha
10. Density of 1M solution of a non-electrolyte C6H12O6 is 1.18 g/mL. If Kf (H2O) is 1.86° mol–1 kg, solution freezes
at :
,d fo|qr vuvi?kV~; C6H12O6 d s1 M foy ;u d k?kuRo 1.18 g/mL gSA ;fn Kf (H2O) 1.86° mol–1 kg gSrksfoy ;u d k
fgekad gksxk %
(1) – 1.58°C (2*) – 1.86°C (3) – 3.16°C (4) 1.86°C
11. Mole fraction of a non-electrolyte in aqueous solution is 0.07. If Kf is 1.86° mol–1 kg, depression in f.p., Tf, is:
,d fo|q r vuvi?kV~ ; foy;u d kt yh; foy;u es aeks
y izHkkt 0.07 gS A ;fn Kf 1.86° mol–1 kg gSrFkkfgekad es avoueu,
Tf, gks
xk%
(1) 0.26° (2) 1.86° (3) 0.13° (4*) 7.78°
12. What is the normal freezing point of the solution represented by the phase diagram ?

izkoLFkkvkjs[k d s}kjkfoy ;u d k lkekU; fgekad fcUnqfd ld s}kjkiznf'kZr fd ;kt k ld rkgSA

(1*) T1 (2) T2 (3) T3 (4) T0

Corporate Office: CG Tower, A-46 & 52, IPIA, Near City Mall, Jhalawar Road, Kota (Raj.)-324005
Website : www.resonance.ac.in | E-mail : contact@resonance.ac.in
Toll Free : 1800 200 2244 | 1800 258 5555 | CIN: U80302RJ2007PLC024029
ADV-SCP - 47
Solution & Colligative Properties
13. Total vapour pressure of mixture of 1 mol of volatile component A (pA° = 100 mmHg) and 3 mol of volatile
component B (pB° = 60 mmHg) is 75 mm. For such case :
(1) there is positive devitation from Raoult’s low
(2) boiling point has been lowered
(3) force of attraction between A and B is smaller than that between A and A or between B and B
(4*) all the above statements are correct
1 mol ok"i'khy inkFkZA (pA° = 100 mmHg) rFkk 3 eks y ok"i'khy inkFkZB (pB° = 60 mmHg) d sfeJ.k d kd qy ok"i
nkc 75 mm gSA bl ifjfLFkfr d sfy , &
(1) jkÅ YV fu;e ls/kukRed fopy u n'kkZrk gSA
(2) DoFkuka
d d e gksrkgSA
(3) A rFkk B d se/; vkd "kZ
.k cy A o A d se/; ;k B o B d se/; d h rqy uk esad e gksrk gSA
(4*) mijks
Dr lHkhd Fku lghgSA
14. A colligative property of a solution depends on the :
(1) arrangement of atoms in solute molecule (2) total number of molecules of solute and solvent
(3*) number of molecules of solute in solution (4) mass of the solute molecules
foy ;u d sv.kql a[; xq.k/keZfuHkZj d jrsgSA
(1) foy s
; v.kqesaijek.kqv ksad h O;oLFkk ij (2) foy s
; rFkk foy k;d d sv.kqv ksad h d qy la[;k ij
(3*) foy ;u es
afoy s; v.kqv ksad h la[;k ij (4) foy s
; v.kqv ksad snzO;eku ij
15. Which has maximum freezing point ?
(1*) 6g urea solution in 100 g H2O (2) 6g acetic acid solution in 100g H2O
(3) 6g sodium chloride in 100 g H2O (4) All have equal freezing point
fd ld k fgekad fcUnqvf/kd re gS?
(1*) 100 g H2O es
a6g ;wfj;k foy ;u (2) 100 g H2O es
a6g ,lhfVd vEy foy ;u
(3) 100 g H2O es
a6g lksfM;e Dy ksjkbM (4) lHkhd kleku fgekad fcUnqgksrkgSA
16. Select correct statements :
(1) The fundamental cause of all colligative properties is the higher entropy of the solution relative to that of
the pure solvent
(2) The freezing point of hydrofluoride solution is larger than that of equimolal hydrogen chloride solution
(3) 1M glucose solution and 0.5 M NaCl solution are isotonic at a given temperature
(4*) All are correct statements
lgh d Fku pqfu;s%
(1) lHkhv.kql a[;d xq.k/keZd kvk/kkjHkwr d kjd 'kq) foy k;d d slkis{kfoy ;u d hmPp ,UVªksihgksukgSA
(2) gkbMª
ks¶y ksjkbZM foy ;u d kfgekad leeksy y gkbMªkst u Dy ksjkbM foy ;u d hvis{kkvf/kd gksrkgSA
(3) fn;sx;srki ij 1 M Xy w d ksl foy ;u rFkk 0.5 M NaCl foy ;u leijkljh gksrsgSA
(4*) mijksDr lHkhlghd Fku gSA
17. The vapour pressure of a pure liquid A is 40 mmHg at 310 K. The vapour pressure of this liquid in a solution
with liquid B is 32 mmHg. Mole fraction of A in the solution, if it obeys Raoult’s law is :
310 K ij 'kq ) nzo A d kok"inkc 40 mmHg gSA nzo B d slkFkbl nzo foy;u d kok"i nkc 32 mmHg gSA ;fn foy;u
jkmYV fu;e d k iky u d jrk gSrksfoy ;u esaA d h eksy fHkUu gSA
(1*) 0.8 (2) 0.5 (3) 0.2 (4) 0.4

18. Mole fraction of the toluene in the vapour phase which is in equilibrium with a solution of benzene (p° = 120
Torr) and toluene (p° = 80 Torr) having 2.0 mol of each is
ok"i voLFkkesaVky wbZu d keksy izHkkt D;kgSA t kscsUt hu rFkkVky wbZu izR;sd d s2.0 eksy d sfoy ;u d slkFklkE; esagSA
(1) 0.50 (2) 0.25 (3) 0.60 (4) 0.40

19. An azeotropic solution of two liquids has a boiling point lower than either of them when it :
(1) shows negative deviation from Raoult’s law (2*) shows positive deviation from Raoult’s low
(3) shows ideal behaviour (4) is saturated

Corporate Office: CG Tower, A-46 & 52, IPIA, Near City Mall, Jhalawar Road, Kota (Raj.)-324005
Website : www.resonance.ac.in | E-mail : contact@resonance.ac.in
Toll Free : 1800 200 2244 | 1800 258 5555 | CIN: U80302RJ2007PLC024029
ADV-SCP - 48
Solution & Colligative Properties
nksnzoksad sfLFkjDokFkh foy ;u d k DoFkukad bu nksuksaesalsfd lh u fd lh d sDoFkukad lsd e gksrk gSA t c ;g
(1) jkW
m YV fu;e ls_ .kkRed fopy u n'kkZrsgSA (2*) jkW
m YV fu;e ls/kukRed fopy u n'kkZrsgSA
(3) vkn'kZO;ogkj iz nf'kZr d jrsgSA (4) larÌr gksrsgSA
20. Depression of freezing point of 0.01 molal aq. CH3COOH solution is 0.02046°. 1 molal urea solution freezes
at – 1.86°C. Assuming molality equal to molarity, pH of CH3COOH solution is :
0.01 eksy y t y h; CH3COOH foy ;u d sfgekad fcUnqd k voueu 0.02046° gSA ,d eksy y ;wfj;k foy ;u – 1.86°C
ij t erk gSA ekuk fd eksy y rk eksy jrk d scjkcj gksrh gSA CH3COOH foy ;u d h pH gSA
(1) 2 (2*) 3 (3) 3.2 (4) 4.2

21. Which of the following azeotropic solutions has the boiling point more than boiling point of the constituents
A and B ?
(1) CH3CH2OH and CH3COCH3 (2) CS2 and CH3COCH3
(3*) CHCl3 and CH3COCH3 (4) CH3CHO and CS2
fuEu esalsd kSu lsfLFkjDokFkh foy ;u d k DoFkukad bld s?kVd A rFkk B d sDoFkukad d h rqy uk esavf/kd gksrk gSA
(1) CH3CH2OH rFkkCH3COCH3 (2) CS2 rFkkCH3COCH3
(3*) CHCl3 rFkkCH3COCH3 (4) CH3CHO rFkkCS2

22. A 0.50 molal solution of ethylene glycol in water is used as coolant in a car. If the freezing point constant of
water is 1.86° per molal, at which temperature will the mixture freeze ?
t y esa,Fkhfy u Xy kbd kWy d s0.50 eksy y foy ;u d k mi;ksx d kj esaB.Md d sfy , fd ;k t krk gSA ;fn t y d k fgekad
fcUnqfLFkjkad 1.86° izfr eksy y gSrksog rki D;k gksxk ft l ij feJ.k B.Mk gkst k, \
(1) 1.56°C (2*) – 0.93°C (3) – 1.86°C (4) 0.93°C

23. The depression of freezing points of 0.05 molal aqueous solution of the following compounds are measured.
1. NaCl 2. K2SO4 3. C6H12O6 4. Al2(SO4)3
Which one of the above compounds will exhibit the maximum depression of freezing point ?
(1) 3 (2) 2 (3*) 4 (4) 1
fuEufy f[kr ;kSfxd ksad s0.05 eksy y t y h; foy ;u d sfgekad voueu d ksekik x;kA
1. NaCl 2. K2SO4 3. C6H12O6 4. Al2(SO4)3
mijksDr esad kSulk ,d ;kSfxd vf/kd re fgekad voueu d ksiznf'kZr d jsxkA
(1) 3 (2) 2 (3*) 4 (4) 1

24. On mixing 10 mL of acetone with 40 mL of chloroform , the total volume of the solution is :
(1*) < 50 mL (2) > 50 mL (3) = 50 mL (4) Cannot be predicted
,lhVksu d s10 mL d ksDy ksjksQ keZd s40 mL d slkFk fey kusij foy ;u d k d qy vk;ru gksxkA
(1*) < 50 mL (2) > 50 mL (3) = 50 mL (4) Kkr ughafd ;k t kld rk gS
a
25. Select correct statement ?
(1) Heats of vaporisation for a pure solvent and for a solution are similar because similar intermolecular forces
between solvent molecules must be overcome in both cases
(2) Entropy change between solution and vapour is smaller than the entropy change between pure solvent
and vapour
(3) Boiling point of the solution is larger than that of the pure solvent
(4*) All are correct statements
lgh d Fku d kspqfu;s\
(1) 'kq
) foy k;d d sfy , rFkkfoy ;u d sfy , ok"iu d hÅ "ekleku gksrhgSD;ksafd nksuksaifjfLFkfr;ksaesafoy k;d v.kqv ksa
d se/; vUrjkf.od vkd "kZ.k cy leku gksrk gSA
(2) 'kq
) foy k;d rFkkok"i d se/; ,UVªksihifjoZru d hrqy ukesafoy ;u rFkkok"i d se/; ,UVªksihifjorZu vYi gksrkgSA
(3) 'kq
) foy k;d d h rqy uk esafoy ;u d k DoFkukad vf/kd gksrk gSA
(4*) lHkhd Fku lgh

Corporate Office: CG Tower, A-46 & 52, IPIA, Near City Mall, Jhalawar Road, Kota (Raj.)-324005
Website : www.resonance.ac.in | E-mail : contact@resonance.ac.in
Toll Free : 1800 200 2244 | 1800 258 5555 | CIN: U80302RJ2007PLC024029
ADV-SCP - 49
Solution & Colligative Properties
26. What will be the molecular weight of NaCl determined experimentally from elevation in the boiling point or
depression in freezing point method ?
(1*) < 58.5 (2) > 58.5 (3) = 58.5 (4) None of these
DoFkukad esamUu;u ;k fgekad easvoueu d svuql kj izk;ksfxd : i lsNaCl d k vkf.od Hkkj D;k gksxkA
(1*) < 58.5 (2) > 58.5 (3) = 58.5 (4) bues
alsd ksbZugha
27. Which characterises the weak intermolecular forces of attraction in a liquid ?
(1) High boiling point (2*) High vapour pressure
(3) High critical temperature (4) High heat of vaporization
nzo esanqcZy vUrjkf.od vkd "kZ.k cy d k vfHky {k.k gSA
(1) vf/kd DoFkuka
d (2*) vf/kd ok"i nkc (3) vf/kd Ø kfUrd rki (4) vf/kd ok"iu d h m"ek
28. On the basis of intermolecular forces predict the correct order of decreasing boiling points of the compounds:
vUrjkf.od cy d svk/kkj ij ;kSfxd ksad s?kVrsgq, DoFkukad d k lgh Ø e igpkfu;s
(1) CH3OH > H2 > CH4 (2*) CH3OH > CH4 > H2 (3) CH4 > CH3OH > H2 (4) H2 > CH4 > CH3OH

29. During depression of freezing point in a solution the following are in equilibrium :
(1*) Liquid solvent, solid solvent (2) Liquid solvent, solid solute
(3) Liquid solute, solid solute (4) Liquid solute, solid solvent
foy ;u esafgekad esavoueu d snkSjku fuEu lkE; esagksrsgSA
(1*) nz
o foy k;d ]Bksl foy k;d (2) nzo foy k;d ]Bksl foy s;
(3) nz
o foy s;]Bksl foy s; (4) nzo foy s;]Bksl foy k;d
30. Which of the following liquid pairs shows a positive deviation from Raoult’s law ?
(1) Acetone – chloroform (2*) Benzene – methanol
(3) Water – nitric acid (4) Water – hydrochloric acid
fuEu esad kSulk nzo ;qXe jkWm YV fu;e ls/kukRed fopy u iznf'kZr d jrk gS
(1) ,lhVks
u&Dy ksjksQ keZ (2*) csUt hu&esFks
ukWy (3) t y &ukbfVª d vEy (4) t y &gkbMª
ks
Dyks
fjd vEy

PART - II : PRACTICE TEST-2 (IIT-JEE (ADVANCED Pattern))


Hkkx - II : PRACTICE TEST-2 (IIT-JEE (ADVANCED Pattern))
Max. Time : 1 Hr. Max. Marks : 66
Important Instructions
egÙoiw.kZfunsZ'k %
A. General l kekU; %
1. The test is of 1 hour duration.
ijh{kk d h vof/k 1 ?kaVsgSA
2. The Test Booklet consists of 22 questions. The maximum marks are 66.
bl ijh{kk iqfLrd k esa22 iz'u gSA vf/kd re vad 66 gSA
B. Question Paper Format
iz'u&i=k d k izk: i
3. Each part consists of five sections.
bl iz'u&i=k esaik¡p [kaM gSaA
4. Section 1 contains 7 multiple choice questions. Each question has four choices (A), (B), (C) and (D) out
of which ONE is correct.
[kaM 1 esa7 cgqfod Yi iz'u gSaA gj iz'u esapkj fod Yi (A), (B), (C) vkSj (D) gSaft uesals,d lgh gSaA
5. Section 2 contains 5 multiple choice questions. Each question has four choices (A), (B), (C) and (D) out
of which ONE OR MORE THAN ONE are correct.
[kaM 2 esa5 cgqfod Yi iz'u gSaA gj iz'u esapkj fod Yi (A), (B), (C) vkSj (D) gSaft uesals,d ;k ,d lsvf/kd lgh gSaA

Corporate Office: CG Tower, A-46 & 52, IPIA, Near City Mall, Jhalawar Road, Kota (Raj.)-324005
Website : www.resonance.ac.in | E-mail : contact@resonance.ac.in
Toll Free : 1800 200 2244 | 1800 258 5555 | CIN: U80302RJ2007PLC024029
ADV-SCP - 50
Solution & Colligative Properties
6. Section 3 contains 6 questions. The answer to each of the questions is a single-digit integer, ranging from
0 to 9 (both inclusive).
[kaM 3 esa6 iz'u gSaA izR;sd iz'u d k mÙkj 0 ls9 rd ¼nksuksa'kkfey ½ d schp d k ,d y vad h; iw.kkZad gSA
7. Section 4 contains 1 paragraphs each describing theory, experiment and data etc. 3 questions relate to
paragraph. Each question pertaining to a partcular passage should have only one correct answer among
the four given choices (A), (B), (C) and (D).
[k.M 4 esafl) kUrks
a]iz;ksxks
avkSj vk¡d M+
ks
avkfn d ksn'kkZusoky s1 vuqPNs
n gS
aA vuqPNs
n lslacaf/kr rhu iz'u gS
aA fd lhHkhvuq
PNsn
esagj iz'u d spkj fod Yi (A), (B), (C) vkSj (D) gSaft uesalsd soy ,d gh lgh gSA
8. Section 5 contains 1 multiple choice questions. Question has two lists (list-1 : P, Q, R and S; List-2 : 1,
2, 3 and 4). The options for the correct match are provided as (A), (B), (C) and (D) out of which ONLY ONE
is correct.
[kaM 5 esa1 cgqfod Yi iz'u gSaA iz'u esankslwfp;k¡(lwp h-1 : P, Q, R vkSj S; lwp h-2 : 1, 2, 3 vkSj 4) gSA lghfey ku d sfy ,
fod Yi (A), (B), (C) vkSj (D) gSaft uesalsd soy ,d lgh gSA
C. Marking Scheme v a d u ;kst uk
9. For each question in Section 1, 4 and 5 you will be awarded 3 marks if you darken the bubble corresponding
to the correct answer and zero mark if no bubble is darkened. In all other cases, minus one (– 1) mark will
be awarded.
[k.M 1, 4 vkSj 6 d sgj iz'u esad s
oy lghmÙkj oky scqy cqy sd ksd ky kd jusij 3 vad vkSj d ksbZHkhcqy cq
y kd kykughad jus
ij 'kwU; (0) vad iznku fd , t k;sxsaA vU; lHkh fLFkfr;ksaesa_ .kkRed ,d (– 1) vad iznku fd ;k t k;sxkA
10. For each question in Section 2, you will be awarded 3 marks. If you darken all the bubble(s) corresponding
to the correct answer(s) and zero mark. If no bubbles are darkened. No negative marks will be answered
for incorrect answer in this section.
[kaM 2 esagj iz'u esalHkhlghmÙkj ¼mÙkjksa½ oky scqy cqy s¼cqy cqy ksa½ d ksd ky kd jusij 3 vad iznku fd ;st k;sxsavkSj d ksbZHkh
cqy cqy k d ky k ughad jusij 'kwU; vad iznku fd ; t k;sxsaA bl [kaM d siz'uksaesaxy r mÙkj nsusij d ksbZ_ .kkRed vad ugha
fn;st k;sxs aA
11. For each question in Section 3, you will be awarded 3 marks if you darken only the bubble corresponding
to the correct answer and zero mark if no bubble is darkened. No negative marks will be awarded for
incorrect answer in this section.
[kaM 3 esagj iz'u esalHkhlghmÙkj oky scqy cqy sd ksd ky kd jusij 3 vad iznku fd ;st k;sxsavkSj d ksbZHkhcqy cqy kd ky kugha
d jusij 'kwU; vad iznku fd ; t k;sxsaA bl [kaM d siz'uksaesaxy r mÙkj nsusij d ksbZ_ .kkRed vad ughafn;st k;sxsaA

SECTION-1 : (Only One option correct Type)


This section contains 7 multiple choice questions. Each questions has four choices (A), (B), (C) and
(D) out of which Only ONE option is correct.
[k.M –1 : (d soy ,zd l gh fod Yi çd kj)
bl [k.M esa7 cgqfod Yi ç'u gSaA çR;sd ç'u esapkj fod Yi (A),(B),(C) vkSj (D) gSa]ft uesalsd soy ,d lgh gSA

1. Assuming each salt to be 90% dissociated which of the following will have highest osmotic pressure ?
(A*) Decimolar Al2(SO4)3 (B) Decimolar BaCl2
(C) Decimolar Na2SO4
(D) A solution obtained by mixing equal volumes of (A), (B) and (C) and filtering.
;g ekud j fd izR;sd y o.k 90% fo;ksft r gksrk gSfuEu esalsfd ld k ijklj.k nkc vf/kd re gksxk \
(A*) Ms
l heksy jAl2(SO4)3 (B) Ms
l heks
y j BaCl2
(C) Ms
l heks
y j Na2SO4
(D) (A), (B) rFkk (C) d sleku vk;ru d ksfey kusij rFkk Nku d j ,d foy ;u izkIr gksrk gSA
2. The melting points of most of the solid substances increases with an increase of pressure acting on them.
However, ice melts at a temperature lower than its usual melting point , when the pressure increases. This
is because :
(A*) Ice is less dense than water (B) Pressure generates heat
(C) The bonds break under pressure (D) Ice is not a true solid

Corporate Office: CG Tower, A-46 & 52, IPIA, Near City Mall, Jhalawar Road, Kota (Raj.)-324005
Website : www.resonance.ac.in | E-mail : contact@resonance.ac.in
Toll Free : 1800 200 2244 | 1800 258 5555 | CIN: U80302RJ2007PLC024029
ADV-SCP - 51
Solution & Colligative Properties
vf/kd rj Bksl inkFkksZd sxy ukad bu ij d k;Zd jusoky snkc d sc<+usd slkFkc<+rsgSA fQ j Hkh]cQ Zt c nkc c<+k;kt krk
gSbld slkekU; xy ukad d h rqy uk esad e rki ij fi?ky rh gSA bld k d kj.k gSA
(A*) cQ Z
]t y d h rqy uk esad e ?kuRo oky k gksrk gSA (B) nkc m"ek mRiUu d jrk gSA
(C) nkc d svUrxZr cU/k VwVrsgSA (D) cQ ZlR; Bksl ughagSA
3. The phase diagrams for the pure solvent (solid lines) and the solution
(non-volatile solute, dashed line) are recorded below :

The quantity indicated by L in the figure is :

(A) p (B) Tf

(C*) Kbm (D) Kfm

'kq) foyk;d ¼xgjhjs[kk½rFkkfoy;u ¼vok"i'khy foys


;]Ms
'kjs
[kk½d sfy,
izkoLFkkvkjs[kuhpsjs[kkafd r fd ;kx;kgSA

fp=k esaL }kjk bafxr ek=kk gS%

(A) p (B) Tf

(C*) Kbm (D) Kfm

4. Available are 1L of 0.1 M NaCl and 2L of 0.2 M CaCl2 solutions. Using only these two solutions what
maximum volume of a solution can be prepared having [Cl–] = 0.34 M exactly. Both electrolytes are strong
(A*) 2.5 L (B) 2.4 L (C) 2.3 L (D) None of these
0.1 M NaCl d k 1 y hVj rFkk 0.2 M CaCl2 foy ;u d k 2 y hVj fy ;st krsgS
aA d soy bu nksuksafoy ;uksad k mi;ksx d j
fd ruk vf/kd re vk;ru cuk;k t k ld rk gSA ft lesa[Cl–] = 0.34 M gksA nksuksafo|qrvi?kV~; izcy gSaA
(A*) 2.5 L (B) 2.4 L (C) 2.3 L (D) buesalsd ksbZugha
5. A teacher one day pointed out to his students the peculiar fact that water is unique liquid which freezes
exactly at 00 C and boils exactly at 1000 C. He asked the students to find the correct statement based on this
fact :
(A) Water dissolves anything however sparingly the dissolution may be
(B) Water is a polar molecule
(C*) Boiling and freezing temperatures of water were used to define a temperature scale
(D) Liquid water is denser than ice
,d v/;kid us,d fnu viusfo|kFkhZ;ks ad ksfizd wfy;krF; d sckjses
acrk;kfd ikuhgh,d ek=knzo gSt ks0°C ij t erk
gSrFkk 100°C ij mcy rk gSA v/;kid usfo|kFkhZ;ksalsbl rF; ij vk/kkfjr lgh d Fku d ksigpkusd sfy , d gk&
(A) t y lHkh d k foy ; d jrk gSfQ j Hkh vka
f'kd ?kqy u'khy rk gksld rh gSA
(B) t y ,d /kzqoh; v.kqgSA
(C*) t y d sDoFkukad rFkk fgekad fcUnqrkieku iSekusd ksifjHkkf"kr d jusesami;ksxh gksrsgSA
(D) nz
o t y d k ?kuRo cQ Zd h rqy uk esavf/kd gksrk gSA
I II III
6. Three different ideal solutions (I, II, III) each containing total 10
moles of A & B in different composition are taken as shown in
figure and pressure over the solutions is gradually reduced. P1

Initially external pressure is same for all three solutions. P ext


At a particular external pressure (P1), II solution is found to have
XA = 0.4 & YA = 0.8
(liq composition)(Vap. composition)
Then select correct statement : Pure A Pure B
Corporate Office: CG Tower, A-46 & 52, IPIA, Near City Mall, Jhalawar Road, Kota (Raj.)-324005
Website : www.resonance.ac.in | E-mail : contact@resonance.ac.in
Toll Free : 1800 200 2244 | 1800 258 5555 | CIN: U80302RJ2007PLC024029
ADV-SCP - 52
Solution & Colligative Properties
(A) At the same external pressure, for solution I, XA < 0.4, YA < 0.8.
(B) At the same external pressure, for solution III, XA > 0.4, YA > 0.8.
(C*) For all three solutions at same external pressure (P1) liquid & vapour composition will be same
(D) None of these.
fp=k esn'kkZ, vuql kj fHkUu laxBu esA rFkk B d s10 eksy ;qDr rhu
I II III
fHkUu&fHkUu vkn'kZ] foy ;u (I, II, III) fy , t krsgSrFkk foy ;u d k nkc
fujUrj ?kVk;kt krkgSA
izkjfEHkd ckâ; nkc rhuksfoy ;uksd sfy , leku gSA P1
fd lh ckâ; nkc (P1) ij, II foy ;u ik;k t krk gSa
XA = 0.4 rFkk YA = 0.8 P ext
¼nzo laxBu½ ¼ok"i laxBu½
rc lgh d Fku d k p;u d hft ,&
(A) leku ckâ; nkc ij] foy ;u I d sfy , XA < 0.4, YA < 0.8. 'kq)A 'kq
)B
(B) leku ckâ; nkc ij]foy ;u III d sfy , XA > 0.4, YA > 0.8.
(C*) lHkh foy ;uksad sfy , leku ckâ; nkc (P1) ij nzo rFkk ok"i laxBu leku gksxkA
(D) bues alsd ksbZugha
7. For an ideal binary solution with PºA / PºB which relation between XA (mole fraction of A in liquid phase) and
YA (mole fraction of A in vapour phase) is correct , XB and YB are mole fraction of B in liquid and vapour phase
respectively : (Given : PºA > PºB)
(A) XA = YA (B) XA > YA
XA YA
(C*) X < Y (D) XA, YA, XB and YB cannot be correlated
B B

okysvkn'kZf}va
PºA / PºB xhfoy ;u d sfy, XA (nzo voLFkkesaA d heksy fHkUu) rFkkYA (ok"i voLFkkes aA d heksy fHkUu)
d se/; lgh lEcU/k gSA XB rFkkYB Ø e'k%nzo vkSj ok"i voLFkk esaB d h eksy fHkUu gSA (fn;k x;k gS: PºA > PºB)
(A) XA = YA (B) XA > YA
XA YA
(C*) X < Y (D) XA, YA, XB rFkkYB lEcfU/kr ughagksld rsgSA
B B

Section-2 : (One or More than one options correct Type)


This section contains 5 multiple choice questions. Each questions has four choices (A), (B), (C) and
(D) out of which ONE or MORE THAN ONE are correct.
[k.M+-2 : (,d ;k ,d l sv f/kd l gh fod Yi izd kj)
bl [k.M esa5 cgqfod Yi iz'u gSA izR;sd iz'u esapkj fod Yi (A), (B), (C) v kSj (D) gSa] ft uesal s,d ;k ,d l sv fèkd
l gh gSA

8. For KH, Henry’s constant, which are correct ?


(A*) KH is characteristic constant for a given gas–solvent system.
(B*) Higher is the value of KH, lower is solubility of gas for a given partial pressure of gas.
(C*) KH has temperature dependence.
(D*) KH increases with temperature.
gsujh fu;rkad KH d sfy , fuEu essalslgh d Fku gS@ gSa&
(A*) KH, ,d nh xbZXkSl &foy k;d ra=k d sfy , vfHky k{kf.kd fu;rkad gSA
(B*) KH d k eku vf/kd gksusd slkFk ,d fn;sx;sxSl d svkaf'kd nkc d sfy , xSl d h foy s;rk d e gkst krh gSA
(C*) KH rki ij fuHkZ
j d jrk gSA
(D*) rki d slkFk KH c<+ rk gSA

Corporate Office: CG Tower, A-46 & 52, IPIA, Near City Mall, Jhalawar Road, Kota (Raj.)-324005
Website : www.resonance.ac.in | E-mail : contact@resonance.ac.in
Toll Free : 1800 200 2244 | 1800 258 5555 | CIN: U80302RJ2007PLC024029
ADV-SCP - 53
Solution & Colligative Properties
9. The vapour pressure of ideal solution of benzene and
toluene is 550 torr at 80ºC then what would be correct
statement about same solution at 100ºC.
(A*) Vapour pressure of solution = 725 torr
(B*) at 725 torr pressure at 90ºC, no vapour form
2 9
(C) Composition of vapour is and
11 11
(D) Composition of liquid remain same at equilibrium condition at any temp.

;fn 80ºC ij cs Ut hu rFkkVkyq


bZ
u d svkn'kZfoy;u d kok"Iknkc
550 Vksj gSrksleku foy ;u d sfy , 100ºC ij fuEu esalslgh
d Fku gS@ gSa&

(A*) foy ;u d k ok"Ik nkc = 725 Vksj


(B*) 725 Vks
j nkc rFkk90ºC rki ij]ok"Ik ugh curh gSA
2 9
(C) ok"Ik d k la
?kBu rFkk gSA
11 11
(D) fd lh Hkh rki ij lkE; voLFkk esanzo d k la?kBu leku jgrk gSA
10. Select incorrect statement :
(A) Na+ and K+ ions are responsible for maintaining isotonic property inside and outside of the cell of organism.
(B) Aquatic species are more comfortable in lakes present at sea level in comparison to lakes present at high
altitude.
(C*) Solubility of N2 decreases, in presence of He when oxygen cylinder is utilised by Scuba divers.
(D*) The KH value of CO2 is higher than KH of N2.
xy r d Fkuksad k p;u fd ft ;sA
(A) Na+ rFkk K+ vk;u]t S c d ksf'kd k d h vkUrfjd rFkk ckg; d h leijkljh xq.k d sfy , mÙkjnk;h gksrsgSA
(B) t y h; izt krh;kWleqnzhLRkj ij mifLFkr >hy esamPp LFky ksaij mifLFkr >hy d hrqy ukes avf/kd lgt eglwl d jrs
gS
A
(C*) leq nzh xksrk[kksjksa}kjk iz;qDr fd ;st kusoky svkWDlht u fly s.Mj esaHe d h mifLFkr esaN2 d h foy s;rk ?kVrh gSA
(D*) CO2 d k KH eku]N2 ls avf/kd gksrkgSA
11. Following is false when in a volatile solvent A and a non volatile solute B is mixed (where symbols have their
usual meaning) :
t c ,d ok"i'khy foyk;d A esavok"i'khy foy s ; B feyk;kt krkgSA ¼;gk¡la dsrksd klkekU; vFkZgS½ rksfuEu eslsd kS
ulk
vlR; gS%
PA0  PB0 nB PA0  PB0 nB
(A*)  (B*) 
PA nA PA n A  nB

0 PA PA0  PA nB
(C) PA  1  X (D) =n
B PA A

12. 0.2 moles of A and 0.3 moles of B are taken in separate beakers and enclosed in chamber I. Another 0.2
moles of A and 0.3 moles of B are mixed in a beaker and enclosed in chamber II. At equilibrium. Which of the
following are not true. (A and B are volatile liquids and they form ideal solution on mixing)
(A*) The vapour pressure in chamber I is greater than vapour pressure in chamber II.
(B*) The vapour pressure in chamber I is less than vapour pressure in chamber II.
(C) The vapour pressure in both chambers are equal.
(D*) The vapour pressure in chamber II can not be determined.

Corporate Office: CG Tower, A-46 & 52, IPIA, Near City Mall, Jhalawar Road, Kota (Raj.)-324005
Website : www.resonance.ac.in | E-mail : contact@resonance.ac.in
Toll Free : 1800 200 2244 | 1800 258 5555 | CIN: U80302RJ2007PLC024029
ADV-SCP - 54
Solution & Colligative Properties
iF̀kd &iF̀kd chd j esaA d s0.2 eksy rFkk B d s0.3 eksy fy ;st krsgSrFkkd {k I esaj[kst krsgSrFkk ,d vU; chd j esaA
d s0.2 eksy rFkk B d s0.3 eksy fefJr fd ;st krsgS,oablsd {k II esaj[kk t krk gSA lkE; ij fuEu esalsd kSulk@d kSulsa
lgh ugh gS@ gSa& ¼A rFkkB ok"i'khy nzo gSrFkk B d k eksy j nzO;eku A lsvf/kd gS½
(A*) ,d I esok"i nkc]d {k II esok”i nkc lsvf/kd gks rk gSA
(B*) ,d I esok"i nkc]d {k II esok”i nkc lsd e gks rk gSA
(C) nksuksd {kksesok"i nkc leku gksrk gSA
(D*) d {k II esok"i nkc fu/kkZ
fjr ughafd ;kt k ld rk gSA

Section-3 : (One Integer Value Correct Type.)


This section contains 6 questions. Each question, when worked out will result in one integer from
0 to 9 (both inclusive)
[k.M+-3: (,d iw.kkZad eku l gh izd kj)
bl [k.M esa6 iz'u gSA izR;sd iz'u d ksgy d jusij ifjek.k 0 l s9 (nksuksa'kkfey ) d schp d k ,d iw.kk±d eku gksxkA

13. A very small amount of a non-volatile solute (non-associative, non-dissociative) is dissolved in 100 cm3 of a
solvent. At room temperature, vapour pressure of this solution is 98.7 mm of Hg while that of pure solvent is
100 mm of Hg. If the freezing temperature of this solution is 0.72 K lower than that of pure solvent, what is the
value of cryoscopic constant of solvent (in K Kg/mol) ? Round off your answer to the nearest whole number.
Report your answer as 0 (zero) if you find data insufficient.
Given : Molar mass of solvent = 78 g/mol.
,d vok”Ik’khy foy s; (val ;ksft r]vfo;ksft r) d hcgqr d e ek=kk100 cm3 d sfoy k;d esa/kqy kgqv kgSA d ejsd srki ij
bl foy ;u d k ok"i nkc 98.7 mm Hg gSA ;fn bl foy ;u d k fgekd 0.72 K ‘kq) foy k;d lsd e gksrksfoy k;d d k
Ø k;ksLd ksfid fu;rkad d keku (K Kg/mol esa) D;kgksxk? viukmÙkj fud VorhZiw.kkZad esansA ;fn fn;sx;svkWd Msiz;kZIr
ugh gksrksviuk mÙkj 0 (‘kqU;) esansA
fn;k gS: foy k;d d k eksy j nzO;eku = 78 g/mol.
Ans. 4

14. At 10ºC the osmotic pressure of urea solution is 500 mm. The solution is diluted and the temperature is
raised to 25ºC, when the osmotic pressure is found to be 105.3 mm. Determine extent of dilution.
10ºC ij ;w fj;kfoy ;u d kijklj.knkc 500 mm gSA bl foy ;u d ksruqd jusij rFkkrki 25ºC d jusij ijklj.knkc
105.3 mm gkst krk gS A ruqrk esaof̀) d h x.kuk fd ft ,A
Ans. 5

15. In aqueous solution of 1 × 10–3 molal Kx[Fe(CN)6] depression in freezing point is 7.2 × 10–3 K. Determine sum
of primary and secondary valency of complex (Kf of H2O = 1.8 K Kg/mole). (Assume that % ionisation of
complex is 100%)
,d 1 × 10–3 eks y y Kx[Fe(CN)6] d st y h; foy ;u esafgeka
d esavoueu 7.2 × 10–3 K gS
A la
dqy d hizkFkfed ,ao f}Ùkh;d
la;kst d rk d k ;ksx Kkr d hft ,A (H2O d kKf = 1.8 K Kg/eksy )
(eku y hft , fd la d qy d k vk;uu %, 100 % gS)
Ans. 9

16. Calculate molarity of final solution obtained by mixing I and II HNO3 solution.
I II
w
% 12.6 6.3
w
d (in g/mL) 1.5 1
Volume (in L) 5 5

Corporate Office: CG Tower, A-46 & 52, IPIA, Near City Mall, Jhalawar Road, Kota (Raj.)-324005
Website : www.resonance.ac.in | E-mail : contact@resonance.ac.in
Toll Free : 1800 200 2244 | 1800 258 5555 | CIN: U80302RJ2007PLC024029
ADV-SCP - 55
Solution & Colligative Properties
I rFkkII HNO3 foy ;u feJ.k }kjk iz
kIr vfUre foy ;u d h eksy jrk Kkr d hft ,A
I II
w
% 12.6 6.3
w
d (g/mL es
a) 1.5 1
vk;ru (L esa) 5 5
Ans. 2

17. 8 × 10–x moles of gas A is dissolved in 36 mL of water when pressure of gaseous mixture above water is 4
atm. Mole percentage of gas A in mixture is 25. Henry constant for gas A in water is 2.5 × 103 atm. Find x.
t c xSl h; feJ.k d k t y ij nkc 4 atm gksrc t y d s36 mL esaA d s8 × 10–x eksy ?kqy sgq;sgSaA xSl A d k feJ.k esa
eksy izfr'kr 25 gSA xSl A d k t y esagsujh fu;rkad 2.5 × 103 atm gSA x d k eku Kkr d hft , \
Ans. 4

18. A solution containing 0.1 g of a non volatile organic substance P(Molecular mass 100) in 100 g of benzene
raises the boiling point of benzene by 0.2ºC while a solution containing 0.1 g of another non volatile substance
Q in same amount of benzene raises the boiling point of benzene by 0.4ºC. If ratio of molecular masses of P
x
and Q is then. Find minimum value of x + y..
y
100 g cs
Ut huesa,d vok"i'khy d kcZfud inkFkZP ¼v.kqHkkj = 100½ d s0.1 g ;qDr foy ;u esacsUt hu d kDoFkukad 0.2ºC
c<+t krkgSt cfd csUt hu d hleku ek=kkesafd lhvU; vok"i'khy inkFkZQ d s0.1 g ;qDr foy ;u esacsUt hu d kDoFkukad
x
0.4ºC c<+t krk gSA ;fn P rFkk Q d k vkf.od nzO;eku d k vuqikr y gksrksx + y d k fuEure eku Kkr d hft ,A

x 2
Ans. =
y 1
Hence ¼vr%½x + y = 3

SECTION-4 : Comprehension Type (Only One options correct)


This section contains 1 paragraphs, each describing theory, experiments, data etc. 3 questions
relate to the paragraph. Each question has only one correct answer among the four given options
(A), (B), (C) and (D)
[k.M –4 : v uqPN sn çd kj (d soy ,d fod Yi l gh)
bl [k.M esafl) karksa]ç;ksxksavkSj vk¡d M+ksavkfn d ksn'kkZusoky s1 v uqPN sn gSA vuqPNsn lslacaf/kr rhu ç'u gSaA vuqPNsn esa
gj ç'u d spkj fod Yi (A), (B), (C) vkSj (D) gSa]ft uesalsd soy ,d gh l gh gSA

Paragraph For Questions 19 to 21


ç'u l a[;k 19 v kSj 21 d sfy , v uqPN sn
Comprehension #
A solution is made by mixing 1 mole benzene ( PB = 100 mm Hg) & 1 mole toluene ( PT = 40 mm Hg).Suppose,
initially the pressure over the solution is very high so that no vapour exist above the liquid. As we gradually
decrease the pressure, a point (bubble point) comes when we aross the bubble point curve & first bubble of
vapour starts forming (hence called bubble point curve). Now we have entered the vapour-liquid equilibrium
region. On further decreasing the pressure, a point (dew point) comes when we cross the dew point curve
then almost all the liquid has evaporated into vapour i.e. only the last drop of liquid (dew) remains. Beyond
this point no liquid exist in the system. Then answer the following questions :

Corporate Office: CG Tower, A-46 & 52, IPIA, Near City Mall, Jhalawar Road, Kota (Raj.)-324005
Website : www.resonance.ac.in | E-mail : contact@resonance.ac.in
Toll Free : 1800 200 2244 | 1800 258 5555 | CIN: U80302RJ2007PLC024029
ADV-SCP - 56
Solution & Colligative Properties
Comprehension #
1 eks
y csUt hu ( PB = 100 mm Hg) rFkk 1 eksy VkWy qbZu ( PT = 40 mm Hg) d ksfefJr d jd s,d foy ;u cuk;k t krk
gSaA ekuk izkjEHk esafoy ;u ij nkc cgqr vf/kd gSA ft llsnzo ij d ksbZHkh ok"i d k vfLrRo ughagSA t Sl s& t Sl sge
/khjs& /khjsnkc ?kVkrsgSrks,d fcUnq(cqy cqy k fcUnq) izkIr gksrk gSt c ge cqy cqy k fcUnqoØ d ksikj d jrsgSrFkk ok"i d k
izFke cqy cqy kcuukizkjEHkgkst krkgSA (blfy , ;g cqy cqy kfcUnqoØ d gy krkgSA). vc ge ok"i&nzo lkE; {ks=kesaizos'k
d jrsgSA vkxsnkc ?kVkusij ,d fcUnq(vks l fcUnq) izkIr gksrkgSt c ge vks l fcUnqoØ d ksikj d jrsgSA ckn esaizk;%leLr
nzo ok"i esaok"ihd r̀ gkst krk gSA vFkkZr~d soy nzo d h vfUre cwan (vksl ) 'ks"k jgrh gSabl fcUnqd svy kok fud k; esad ksbZ
Hkh nzo d k vfLrRo ughagksrk gSA rc fuEu iz'uksad smÙkj nhft ,A
19. If the pressure over the mixture at 300 K is reduced, at what pressure does the first bubble form :
;fn 300 K ij feJ.k ij nkc vipf;r gksrk gS]rksd kSulsnkc ij loZizFke cqy cqy k cusxk %
(A) 140 mm Hg (B) 90 mm Hg (C) 65 mm Hg (D*) 70 mm Hg
20. What is the composition of first bubble formed ?
fufeZr loZizFke cqy cqy sd k la?kBu D;k gS\
(A*) YA = 2/7, YB = 5/7 (B) YA = 3/7 , YB = 4/7
(C) YA = 1/7 , YB = 6/7 (D) none of these (bues alsd ksbZugha)
21. What will be the pressure when 1 mole of mixture has been vapourised ?
nkc D;k gksxk]t c 1 eksy feJ.k ok"ihd r̀ gksrk gS\
(A) 70 mm Hg (B*) 63.25 mm Hg (C) 100 mm Hg (D) 40 mm Hg

SECTION-5 : Matching List Type (Only One options correct)


This section contains 1 questions, each having two matching lists. Choices for the correct combination
of elements from List-I and List-II are given as options (A), (B), (C) and (D) out of which one is correct
[k.M -5 : l qesy u l wp h izd kj ¼d soy ,d fod Yi l gh½
bl [k.M esa1 cgqfod Yi iz'u gSA izR;sd iz'u esankslqesy u lwfp;k¡gSA lwfp;ksad sfy , d wV d sfod Yi (A), (B), (C) vkSj
(D) gSaft uesalsd soy ,d lgh gSA

22. Match each List-I with an appropriate pair of characteristics from List-II and select the correct answer
using the code given below the lists.
0.1 mol of each solute in the list-I are dissolved in 10 mole water separately.

List-I List-II
P. AlCl3 if  = 0.8 1. i = 3.4
Q. BaCl2 if  = 0.9 2. has minimum osmotic pressure among the given solutions.
R. Na3PO4 if  = 0.9 3. has minimum freezing point among the given solutions.
S. K4[Fe(CN)6] if  = 0.7 4. has RLVP = 37/1037.
l wp h-I d ksl wp h-II d sv fHky {k.kksad smfpr
;qXe d sl kFk l qesfy r d hft , rFkk l wp h d suhpsfn;sx;sd ksM ksad k iz;ksx
d jrsgq, l gh mÙkj d k p;u d hft ,A
l wp h-I d sizR;sd foy s; d s0.1 eksy d ksiF̀kd : i l s10 eksy t y esa?kksy k t krk gSA

lwp h-I l wp h-II


P. AlCl3 ;fn  = 0.8 1. i = 3.4
Q. BaCl2 ;fn  = 0.9 2. fn;sx;sfoy ;uksaesals;g U;wure ijklj.k nkc j[krk gSA
R. Na3PO4 ;fn  = 0.9 3. fn;sx;sfoy ;uksaesals;g U;wure fgekad j[krk gSA
S. K4[Fe(CN)6] ;fn  = 0.7 4. RLVP = 37/1037 j[krk gS A
Code (d ks
M) :
P Q R S
(A) 4 2 3 1
(B*) 1 2 4 3
(C) 2 1 3 4
(D) 1 2 3 4
Corporate Office: CG Tower, A-46 & 52, IPIA, Near City Mall, Jhalawar Road, Kota (Raj.)-324005
Website : www.resonance.ac.in | E-mail : contact@resonance.ac.in
Toll Free : 1800 200 2244 | 1800 258 5555 | CIN: U80302RJ2007PLC024029
ADV-SCP - 57
Solution & Colligative Properties

PART - III : OLYMPIAD (PREVIOUS YEARS)

Hkkx - III : OLYMPIAD ¼fiN y so"kksZ½ d siz'u


STAGE - I (NATIONAL STANDARD EXAMINATION IN CHEMISTRY (NSEC))
1. Solutions having the same osmotic pressure are called [NSEC-2000]
(A*) isotonic solutions (B) molar solutions
(C) hypotonic solutions (D) ideal solutions
leku ijklj.k nkc j[kusoky k foy ;u d gy krk gS& [NSEC-2000]
(A*) vkblksVkW
fud foy ;u (B) eks
y foy ;u
(C) gkbiks
Vks
fud foy ;u (D) vkn'kZfoy ;u

2. A colligative property of a solution depends on the [NSEC-2000]


(A) arrangement of atoms in solute molecule. (B) total number of molecules of solute and solvent
(C*) number of molecules of solute in solution. (D) mass of the solute molecules.
foy ;u d h v.kql a[; xq.k/keZfd l ij fuHkZj d jrk gS& [NSEC-2000]
(A) foy s
; v.kqv ksaesaijek.kqv ksad hO;oLFkk ij (B) foy s
; rFkk foy k;d d sd qy v.kqv ksad h la[;k ij
(C*) foy ;u esafoy s; d sv.kqv ksad h la[;k ij (D) foy s
; d sv.kqv ksad h la[;k nzO;eku ij
3. When 0.6 gm of urea dissolved in 100g of water, the water will boil at (Kb for water = 0.52 kJ. mol–1 and normal
boiling point of water = 100ºC) : [NSEC-2001]
t c 0.6 gm ;qfj;k d ks100g t y esafey k;k t krk gS]rkst c t y mcy rk gS& (t y d sKb = 0.52 kJ. mol–1 rFkk t y
d k lkekU; DoFkukad = 100ºC) : [NSEC-2001]
(A*) 373.052 K (B) 273.52 K (C) 372.48 K (D) 273.052 K

4. The osmotic pressure of a solution is given by the equation : [NSEC-2001]


,d foy ;u d sijklj.k nkc d ksfuEu lehd j.k }kjk fn;k t k ld rk gS& [NSEC-2001]
CR  CT R
(A) p = (B*) = RT (C) p = (D) p = C.
T C R T

5. The relative lowering of vapour pressure is equal to the mole fraction of the solute. This is the statement of:
[NSEC-2001]
(A*) Raoult’s law (B) Boyle’s law
(C) Osmotic pressure law (D) Graham’s law
ok"i nkc esavkisf{kd voueu foy s; d h eksy fHkUu d scjkcj gksrk gSA ;g d Fku fd ld k gS& [NSEC-2001]
(A*) jkmYV d k fu;e (B) ckW
;y d k fu;e (C) ijklj.k nkc d k fu;e (D) xzkge d k fu;e
6. Azeotropes are : [NSEC-2002]
(A*) liquid mixtures which distil unchanged in composition
(B) liquids which can mix with each other in all proportions
(C) solids which form solid solutions of definite compositions
(D) gases which can be separated.
fLFkjDokFkhgS& [NSEC-2002]
(A*) nzo feJ.k t ksla| Vu esavklfor vifjofrZr gksrsgS
(B) nzo t kslHkh vuqikrksesa,d &nwl jsd slkFk fefJr gksrsgS
(C) Bksl t ksfuf'pr la| Vu d sBksl foy ;u lscursgSA
(D) xSl sft Ugsvy x vy x fd ;k t k ld rk gSA
7. Swimming for a long time in salt water makes the skin of one’s finger tips wrinkled. Which one of the
following properties is responsible for this observation ? [NSEC-2002]
(A*) osmosis (B) dialysis (C) electrodialysis (D) coagulation.

Corporate Office: CG Tower, A-46 & 52, IPIA, Near City Mall, Jhalawar Road, Kota (Raj.)-324005
Website : www.resonance.ac.in | E-mail : contact@resonance.ac.in
Toll Free : 1800 200 2244 | 1800 258 5555 | CIN: U80302RJ2007PLC024029
ADV-SCP - 58
Solution & Colligative Properties
yo.kt y es ayEcsle; rd rS
juslsRopkij vxq
fy;kd sleku >q
a fjZ
;kcu t krhgSbl i;Zos
{k.kd sfy, fuEu es
alsd kS
u&lk
xq.k/keZmÙkjnk;h gS& [NSEC-2002]
(A*) ijklj.k (B) viks
gu (C) fo|q
r viksgu (D) Ld a
nu
8. A 2% solution of glucose has the same elevation in the boiling point as that of a 5% solution of a non-volatile
solute. The molar mass of the solute is : [NSEC-2003]
Xy wd ksl d s2% foy ;u }kjk fgekad esamrukvoueu n'kkZ;kt krkgSft ruk,d vok"i'khy foy s; d s5% foy ;u }kjkA
foy s; d kv.kqHkkj gksxk& [NSEC-2003]
(A) 180 (B*) 450 (C) 72 (D) 18
9. A 1.0 molal solution with the lowest freezing point is that of : [NSEC-2004]
fuEu esalsfd l Lih'kht d k 1.0 eksy y foy ;u d k fgekad fcUnqU;wure gksxk % [NSEC-2004]
(A*) FeCI3 (B) HCI (C) KCI (D) MgCI2.
10. From among the following, the aqueous solution which has the highest freezing point depression is :
[NSEC-2005]
(A*) 0.1 M Sr(NO3)2 (B) 0.1 M KCI (C) 0.1 M HNO3 (D) 0.1 M glucose.
fuEu esalsog t y h; foy ;u ft ld sfgekad esavoueu mPpre gksrk gS& [NSEC-2005]
(A*) 0.1 M Sr(NO3)2 (B) 0.1 M KCI (C) 0.1 M HNO3 (D) 0.1 M Xy w
d kst
11. In chemical industries, the preferred method of purification of liquids is : [NSEC-2006]
(A) differential extraction (B*) fractional distillation
(C) chromatography (D) leaching.
jlk;u m|ksxksesanzo d s'kqf) d j.k d sfy , loksZÙke fo/kh gS& [NSEC-2006]
(A) fu"d "kZ
.k (B*) iz
Hkkt hvklou
(C) Ø ks
es
VksxkzQ h (D) fu{kkyu

12. The solubility of a gas in a liquid is driectly proportional to the partial pressure of the gas over the solution.
This statement is known as: [NSEC-2007]
(A) Raoult’s law (B*) Henry’s law
(C) Boyle’s law (D) Charles’ and Gay Lussac’s Law
nzo esaxSl d h foy s;rkfoy ;u d sÅ ij xSl d svkaf'kd nkc d slekuqikrh gksrh gS;g d Fku fd ld kgS& [NSEC-2007]
(A) jkmYV fu;e (B*) gsujh fu;e
(C) ck;y fu;e (D) pkYlZo xs y wl kd fu;e
13. Which of the following is not a colligative property ? [NSEC-2007]
(A*) solubility. (B) vapor pressure lowering.
(C) boiling point elevation. (D) osmotic pressure.
fuEu esalsd kSulh v.kql a[;d xq.k/keZughagS& [NSEC-2007]
(A*) foy s
;rk (B) ok"i nkc es
avoueu
(C) DoFkukad esamUu;u (D) ijklj.knkc

14. Which of the following has the lowest freezing point and the highest boiling point? [NSEC-2007]
(A*) 1.5 m magnesium phosphate (B) 1.0 m sodium chloride
(C) 1.5 m aluminum nitrate (D) 1.5 m calcium chloride
fuEu esalsfd ld k fgekad fcUnqlclsd e o DoFkukad fcUnqlclsvf/kd gS\ [NSEC-2007]
(A*) 1.5 m eSxuhf'k;e Q kLQ sV (B) 1.0 m lksfM;e Dy ksjkbM
(C) 1.5 m ,Y;qfefu;e ukbVªsV (D) 1.5 m d s
fY'k;e Dy ksjkbM
15. A solution of urea was found to be isotonic with a solution of salt XY of molecular weight 74.6. If 0.15 moles
of urea are dissolved in a certain volume V mL of the isotonic solution, the amount of salt in the solution will
be : [NSEC-2008]
y o.kXY (v.kqHkkj = 74.6) d s,d foy ;u d slkFk;wfj;k d k,d foy ;u leijkljhik;k x;kA ;fn leijkljh foy ;u
d s,d fuf'pr vk;ru V mL esa0.15 eksy ;wfj;k ?kqy k gqv k gks]rksfoy ;u esamifLFkr y o.k d h ek=kk D;k gksxh \
[NSEC-2008]
(A) 22.4g (B*) 5.6 g (C) 11.2 g (D) 7.46 g

Corporate Office: CG Tower, A-46 & 52, IPIA, Near City Mall, Jhalawar Road, Kota (Raj.)-324005
Website : www.resonance.ac.in | E-mail : contact@resonance.ac.in
Toll Free : 1800 200 2244 | 1800 258 5555 | CIN: U80302RJ2007PLC024029
ADV-SCP - 59
Solution & Colligative Properties
16. The desalination of sea water involves the phenomenon of : [NSEC-2008]
(A) Sedimentation (B) Distillation (C) Precipitation (D*) Reverse osmosis
leqnzh t y d k foy o.khd j.k fuEu esalsfd l ifj?kVuk }kjk fd ;k t krk gS\ [NSEC-2008]
(A) ry NVu (B) vklou (C) vo{ks
i.k (D*) iz
frykse ijklj.k
17. According to this phase diagram, which phases can exist at pressures lower than
the triple point pressure ? [NSEC-2008]
(A) gas only
(B*) solid and gas only
(C) liquid only
(D) solid and liquid only
çnf'kZ
r izkoLFkkoØ vuq
l kj d kuSlhiz
koLFkkfr;Z
d fcUnqnkc lsd e nkc ij vfLrRo es
agksld rh
gS [NSEC-2008]
(A) d s
oy xSl
(B*) d s
oy Bksl ,oaxSl
(C) d s
oy nzo
(D) d s
oy Bksl ,oanzo
18. For a dilute solution, Raoult's law states that : [NSEC-2009]
(A) the lowering of vapour pressure is equal to the mole fraction of the solute.
(B*) the relative lowering of vapour pressure is equal to the mole fraction of the solute
(C) the vapour pressure of solution is equal to the mole fraction of solution.
(D) the relative lowering of vapour pressure is proportional to the amount of solute in the solution.
jkmYV fu;e ls] ,d ruqfoy ;u d sfy , % [NSEC-2009]
(A) ok"i nkc d k voueu foy s; d seksy fHkUu d scjkcj gksrk gSA
(B*) ok"i nkc d k vkis
f{kd voueu foy s; d seksy fHkUu d scjkcj gksrk gSA
(C) foy ;u d k ok"i nkc foy ;u d seksy fHkUu d scjkcj gksrk gSA
(D) ok"i nkc d k vkis
f{kd voueu foy ;u easfoy s; d h ek=kk d slekuqikrh gksrk gSA
19. A mixture of two liquids which boils without change in composition is called [NSEC-2010]
(A) Stable mix ture (B) Binary liquid mixture
(C*) Azeotropic mixture (D) Zerotropic mixture
nksnzoksad k og feJ.k t ksla?kVu esafcuk ifjorZu fd ;smcy rk gS]d gy krk gS& [NSEC-2010]
(A) LFkk;hfeJ.k (B) f} nzo feJ.k
(C*) fLFkjDokFkhfeJ.k (D) ft jks

kW
ihd feJ.k
20. The aqueous solution having osmotic pressure nearest to that of an equimolar solution of K4[Fe(CN)6] is
[NSEC-2010]
fuEu esalsfd ld st y h; foy ;u d k ijklj.k nkc K4[Fe(CN)6] d sleeksy j foy ;u d sleku gksxk& [NSEC-2010]
(A) K2SO4 (B) Na3PO4 (C*) Al2(SO4)3 (D) C6H12O6

21. The elevation in boling point of a solution containing 13.44 g of CuCl2 in 1 kg of water is :
(Kb = 0.52 K kg mol–1) [NSEC-2011]
1 kg t y esa13.44 g CuCl2 ;qDr ,d foy ;u d sDoFkukad esamUu;u gksxk&
(Kb = 0.52 K kg mol–1) [NSEC-2011]
(A) 0.05 (B) 0.10 (C*) 0.16 (D) 0.21

22. The freezing point of a solution containing 8.1 g of HBr in 100 g of water, assuming the acid to be 90% ionized
is [H = 1, Br = 80, Kf for water = 1.86 K kg mol–1] [NSEC-2011]
100 t y es a8.1 g HBr ;qDr ,d foy ;u d k fgekad D;k gksxk \ ;g ekursgq, fd vEy 90% vk;fur gksrk gS&
[H = 1, Br = 80, t y d sfy , Kf = 1.86 K kg mol–1] [NSEC-2011]
(A) 0.85ºC (B*) –3.53ºC (C) 0ºC (D) –0.35ºC

Corporate Office: CG Tower, A-46 & 52, IPIA, Near City Mall, Jhalawar Road, Kota (Raj.)-324005
Website : www.resonance.ac.in | E-mail : contact@resonance.ac.in
Toll Free : 1800 200 2244 | 1800 258 5555 | CIN: U80302RJ2007PLC024029
ADV-SCP - 60
Solution & Colligative Properties
23. Which of the following observation indicates colligative properties ? [NSEC-2012]
I. A 0.5 M NaBr solution has a higher vapour pressure than 0.5 M BaCl2.
II. A 0.5 M NaOH solution freezes at a lower temperature than pure water.
III. Pure water freezes at a higher temperature than pure ethanol.
(A) Only I (B) Only II (C) Only III (D*) I and II
fuEu esalsd kSulk izs{k.k v.kql a[; xq.k/keZd ksn'kkZrk gS& [NSEC-2012]
I. 0.5 M NaBr foy ;u d k ok"i nkc 0.5 M BaCl2 foy ;u d h rq y uk esamPp gksrk gSA
II. 0.5 M NaOH foy ;u 'kq ) t y d h rqy uk esad e rki ij t erk gSA
III. 'kq
) t y ]'kq) ,sFksukWy d h rqy uk esamPp rki ij t erk gSA
(A) d soy I (B) d s
oy II (C) d s
oy III (D*) I d s
oy II
24. Osmotic pressure of a 2 % w/v solution of glucose is same as 5% w/v solution of a nonvolatile non-electrolyte
solute. The molar mass of the solute is : [NSEC-2014]
,d 2 % Hkkj@vk;ru d sXy wd ks
l d sfoy;u d kijklj.knkc],d vok"i'khy vfo|q r vi?kV~ ; foy s
; d s5% Hkkj@vk;ru
foy ;u d k cjkcj gSA foy s; d k eksy j nzO;eku gS% [NSEC-2014]
(A) 180 (B*) 450 (C) 72 (D) 45
25. The colligative property used in the determination of molar mass of a polymer is : [NSEC-2014]
(A) lowering of the vapour pressure
(B) elevation in the boiling point
(C) depression in the freezing point
(D*) osmotic pressure.
,d cgqy d d seksy j nzO;eku Kkr d jusgsrqmi;qDr v.kql a[;d xq.k/keZgS% [NSEC-2014]
(A) ok"i nkc es
avoueu (B) DoFkukad esamUu;u
(C) fgeka
d esavoueu (D*) ijklj.kes ankc

STAGE - II (INDIAN NATIONAL CHEMISTRY OLYMPIAD (INChO))


Problem 1 16 Marks
Sea Water
In polar regions, life exists below the frozen crust of ice. Only the layer of the ocean is frozen and the ice
floats on water.
Problem 1 16 Marks
l eqnzh t y
/kqzoh; {ks=kksaesat hou d k vfLrRo cQ Zd h t eh gqbZiiZVh (frozen crust) d suhpsgksrk gSA d soy egklkxj d h ijr t erh
gSrFkk t y ij cQ ZrSjrh jgrh gSA
1.1 The density of ice differs from that of water because [InChO-2007, 4 M]
(A) ice is a solid and water is a liquid
(B) hydrogen bonding exists only in water
(C) ice has an open cage-like structure
(D) ice is covalently bonded but water involves ionic bonding
cQ Zd k ?kuRo t y lsfHkUu gksrk gS]D;ksafd [InChO-2007, 4 M]
(A) cQ ZBks l gSrFkk t y nzo gSA
(B) gkbMªkst u ca/ku d k vfLrRo d soy t y esagksrk gSA
(C) cQ Zd h [kqy sfiat jsd sleku lajpuk gksrh gSA
(D) cQ Zlgla ;kst d : i lscaf/kr gksrh gSy sfd u t y esavk;fud ca/ku gksrk gSA
1.2 Sea water contains many salts such as halides and sulphates of sodium, potassium, magnesium, calcium
etc. Hence, sea water freezes at a temperture lower than the freezing point of pure water. The freezing point
of a typical sample of sea water is –1.94ºC at 1atm . Calculate the boiling point of sea water at 1 atm.
Kf (water) = 1.86 K molal –1, Kb (water) = 0.51 K molal–1
Boiling point of pure water at 1 atm is 373.1 K. [InChO-2007, 2 M]

Corporate Office: CG Tower, A-46 & 52, IPIA, Near City Mall, Jhalawar Road, Kota (Raj.)-324005
Website : www.resonance.ac.in | E-mail : contact@resonance.ac.in
Toll Free : 1800 200 2244 | 1800 258 5555 | CIN: U80302RJ2007PLC024029
ADV-SCP - 61
Solution & Colligative Properties
leq nz
ht y esavus d gS
y kbMks
arFkklksfM;e]iks Vs
f'k;e]eSfXuf'k;e]d S
fY'k;e bR;kfn d slYQ sVksad sleku yo.kgksrsgS
A vr%
leq nz
ht y ]'kq) t y d sfgekad lsd e rki ij t erkgS A leq nzht y d s,d iz
k: fid izkn'kZd kfgekad 1 atm ij –1.94ºC
gksrk gSA 1atm ij leqnzh t y d k DoFkukad ifjd fy r d hft ,A
Kf (t y ) = 1.86 K molal –1, Kb (t y ) = 0.51 K molal–1
1 atm ij 'kq ) t y d k DoFkukad 373.1 K gSA [InChO-2007, 2 M]

1.3 The van’t Hoff factor i is a measure of association. [InChO-2007, 2 M]


no.of particlesafter association
i = no.of particlesbeforeassociation

(A) van’ Hoff factor for 0.1 M aqueous sodium chloride is 1.87. The van’t Hoff factor for 0.1 M magnesisum
sulphate is expected to be :
(a) 1.87 (b) 1.25 (c) 1.92
(B) In magnesium sulphate the ionic interaction :
(a) is same as that observed in NaCI (b) is weaker than that observed in NaCI
(c) is stronger than obesrved in NaCI (d) is not responsible for the i value

okUVgkWQ xq.kkad i laxq.ku d k eki gSA [InChO-2007, 2 M]

l axq.ku d si'pkr ~d .kksad h l a[; k


i= l axq.ku d siwoZd .kksad h l a[; k
(A) 0.1 M t y h; lksfM;e Dy ksjkbM d k okUVgkWQ xq.kkad 1.87 gSA 0.1 M eSfXuf'k;e lYQ sV d sfy , okUVgkWQ xq.kkad
vkisf{kd : i lsgks
xkA
(a) 1.87 (b) 1.25 (c) 1.92
(B) eSfXuf'k;e lYQ sV esavk;fud vU;ksU; fØ ;k %
(a) NaCI esaizsf{kr vk;fud vU;ksU; d sleku gksrhgSA
(b) NaCI esaizsf{kr vk;fud vU;ksU; fØ ;klsnqcZy gksrhgSA
(c) NaCI esaizsf{kr vk;fud vU;ksU; fØ ;klsizcy gksrhgSA
(d) ;g i eku d sfy , ft Ees nkj ughagSA
1.4 Sea water is a rich source of halides. However, the concentration of iodide is less than that of the other
halides, as the iodide ions are absoreded by the sea weeds and stored as iodine. Iodine can be extracted
from sea weeds. The distribution of iodine between two immiscible solvents is given by the distribution ratio
(D), which is constant a given temperature. [InChO-2007, 2 M]
Conc of I2 in solvent1
D = Conc of I in solvent 2
2
(A) One litre of an aqueous solution containing 0.127 g of iodine was shaken with 10 ml of CCI4. Certain
amount of iodine was extracted into the organic layer.The organic layer was found to be 0.0465 M with
respect to iodine. If another extraction was performed with 10mL of CCI4 what mass of iodine will be left in
water layer? Atomic mass of is 127.
(B) The solubility of iodine in aqueous KI is far higher than that in water due to the formation of the triiodide
ion. Give the hybridization of the central atom of the triiodide ion and show geometry of the bonded pairs of
electrons.
leqnzht y gSy kbMksd k/kuhL=kks
r gSA ;|fi vk;ksM kbM d hlkUnzrkvU; gS y kbMks
ad hvis{kkd e gks
rhgSA D;ksa
fd vk;ks
M kbM
vk;u leqnzh [kjirokj (sea weeds) }kjk vo'kksf"kr gksrsgSrFkk vk;ksM hu d s: i esalaxzfgr gksrsgSA vk;ksM hu leqnzh
[kjirokj lsfu"d f"kZ
r gksld rhgS A nksvfeJ.kh; foyk;d kasd se/; vk;ks
Mhu d kforj.k]forj.kvuq ikr (D) }kjkfn;kt krk
gS]t ksfn, x;srki ij fu;r gksrk gSA [InChO-2007, 2 M]
foy k; d 1 esaI2 d h l kUnzrk
D = foy k; d 2 esaI2 d h l kUnzrk

Corporate Office: CG Tower, A-46 & 52, IPIA, Near City Mall, Jhalawar Road, Kota (Raj.)-324005
Website : www.resonance.ac.in | E-mail : contact@resonance.ac.in
Toll Free : 1800 200 2244 | 1800 258 5555 | CIN: U80302RJ2007PLC024029
ADV-SCP - 62
Solution & Colligative Properties
(A) 0.127 g vk;ks
Mhu oky s,d y hVj t y h; foy ;u d ks10 ml CCI4 d slkFkfgy k;kt krkgSA vk;ksM hu d hfuf'pr ek=kk
d ks,d d kcZfud ijr esafu"d f"kZr fd ;kx;kA d kcZfud ijr vk;ksM hu d slaxr 0.0465 M ik;hx;hA ;fn vU; fu"d "kZ.k
d ks10 mL CCI4 d slkFk mipkfjr fd ;k x;k]rkst y h; ijr esavk;ksM hu d k fd ruk nzO;eku 'ks"k jgsxkA I d k ijek.oh;
nzO;eku 127 gSA
(B) t y h; KI es
avk;ksMhu d hfoy s ;rkVª
kbZv k;ksMkbM vk;u d sfuekZ.kd sd kj.kt y esavk;ks M hu d hfoys;rkd hvis{kkvf/
kd mPp gksrh gSA VªkbZv k;ksM kbM vk;u d sd sUnzh; ijek.kqd k lad j.k crkb, rFkk by sDVªkWuksad scaf/kr ;qXeksad h T;kferh
n'kkbZ,A
Ans. (A) 6.223 × 10–4 g ; (B) sp3d, linear geometry
Ans. (A) 6.223 × 10–4 g ; (B) sp3d, js[kh; T;kferh
1.5 In an attempt to quantify sea water containing sodium, magnesium, calcium and potassium ions as the only
cations, the total cation content of the sea water was estimated. 10ml of a sample of sea water was diluted
to 1L. The cations were replaced by hydrogen ions and the salt were converted to their corresponding mineral
acids using the ion exchange resin zeolite 225. The exchange of ions increases with increase in the valency
as well as the size of cations exchanged. 100 mL aliquot of the diluted sample was passed through the
cation exchanger zeolite 225 and the sample obtained from the exchanger required 32 mL of 0.05 M NaOH
for exact neutralization. Molar mass of calcium carbonate = 100 .
One may obtain total cation concentration in terms that of Ca2+ or any of its salts .
(A) Express the total salt content sea water in terms of g L–1 of CaCO3. [InChO-2007, 1.5 M]
(B) If the cations retained in the resin are washed using dilute HCl, which would be the first to come out of the
ion exchange resin ? [InChO-2007, 0.5 M]
(C) Total exchange capacity of the resin zeolite is 5 eq of the solute per gram of the resin. If 4 g of resin was
used in the determination cited above, what % of active sites of the resion undergoes ion exchange ?
[InChO-2007, 1 M]
d soy /kuk;uksad s: i lksfM;e]eSfXuf'k;e]d SfY'k;e rFkkiks Vsf'k;e vk;uksaoky sleqnzht y d ksizekf.kr d jusd hd ksf'k'k
esaleq nzht y d sd qy /kuk;u ?kVd d kvkd y u fd ;kx;kA leqnz ht y d s10 ml izkn'kZd ks1L rd ruqfd ;kx;kA /kuk;u
gkbMªkst u vk;uksa}kjk izfrLFkkfir gksrsgSrFkk y o.k vk;u fofue; jst hu ft ;ksy kbV 225 d k mi;ksx d jd sbld slaxr
[kfut vEy ksaesaifjofrZr gkst krkgSA vk;uksad kfofue; la ;kst d rkc<+usd slkFk&lkFkfofue; gq, /kuk;uks ad svkd kj esa
of̀) d slkFkc<+rkgSA ruqizkn'kZd s100 mL nzo d ks/kuk;u fofue;d (exchanger) ft ;ksy kbV 225 esalsizokfgr fd ;k
x;krFkkfofue; lsiz kIr iz
kn'kZd ksiw
.kZmnklhuhd j.kd sfy , 0.05 M NaOH d s32 ml d hvko';d rkgks rhgSA d SfY'k;e
d kcksZusV d k eksy j nzO;eku = 100 .
d ksbZHkh d SfY'k;e vk;u vFkok bld sfd lh Hkh y o.k d s: i esad qy /kuk;u lkUnzrk izkIr d j ld rk gSA
(A) CaCO3 d sgL–1 d sinks aesaleqnzh t y d sd qy y o.k ?kVd d ksO;Dr d hft ,A [InChO-2007, 1.5 M]
(B) ;fn js ft u esaj[ks/kuk;uksad ksruqHCl d k iz;ksx d jd s/kks;kt krk gS]rksvk;u fofue; jsft u esad kSu loZizFke ckgj
vk;sxk \ [InChO-2007, 0.5 M]
(C) js ft u ft ;ksy kbV d hd qy fofue; {kerkjs ft u d sizfr xzke foys; d k5 rq
Y;kad gSA ;fn 4 g jsft u d ksÅ ij mYy sf[kr
fu/kkZj.k esaiz;qDr fd ;k x;k gSrksjsft u d slfØ ; LFky ksa(site) d k fd ruk %, vk;u fofue; nsrk gS\
[InChO-2007, 1 M]
Ans. (A) 0.8 g/L, (B) Potassium, (C) 8 × 10–3

1.6 At a certain place sea water is found to be 0.6 M with respect to NaCl and 0.05M with respect ot MgCl2. It is
concentrated by a manufacturer of common salt to get 4.8 M solution with respect to NaCl and he wants to
precipitate pure common salt by passing HCl gas. What is the minimum volume of HCl in L at STP that must
be passed to initiate the salting out of NaCl from IL of the solution ? (Ksp of NaCl = 36)
[InChO-2007, 3 M]
,d fuf'pr LFkku ij leqnzht y NaCl d slaxr 0.6 M rFkkMgCl2 d slaxr 0.05 M ik;kt krkgSrks;g NaCl d slaxr
4.8 M foy ;u izkIr d jusd slkekU; y o.k d sfuekZ.k}kjk lkfUnzr gksrk gSA rFkk og HCl xSl izokfgr d jd s'kq) lkekU;
yo.kd ksvo{ksfir d jukpkgrkgS A STP ij y hVj es aHCl d kU;wure vk;ru D;kgks xk]t ks1 L foy;u lsNaCl d kyo.k
cukusd sfy , izokfgr d juk pkfg, \ (NaCl d k Ksp = 36) [InChO-2007, 3 M]
Ans. 60.48 L

Corporate Office: CG Tower, A-46 & 52, IPIA, Near City Mall, Jhalawar Road, Kota (Raj.)-324005
Website : www.resonance.ac.in | E-mail : contact@resonance.ac.in
Toll Free : 1800 200 2244 | 1800 258 5555 | CIN: U80302RJ2007PLC024029
ADV-SCP - 63
Solution & Colligative Properties
Problem 2 14 marks
Phase equilibrium
Phase diagram of a one component system (s), is shown below. Answer the following questions with the help
of this diagram.
izkoLFkk lkE; (Phase equilibrium)

,d y ?kVd fud k; (S) d k izkoLFkk fp=k uhpsfn;k x;k gSA fuEu iz'uksad smÙkj fp=k d h lgk;rk lsnhft ,A

72.9 B •
X5
C
76.0
P/atm

• • • •
X1 X2 X3 X4

5.2
O
1.0
A

194.7 216.8 298.15 304.2 T/K

2.1 What kind of phase change will take place if solid S is kept in the open under normal condition ?
[InChO-2008, 1 M]
(A*) Sublimation (B) Melting (C) Evaporation (D) No change
;fn Bksl S d ks[kqy sesalkekU; fLFkfr;ksaesaj[kk x;k gS]rksizkoLFkk esafd l izd kj d k ifjorZu gksxk \
[InChO-2008, 1 M]
(A*) Å /oZ
ikru (B) xy u (C) ok"ihd j.k (D) d ks
bZifjorZu ugha
2.2 Under which condition, are all the three phases of the system S in equilibrium ?
(A) T > 304.2 K and P > 72.9 atmosphere (B*) T = 216.8 K and P = 5.2 atmosphere
(C) T > 304.2 K (D) P > 72.9 atmosphere
fd u fLFkfr;ksaesalHkhrhuksaizkoLFkk;safud k; S esalkE;koLFkkesagksxh\
(A) T > 304.2 K ,oaP > 72.9 ok;q
e.My h; (B*) T = 216.8 K ,oaP = 5.2 ok;q
e.My h;
(C) T > 304.2 K (D) P > 72.9 ok;q
e.My h;
2.3 Temperature of the system at X1 is increased at constant pressure to reach X4. What are the phase/s of the
system at the four different states X1 to X4 ? [InChO-2008, 2 M]
fu;r nkc ij fud k; d kX1 ij rki c<+kusij X4 ij igq¡p rkgSA X1 lsX4 rd fud k; d hfHkUu&fHkUu izkoLFkkD;kgksxh?
Ans. X1 : Solid phase ; X2 : Solid-Liquid equilibrium phase ; X3 : Liquid - Gas equilibrium phase ; X4 : Gas Phase
X1 : Bks
l voLFkk; X2 : Bksl &nzo lkE;kLFkk; X3 : nzo&xSl lkE;koLFkk; X4 : xSl voLFkk
2.4 Show graphically the heating curve (Temperature vs. Time) for the process in the above problem (2.3) with
appropriate labeling physical states. [InChO-2008, 2 M]
mijksDr iz'u (2.3) es
afn;sx;siz
Ø e d ksm"eh; oØ ¼rki rFkkle; d se/;½ esaHkkSfrd voLFkkvksad kmfpr va
d u d hft ,A
[InChO-2008, 2 M]

Ans.

Corporate Office: CG Tower, A-46 & 52, IPIA, Near City Mall, Jhalawar Road, Kota (Raj.)-324005
Website : www.resonance.ac.in | E-mail : contact@resonance.ac.in
Toll Free : 1800 200 2244 | 1800 258 5555 | CIN: U80302RJ2007PLC024029
ADV-SCP - 64
Solution & Colligative Properties
2.5 If pressure is increased, the melting temperature of solid S will [InChO-2008, 1 M]
(a) Not change (b*) Increases (c) Decreases
;fn nkc c<+k;k t k;srksBksl S d k xy ukad gksxkA [InChO-2008, 1 M]
(a) d ks
bZifjorZu ugha (b*) c<+
rkgS (c) ?kVrk gS

2.6 With the help of Clapeyron equation for phase change (dp /dT = Hº /TV), find what happens to the volume
of the system at X2 on heating ? [InChO-2008, 1 M]
izkoLFkkifjorZu (dp /dT = Hº /TV) d sfy , Dy sifj;kWu lehd j.kd hlgk;rk lsKkr d hft , fd X2 ij xeZd jusij
fud k; d k vk;ru D;k gksxk \ [InChO-2008, 1 M]
Ans. Volume will increase on melting.
fi?ky usij vk;ru c<s+xk
2.7 What is the state of the system S, if it is heated in a sealed container above the critical temperature (304.2
K)? [InChO-2008, 1 M]
;fn fud k; S d kscan ik=k esaØ kafrd rki (304.2 K) lsÅ ij xeZfd ;k t k;s]rksfud k; d h voLFkk D;k gksxh \
[InChO-2008, 1 M]
Ans. Single Phase system called supercritical Fluid.
,d y voLFkk iz.kky hd kslqijfØ fVd y ¶y M (supercritical Fluid) d grsgSaA
2.8 What happens to the system S at X5, if pressure is changed slowly keeping temperature constant?
(Mark X in the correct box) [InChO-2008, 1 M]
(A) At a higher pressure, the system will become liquid.
(B) At a lower pressure, the system will become liquid.
(C*) The system will be in the same phase.
fud k; S d sX5 ij ;fn rki d ksfu;r j[krsgq, nkc d ksifjofrZr fd ;k t k,s]rksD;k gksxk \ [InChO-2008, 1 M]
(A) mPp nkc ij]fud k; nzo gksxkA
(B) fuEu nkc ij]fud k; nzo gksxkA
(C*) fud k; leku iz
koLFkkesagksxkA
2.9 Raoult’s law states that the partial pressure (p1) of a solvent over a solution is given by the vapor pressure
(p10) of the pure solvent times the mole fraction (X1) of the solvent. Derive an expression relating lowering of
vapor pressure with solute mole fraction. [InChO-2008, 1 M]
jkÅ YV fu;e d svuq l kj fd lhfoy ;u d sfoyk;d d kvka f'kd nkc (p1), 'kq) foy k;d d sok"i nkc (p10) rFkkfoyk;d d h
eksy fHkUu (X1) d sxq.kt d scjkcj gksrk gSA ok"i nkc esavoueu ,oafoy s; d seksy fHkUu d se/; lEcU/k fy f[k;sA
[InChO-2008, 1 M]
Ans. p1 = 1 . p1º = (1 – 2)p1º
 p1º – p1 = 2 . p1º or ;k p1 = 2 p1º
 p1 / p1º = 2

2.10 The vapor pressure of water at 20ºC is 17.54 mm of Hg and that of 10% (w/w) solution of an organic solute
at the same temperature is 16.93 mm of Hg. Calculate the molecular weight of the solute using the expression
derived in 2.9. [InChO-2008, 1 M]
20ºC ij ikuh d k ok"inkc 17.54 mm Hg gSvkS j d kcZfud foy s; d s10% (w/w) foy ;u d k leku rki ij ok"i nkc
16.93 mm Hg gS
A iz'u 2.9 esaLFkkfir fd ;sx;slEcU/k d ksiz;ksx esay srsgq, foy s; d k v.kqHkkj Kkr d hft ,A
[InChO-2008, 1 M]
Ans. 56
2.11 A solute is dissolved in two immiscible liquids in contact. Nernst’s distribution law states that at equilibrium,
the ratio of the concentration of the same molecular species in the two phases is constant (KD) at constant
temperature.
Solute A is present as monomeric species A in solvent S1 and associates to form An in solvent S2. Species A
and An are equilibrium with an equilibrium constant K.
(a) What is the concentration of A in solvent S2. if concentration of An is C2 ? [InChO-2008, 1 M]
(b) If the concentration of A in solvent S1 is C1, calculate the distribution constant KD of the solute.
[InChO-2008, 1 M]
Corporate Office: CG Tower, A-46 & 52, IPIA, Near City Mall, Jhalawar Road, Kota (Raj.)-324005
Website : www.resonance.ac.in | E-mail : contact@resonance.ac.in
Toll Free : 1800 200 2244 | 1800 258 5555 | CIN: U80302RJ2007PLC024029
ADV-SCP - 65
Solution & Colligative Properties
,d foy s; d ksnksvfeJ.kh; nz oksaesa?kksy kt krkgS
]uLVZforj.kd svuq
l kj lkE; ij nksleku vkf.od iz
t kfr;ksad hlkUnzrk
d k vuqikr nksuksaizoLFkkvksaesafu;r rki ij leku jgrk gSA
foy k;d S1 esafoy s; A ,d y d izt kfr gS;g foy k;d S2 esalaxq.ku ij An cukrk gSA izt kfr A rFkk An lkE; fu;rkad K
d slkFklkE;koLFkkesagksxhA
(a) A d h foy k;d S2 es alkUnzrk D;k gS];fn An d h lkUnzrk C2 gSa\ [InChO-2008, 1 M]
(b) ;fn A d h foy k;d S1 es alkUnzrk C1 gS]rksfoy s; d k forj.k fu;rkad KD Kkr d hft ,A [InChO-2008, 1 M]
Ans. (a) nA = An
[A] = n KC
2

C1
(b) KD =
nC
2

STAGE - V (INTERNATIONAL CHEMISTRY OLYMPIAD (IChO))


Problem 1 : Hard Water [18 Marks]
In Denmark the subsoil consists mainly of limestone. In contact with ground water containing carbon dioxide
some of the calcium carbonate dissolves as calcium hydrogen carbonate. As a result, such ground water is
hard, and when used as tap water the high content of calcium hydrogen carbonate causes scales in, for
example, kitchen and bathroom environments.
Carbon dioxide, CO2.is a diprotic acid in aqueous solution. The pKa-values at 0ºC are:
CO2(aq) + H2O(l)  HCO3-(aq) + H+(aq) pKa1 = 6.630
HCO3-(aq)  CO32-(aq) + H+(aq) pKa2 = 10.640
The liquid volume change associated with dissolution of CO2 may be neglected for all of the following
problems. The temperature is to be taken as being 0ºC.
Problem 1 : d Bks
j ty [18 Marks]
Msuekd Zes
avoeǹk(subsoil) esaeq[;r%y kbe LVksu gksrkgSaA d kcZuMªkbZv kWDlkbM d qN d SfY'k;e d kcksZusV oky sHkwfexr t y
d slEid Zes a;g d SfY'k;e gkbMª ks
t u d kcksZusV d s: i es
a?kq
y t krkgS
A ifj.kke Lo: i bl izd kj d kHkwfexr t y d Bksj gks
rk
gSrFkkt c blsuy d sikuhd s: i esami;ksx d jrsgSrksblesad SfY'k;e gkbMªkst u d kcks ZusV d smPp ?kVd d sd kj.k'kYd
;k [kqjpu d k fuekZ.k gksrk gSA mnkgj.k d sfy , jlksbZ?kj rFkk ckFk: e d sifjos'k esaA
d kcZu MkbZv kWDlkbM , CO2 t y h; foy ;u esa,d f}izksfVd vEy gSA 0ºC ij pKa-eku gS&
CO2(aq) + H2O(l)  HCO3-(aq) + H+(aq) pKa1 = 6.630
HCO3-(aq)  CO32-(aq) + H+(aq) pKa2 = 10.640
CO2 d h?kq
y u'khy rkd slkFklaxqf.kr nzo vk;ru ifjorZu d ksfuEu iz
'uksad sfy , ux.; ekukt kld rkgSA rki d ks0ºC
fy ;kt krkgSA
1.1 The total concentration of carbon dioxide in water which is saturated with carbon dioxide at a carbon dioxide
partial pressure of 1.00 bar is 0.0752M. Calculate the volume of carbon dioxide gas which can dissolved in
one litre of water under these conditions. The gas constant
R = 8.314 J mol- K-1 = 0.08314 L bar mol-1 K-1 [2 Marks]
t y ]t ks1.00 ckj d sd kcZu MkbZv kWDlkbM vkaf'kd nkc ij d kcZu MkbZv kWDlkbM d slkFk larÌr gksrk gS]d h d qy lkUnzrk
0.0752M gS ]d kcZu MkbZv kWDlkbM xSl d kvk;ru ifjd fyr d hft ,]ft ld ksbu ifjfLFkfr;ksad svUrxZ r ,d y hVj t y esa
?kksy k t k ld rk gSA xSl fu;rkad R = 8.314 J mol K = 0.08314 L bar mol K
- -1 -1 -1
[2 Marks]
Sol. c(CO2) = 0.0752 M n(CO2) = 0.0752 mol
The ideal gas equation ¼vkn'kZxS l lehd j.k½ : pV = nRT
1.00 bar × V = 0.0752 mol × 0.08314 L bar mol–1 K–1 × 273.15 K
V = 171 L
1.2 Calculate the equilibrium concentration of hydrogen ions and the equilibrium concentration of CO2 in water
saturated with carbon dioxide at a carbon dioxide partial pressure of 1.00 bar. [1 Mark]
1.00 ckj d sd kcZ
u MkbZ
v kDWlkbM vkfa'kd nkc ij d kcZ
u MkbZ
v kDWlkbM d slkFkla
rÌr t y es
agkbMª
kts u vk;ukasd hlkE; lkUnz
rk
rFkkCO2 d hlkE; lkUnz rkifjd fyr d hft ,A [1 Mark]
Sol. CO2(aq) + H2O(  )  HCO3– + H+(aq)

Corporate Office: CG Tower, A-46 & 52, IPIA, Near City Mall, Jhalawar Road, Kota (Raj.)-324005
Website : www.resonance.ac.in | E-mail : contact@resonance.ac.in
Toll Free : 1800 200 2244 | 1800 258 5555 | CIN: U80302RJ2007PLC024029
ADV-SCP - 66
Solution & Colligative Properties
[H+] = [HCO3–] = x and rFkk[CO2] + [HCO3–] = 0.0752 M

[H ] [HCO3 ] x2
Ka = 10–6.63 M = =
[CO 2 ] 0.0752M  x
[H+] = 0.000133 M and rFkk[CO2] = 0.071 M

1.3 Calculate the equilibrium concentration of hydrogen ions in a 0.0100 M aqueous solution of sodium hydrogen
carbonate saturated with carbon dioxide at a carbon dioxide partial pressure of 1.00 bar. [1 Mark]
1.00 ckj d sd kcZ
u MkbZ
v kW
DlkbM vkfa'kd nkc ij d kcZ
u MkbZv kW
DlkbM d slkFkla
rÌr lkfsM;e gkbMª
ks
t u d kcks
us
ZV d s0.0100
M t y h; foy ;u esagkbMªkst u vk;uksad hlkE; lkUnzrk ifjd fy r d hft ,A [1 Mark]
Sol. CO2(aq) + H2O(  )  HCO3– + H+(aq)
[CO2] = 0.0751 M and rFkk[HCO3–] = 0.0100 M

[H ] [HCO3 ] x  0.0100 M


Ka = 10–6.63 M = = 0.0751 M
[CO 2 ]
x = [H+] = 1.76 × 10–6 M
1.4 Calculate the equilibrium concentration of hydrogen ions in a 0.0100M aqueous solution of sodium carbonate
saturated with carbon dioxide at a carbon dioxide partial pressure of 1.00 bar. lgnore water dissociation
effects. [2 Marks]
1.00 ckj d sd kcZu MkbZ
v kW
DlkbM vkaf'kd nkc ij d kcZ
u MkbZ
v kW
DlkbM d slkFkla
rÌr lks
fM;e d kcks
Zus
V d s0.0100 M t yh;
foy ;u esagkbMªkst u vk;uksad hlkE; lkUnzrkifjd fy r d hft ,A [2 Marks]
2–
Sol. CO2(aq) + CO3 (aq) + H2O(  )  2HCO3 (aq) –
[HCO3 ] = 0.0200 M

CO2(aq) + H2O(  )  HCO3– + H+(aq)


[H ] [HCO3 ] x  0.0200 M
Ka = 10 –6.63
M= = 0.0751 M
[CO 2 ]
x = [H+] = 8.8 × 10–7 M
1.5 The solubility of calcium carbonate in water at 0oC is 0.0012 g per 100 mL of water. Calculate the concentration
of calcium ions in a saturated solution of calcium carbonate in water. [1 Mark]
0ºC ij t y es ad SfY'k;e d kcksZ
us
V d hfoys;rk0.0012 g izfr 100 mL t y gSA t y esad SfY'k;e d kcksZ
us
V d slarÌr foy ;u
esad SfY'k;e vk;uksad hlkUnzrkifjd fy r d hft ,A [1 Mark]
Sol. 0.0012 g CaCO3 in 100 ml of water
0.0012 g / 100.0872 g mol–1 = 0.000012 mol CaCO3 in 100 mL of water
[Ca2+] = 1.2 × 10–4 M
Sol. 100 ml t y es a 0.0012 g CaCO3
0.0012 g / 100.0872 g mol–1 = 100 ml t y es a0.000012 mol CaCO3
2+
[Ca ] = 1.2 × 10 M –4

The hard groundwater in Denmark is formed via contact of water with limestone in the subsoil which reacts
with carbon dioxide dissolved in the groundwater according to the equilibrium equation:
CaCO3(s) + CO2(aq) + H2O(l)  Ca2+ (aq) + 2HCO3- (aq)
The equilibrium constant, K, for this reaction is 10-4.25 M2 at 0oC.

Msuekd Zesad Bksj Hkwfexr t y voeǹk esay kbe LVksu d slkFk t y d slEid Zesavkuslscurk gS]t kslkE; lehd j.k d s
vuql kj vk| t y esa?kqy u'khy d kcZuMkbZvkWDlkbM d slkFkfØ ;kd jrkgSA
CaCO3(s) + CO2(aq) + H2O(l)  Ca2+ (aq) + 2HCO3- (aq)
mijksDr vfHkfØ ;k d sfy , 0°C ij lkE; fu;rkad K ,10-4.25 M2 gSA
1.6 Calculate the concentration of calcium ions in water in equilibrium with calcium carbonate in an atmosphere
with a partial pressure of carbon dioxide of 1.00 bar. [3 Marks]
1.00 ckj d sd kcZ
u MkbZ
v kW
DlkbM d svkaf'kd nkc ;qDr ,d ok;q
e.My es ad S
fY'k;e d kcks
us
ZV d slkFklkE; es
at y es
ad S
fY'k;e
vk;uksad hlkUnzrkifjd fy r d hft ,A [3 Marks]

Corporate Office: CG Tower, A-46 & 52, IPIA, Near City Mall, Jhalawar Road, Kota (Raj.)-324005
Website : www.resonance.ac.in | E-mail : contact@resonance.ac.in
Toll Free : 1800 200 2244 | 1800 258 5555 | CIN: U80302RJ2007PLC024029
ADV-SCP - 67
Solution & Colligative Properties

[Ca 2 ] [HCO3 ]2
Sol. K= = 10–4.25 M2 and rFkk 2 [Ca2+] = [HCO3–]
[CO2 ]

4[Ca 2 ] 3
= 10–4.25 M2 [Ca2+] = 1.02 × 10–2 M
0.0751 M

1.7 A 0.0150M solution of calcium hydroxide is saturated with carbon dioxide gas at a partial pressure of 1.00
bar. Calculate the concentration of calcium ions in the solution by considering the equilibrium equation given
above in connection with problem 1.6. [3 Marks]
dSfY'k;e gkbMªkWDlkbM d k0.0150 M foy;u 1.00 ckj d svka f'kd nkc ij d kcZu MkbZv kW
DlkbM xSl d slkFklarÌr gks
rkgS
A
iz
'u 1.6 esamijks Dr nhxbZlkE; lehd j.kd kvoykds u d jrsgq , foy;u esad fSY'k;e vk;ukasd hlkUnz
rkifjd fyr d hft ,A
[3 Marks]
Sol. c(Ca(OH)2) = 0.015 M
OH–(aq) + CO2(aq)  HCO3–(aq)
All hydroxide has been consumed [K = 107.37 M–1]
From problem 1.6 we found that the maximum possible calcium ion concentration is smaller, i.e.
precipitation of CaCO3
[Ca2+] = 1.02 × 10–2 M
Sol. c(Ca(OH)2) = 0.015 M
OH–(aq) + CO2(aq)  HCO3–(aq)
leLr gkbMªksDlkbMksad kmiHkksx gkst krkgSA [K = 107.37 M–1]
iz'u 1.6 lsgesaik;k t krk gSfd vf/kd re lEHko d SfY'k;e vk;u lkUnzrk d e gksrh gSvFkkZr~CaCO3 d kvo{ksi.k
[Ca2+] = 1.02 × 10–2 M
1.8 The calcium hydroxide solution referred to in problem 1.7 is diluted to twice the volume with water before
saturation with carbon dioxide gas at a partial pressure of 1.00 bar. Calculate the concentration of calcium
ions in the resulting solution saturated with CO2. [2 Marks]
iz'u 1.7 lslEcfU/kr d SfY'k;e gkbMªkWDlkbM foy ;u d ks1.00 ckj d svkaf'kd nkc ij d kcZu MkbvkWDlkbM xSl d slkFk
larÌrrklsiwoZt y lsvk;ru d ksnqxqukruqfd ;kt krkgSA CO2 d slkFklarÌr ifj.kkehfoy ;u esad SfY'k;e vk;uksad h
lkUnzrkifjd fyr d hft ,A [2 Marks]
Sol. c(Ca(OH)2) = 0.0075 M
From problem 1.6 we found that the maximum possible calcium ion.
concentration we can have is 1.02 × 10–2 M, i.e. no precipitation
[Ca2+] = 0.75 × 10–2 M
Sol. c(Ca(OH)2) = 0.0075 M
iz'u 1.6 lsgesaik;k t krk gSfd vf/kd re lEHko d SfY'k;e vk;u d h lkUnzrk 1.02 × 10–2 M gksrh gSvFkkZr~
vo{ksi.k ugh gksrk gSA [Ca2+] = 0.75 × 10–2 M
1.9 Calculate the solubility product constant for calcium carbonate from the data given above. [3 Marks]
mijksDr fn;sx;svkad M+ksalsd SfY'k;e d kcksZusV d sfy , foy s;rk xq.kuQ y fu;rkad ifjd fy r d hft ,A [3 Marks]
[Ca 2 ] [HCO3 ]2 [Ca 2 ] [HCO3 ]2 [CO 32 – ] [H ] K spK a1
Sol. K= = × 2– = K
[CO2 ] [CO2 ] [CO 3 ] [H ] 
a2

Ksp = 10–8.26 M2

Corporate Office: CG Tower, A-46 & 52, IPIA, Near City Mall, Jhalawar Road, Kota (Raj.)-324005
Website : www.resonance.ac.in | E-mail : contact@resonance.ac.in
Toll Free : 1800 200 2244 | 1800 258 5555 | CIN: U80302RJ2007PLC024029
ADV-SCP - 68

You might also like